Sie sind auf Seite 1von 92

PATHOLOGY

PREVIOUS
BOARD EXAM
QUESTIONS

Prepared by
Julius Matthew M.Luzana

PATHO BOARD EXAM QUESTIONS 1


MCU-FDTMF Medical Foundation
College of Medicine
Department of Pathology

MULTIPLE CHOICE:

1. The hallmark of acute inflammation is:


A. Increased blood flow
B. Rubor, calor, dolor increased permeability of microcirculation
C. Increased vascular permeability
D. Vascular stasis

2. The most efficient bactericidal compound present in neutrophils is the:


A. Arachidonic acid metabolites C. Lysosomal enzymes
B. H202-MPO-Halide system D. Hydrogen peroxide

3. Leukotrienes are derived from arachidonic acid through the help of this enzyme:
A. Phospholipase C. Cyclooxygenase
B. Lipooxygenase D. Lipoxins

4. The hallmark of chronic inflammation is:


A. Increased permeability of microcirculation
B. Migration of macrophages at site of injury
C. Tissue destruction
D. Mononuclear cell infiltration

5. An example of tumor suppressor gene:


A. p53 C. ras
B. C-myc D. bcr

6. A lymph node biopsy was diagnosed as tuberculosis. This type of necrosis is


characteristic of:
A. Coagulative C. Caseous
B. Liquefactive D. Gangrenous

7. Which of the following substances accumulates in atherosclerotic blood vessels?


A. Fatty acids C. Tryglicerides
B. Cholesterol esters D. Phospholipids

8. "Heart failure cells" found in the lungs in Chronic Passive Congestion are actually….
A. Lymphocytes C. Neutrophils
B. Eosinophils D. Macrophages

9. The outcome of thrombosis wherein the thrombi tends to GROW due to deposition of
additional platelets, fibrin and red blood cells:
A. Resolution C. Propagation
B. Organization D. Recanalization

10. Most common cause of death in burn patients who got into shock is:
A. Sepsis C. Dehydration
B. CHF D. Hemorrhage
11. This type of hypersensitivity reaction is mediated by IgE:
A. Type I C. Type III
B. Type II D. Type IV

12. One of the three major components of acute inflammation include:


A. Alteration in vascular caliber that leads to a decrease in blood flow
B. Structure changes in the microvasculature permitting plasma proteins and leukocytes to leave the circulation
C. Tissue destruction
D. Healing by connective tissue

13. Heat and redness in acute inflammation is due to the following pathogenetic
mechanism:
A. Increased vascular permeability C. Stasis
B. Increased blood flow D. Leukocytic margination

14. The uterine endometrial lining thickens during the proliferative phase of the
menstrual cycle. This is due to:
A. Hypertrophy C. Hyperplasia
B. Dysplasia D. Metaplasia

15. Which of the following disorders is most likely to be associated with an exudates
PATHO BOARD EXAM QUESTIONS 2
rather than a transudate?
A. Congestive heart failure C. Suppurative inflammation
B. Nephrotic syndrome D. Peritonitis

16. Which of the following conditions will NOT cause edema?


A. Sodium retention C. Obstruction of lymph flow
B. Increased hydrostatic pressure D. Increased oncotic pressure

17. The role of histamine in acute inflammatory response include:


A. Platelet release and aggregation
B. Increased vascular permeability of the venules
C. Increased vascular permeability of the arterioles
D. Membrane lysis

18. The central figure in chronic inflammation is the Macrophage because of its role in:
A. Breakdown of collagen and fibronectin
B. Production of 02 and Nitric oxide metabolites
C. Emigration from the blood stream to site of injury
D. Inhibition by cytokines and oxidized lipids

19. A biopsy of the stomach reveal an area of malignant change characterized by


varisized, compactly arranged neoplastic glands lined by moderately differentiated
cells. Which of the following terms refer to malignant tumor of glandular epithelium?
A. Sarcoma C. Teratoma
B. Choristoma D. Adenocarcinoma

20. Which of the following statements in NOT true of red infarcts?


A. It occurs in organs with double blood supply
B. Commonly found in the heart and kidneys
C. Seen in venous occlusion with infarction
D. Seen in organs with well developed anastomosis

21. Which of the following conditions is a complication of both direct injuries to the lungs
and systemic disorders?
A. Atelectasis C. Diffuse alveolar damage
B. Pulmonary edema D. Pulmonary congestion

22. Which of the following conditions of the lung is characterized by abnormal permanent
enlargement of the air space distal to the terminal bronchiole accompanied by
destruction of their walls without obvious fibrosis?
A. Emphysema C. Chronic bronchitis
B. Bronchial asthma D. Bronchiectasis

23. A spectrum of immunologically mediated predominantly interstitial lung disorders


caused by intense often prolonged exposure to inhaled organic dusts and
occupational antigens is:
A. Pulmonary alveolar proteinosis
B. Bronchiolis obliterans-organizing pneumonia
C. Desquamative interstitial pneumonitis
D. Hypersensitivity pneumonitis

24. An acute febrile respiratory disease characterized by patchy inflammatory changes


in the lungs confined to the alveolar septa and pulmonary interstitium:
A. Primary atypical pneumonia C. Bronchopneumonia
B. Lobar pneumonia D. Lobular bronchopneumonia

25. A systemic disease of unknown cause characterized by non-caseating granuloma in


many tissues and organs particularly in the hilar lymph nodes, lungs, eye, and skin
is:
A. Silicosis C. Idiopathic pulmonary fibrosis
B. Sarcoidosis D. Asbestosis

26. Most common benign tumor of the lung discovered incidentally as a coin lesion on
routine X-Ray examination is:
A. Bronchial carcinoids C. Hamartoma
B. Thymoma D. Bronchogenic cyst

27. Most common cause of aspiration pneumonia is hospitalized patients:


A. Streptococcus pneumonia C. Haemophilus influenzae
B. Staphylococcus aureus D. Pseudomonas aeroginosa

28. Most common type of asthma that begins in childhood and triggered by
PATHO BOARD EXAM QUESTIONS 3
environmental antigens such as dust, pollens and food is:
A. Non-atopic asthma C. Atopic asthma
B. Drug induced asthma d. Occupational Asthma

29. Most common manifestation of asbestos exposure often containing calcium seen on
the anterior and posterior aspect of the parietal pleura:
A. Asbestos body C. Ferruginous bodies
B. Plaques D. Amphibole

30. Which of the following syndrome is characterized by the appearance of proliferative


glomerulonephritis and necrotizing hemorrhagic interstitial pneumonitis?
A. Goodpasture syndrome C. Karteagner syndrome
B. Paraneoplastic syndrome D. Lambert-Eaton myasthenic syndrome

31. Most common location of secondary tuberculosis in the lung:


A. Hilar area C. Peripheral
B. Base D. Apex

32. A fatal complication of ruptured aortic aneurysm and vascular trauma:


A. Chylothorax C. Hydrothorax
B. Hemothorax D. Empyema

33. Mang Juanito, 60-year-old man, has been smoking for many years, he experienced
chronic cough and weight loss for the past 6 months, no fever no nausea and
vomiting. He had one bout of hemoptysis and went to the ER for consultation. X-ray
shows a 6 cm mass on the medial upper lobe. Bronchoscopy shows a mass on the
segmental bronchus. Which of the following cytologic findings is likely to be found in
this patient?
A. Presence of acid fast organism on sputum exam
B. Presence of malignant squamous cells in sputum
C. Presence of numerous necrotic debris and inflammatory cells in sputum
D. Presence of reactive mesothelial cells in pleural fluid exam

34. Mang Juan, 65-year-old male with a history of smoking with productive cough for
many years came in at the emergency room with difficulty of breathing. Chest X-ray
shows prominent blood vessels and large heart. Patient went into cardiorespiratory
arrest. Resuscitation done but to no avail. Histologic section taken from the lung at
autopsy shows enlargement of the mucus secreting glands of the trachea and
bronchi. What is the most likely diagnosis?
A. Emphysema C. Chronic bronchitis
B. Bronchial asthma D. Bronchiectasis

35. Mang Tony, 55-year-old man has no major medical problems in the past year, 4
months prior to consultation the patient experience malaise and weight loss of 10 kg.
The patient is a non-smoker, no fever, no difficulty of breathing. Chest X-ray shows a
multiple solid nodules scattered throughout the lung fields. What is the most likely
diagnosis?
A. Bronchogenic carcinoma C. Malignant mesothelioma
B. Metastatic carcinoma D. Bronchioalveolar carcinoma

36. Mang Tomas, 45-year-old man had an episode of myocardial infarction he was
brought to the emergency room and was intubated. Upon intubation he suffers
aspiration of gastric contents. For the next 5 days he develops non-reproductive
cough and fever. Chest X-ray shows a 3 cm mass in the right lung with elevated air
fluid level. What is the most likely diagnosis?
A. Bronchopulmonary sequestration C. Atelectasis
B. Bronchiectasis D. Lung abscess

37. Hemodynamic pulmonary edema seen in congestive heart failure is due to:
A. Increased oncotic pressure C. Increased hydrostatic pressure
B. Decreased interstitial osmotic pressure D. Increased albumin

38. Sudden death in patients with pulmonary embolism is caused by:


A. Blockage of blood flow through the lungs
PATHO BOARD EXAM QUESTIONS 4
B. Atelectasis
C. Development of chronic obstructive pulmonary disease
D. Development of pulmonary hypertension

39. A lymphohematogenous dissemination of pulmonary tuberculosis would give rise to:


A. Vacitary fibrocaseous tuberculosis C. Tuberculous bronchopneumonia
B. Potts disease D. Miliary tuberculosis

40. The plausible mechanism in the development of emphysema is:


A. Increase release of elastase in neutrophils and macrophages
B. Increase alpha-1 anti-trypsin enzyme
C. Presence of obstruction of a tumor of foreign body with concomitant infection
D. Increase responsiveness of the bronchial tree

41. The major cause of celiac sprue is:


A. Bacterial infection
B. Deficiency of Vitamin B12
C. Hypersensitivity to gliadin fraction of wheat gluten
D. Lymphatic obstruction

42. Which of the following statements is NOT a characteristic of Whipple's Disease?


A. Caused by a gram-positive actinomycete
B. It is a systemic condition that may involve any organ of the body
C. There is an increased incidence of small intestinal adenocarcinoma
D. Villi expanded by the dense infiltrate of macrophages imparts a shaggy gross appearance to the intestinal
mucosal surface

43. The histologic criterion for the diagnosis of acute appendicitis is based on finding this
type of cell infiltrating the muscularis propia:
A. Histiocytes C. Lymphocytes
B. Monocytes D. Neutrophils
44. Which of the following statements is NOT a characteristic of Crohn's Disease?
A. Associated with extraintestinal manifestations like migratory polyarthritis and ankylosing spondylitis
B. Granulomatous reaction in all layers of involved intestine
C. Response to surgery is good
D. Skip areas are seen between involved segments of intestine

45. Disaccharidase deficiency is classified under which of the following major causes of
malabsorption syndrome:
A. Defective Intraluminal digestion
B. Lymphatic obstruction
C. Primary mucosal cell abnormalities
D. Reduced small intestinal surface area

46. In ulcerative colitis, inflammation is limited to the:


A. Mucosa C. Serosa
B. Submucosa D. Transmural layer
47. Large areas of hemorrhagic green ulceration of the mucosa and green-black
necrosis through the wall, extending to the serosa of the appendix is most likely
associated to which of the following stages of acute appendicitis?
A. Acute gangrenous appendicitis C. Acute suppurative appendicitis
B. Early acute appendicitis D. Ruptured acute appendicitis

48. Leon, 25-year-old male complained of intermittent diarrhea and lower abdominal
pain. An upper G-I series showed segmental narrowing in the ileum. Microscopic
findings of the lesion showed inflammation from the mucosa to the serosa with the
presence of non-caseating granulomas. The most likely diagnosis is:
A. Celiac sprue C. Crohn's disease
B. Tropical sprue D. Whipple's disease

49. Henry, 45-year-old, male suffers from episodic abdominal bloating with flatulence
and explosive diarrhea after attending a week-long community celebration of the
dairy industry during the rest of the year, he does not consume milk shakes or ice
cream and is not symptomatic. Which of the following conditions best accounts for
these findings?
A. Celiac sprue C. Disaccharidase deficiency
B. Ulcerative colitis D. Whipple's disease

50. Josie, 40-year-old female, has a total serum bilirubin concentration of 8. 9 mg/dl and
a direct bilirubin level of 6.8 mg/dl. The serum Alanine aminotrasferase (ALT) level is
125 U/L, and the aspartate aminotransferase (AST) level is 108 U/L. A liver biopsy
shows histologic findings characteristic for Sclerosing cholangitis. Which of the
PATHO BOARD EXAM QUESTIONS 5
following gastrointestinal tract disease is most likely to coexist in this patient?
A. Celiac sprue C. Tropical sprue
B. Ulcerative colitis D. Whipple's disease

51. A middle-aged male complains of food sticking somewhere between mouth and
stomach. Monometry reveals a peristalsis. The most possible diagnosis is:
A. Stenosis due to severe gastroesophageal reflux
B. Paraesophageal hiatal hernia
C. Achalasia
D. Zenker's diverticulum

52. Joshua, 25-year-old, medical student developed sharp epigastric pain relieved by
eating. An upper G. I. series demonstrated a 1 cm gastric ulcer. Which of the
following features is not compatible with the clinical findings?
A. The size of the ulcer does not differentiate a benign from malignant ulcer.
B. Heaping-up margin is rare in the benign ulcer but characteristic of malignant lesion
C. The base of the peptic ulcer is smooth and clean
D. Ulcerative lesion in the greater curvature is more likely to be classic peptic ulcer

53. Joan, 25-year-old, female, developed diarrhea, abdominal pain, and rectal bleeding.
Sigmoidoscopy showed numerous ulcers. Idiopathic inflammatory bowel disease
was considered. Which of the following findings is more compatible to Crohn's
Disease than to ulcerative colitis?
A. Crypt abscess formation deep in mucosa
B. Increase incidence of adenocarcinoma of colon
C. Pseudopolyp formation between ulcers
D. Presence of granulomas in the colonic wall

54. Cesar, 35-year-old male had a history of heart burn, regurgitation of sour brash,
dyspnea, burning esophageal pain and slowly but progressive dysphagia. He was
admitted to the emergency room because of lobar pneumonia. The most possible
cause of his pneumonia is due to aspiration and can be attributed to which of the
following conditions?
A. Esophageal atresia C. Sliding hiatal hernia
B. Reflux esophagitis D. Esophageal diverticulum

55. Imelda, 6-year-old, girl, had a blocky, reddish brown rash and was treated with
aspirin, she developed fatty change of the liver. The most likely diagnosis is:
A. Subacute Sclerosing panencephalitis
B. Varicella-Zoster infection
C. Reye's syndrome
D. Poliomyelitis

56. Ronnie, 45-year-old, alcoholic went on a binge for 2 weeks. He was found comatose
and in liver failure. His liver at autopsy showed several hepatocytes with hyaline
Mallory bodies in the cytoplasm. This finding is most typical of:
A. Alcoholic hepatitis C. Hepatic steatosis
B. Alcoholic cirrhosis D. Hepatocellular tumor

57. Fely, 18-year-old, female, presents with abdominal pain localized to the right lower
quadrant, nausea and vomiting, mild fever, and an elevation of the peripheral
leukocyte count to 17x109/L. Examination of the surgically resected appendix is most
likely to reveal:
A. An appendix with normal appearance
B. Neutrophils within the muscular wall
C. Lymphoid hyperplasia and multinucleated giant cells within the muscular wall
D. A dilated lumen filled with mucus

58. Nelia, 38-year-old, female, complains of fatigue and pruritus. She is found to have
high serum alkaline phosphatase and slightly elevated serum bilirubin levels, and
serum antimitochondrial autobodies are present. A liver biopsy reveals a marked
Lymphocytic infiltrate in the portal tracts. Occasional granulomas are also seen. The
most likely diagnosis is:
A. Primary Sclerosing cholangitis C. Viral hepatitis B infection
B. Primary biliary cirrhosis D. Impacted gallstone

59. The findings of multiple, pale, yellow, hard round stones within the gallbladder is
NOT associated with which of the following?
A. Oral contraceptive C. Obesity
B. Biliary infection D. Hyperlipidemia syndromes

60. Dilated sinusoids and irregular cystic spaces filled with blood within the liver which
PATHO BOARD EXAM QUESTIONS 6
may rupture leading to massive intra-abdominal hemorrhage, are most commonly
associated with:
A. Salicylates C. Estrogen
B. Anabolic steroids D. Acetaminophen

61. Transmural infarcts commonly involve these three arterial vessels of the heart. The
LEAST affected one is the:
A. Left anterior descending C. Right coronary
B. Left circumflex D. Right circumflex
62. The heat failure cells are by nature:
A. Alveolar lining cells C. Polymorphonuclear cells
B. Macrophages D. Eosinophils

63. Angina that is unstable is clinically described as:


A. Transient C. Precipitated by physical stress
B. Relieved by rest D. Progressive

64. This organ is not prominent affected in right-sided failure:


A. Liver C. Kidneys
B. Lungs D. Brain

65. A cardiac mass form an infant on microscopic examination revealed "spider cells".
What type of tumor is this?
A. Leiomyoma C. Rhabdomyoma
B. Myxoma D. Fibroma

66. Cases of patent ductus arteriosus that are not isolated are commonly associated
with the following conditions, Except:
A. VSD C. Coarctation
B. ASD D. Pulmonary stenosis

67. Histopathology examination of the heart of a patient who died of AMI showed
coagulative necrosis, edema, hemorrhage and neutrophilic infiltrates. How old is the
infarct?
A. 30 minutes C. 1 day
B. 12 hours D. 1 week

68. Noli, 15-year-old, student, had sore throat and a week later developed swollen joints
and a murmur in the aortic valve area. What is the diagnosis?
A. Infective endocarditis C. SLE
B. Rheumatic heart disease D. Diphtheria

69. Daniel, 35-year-old man, who recently had an infarct developed a loud pericardial
friction rub. This is due to:
A. Another infarct C. Serous pericarditis
B. Purulent pericarditis D. Fibrinous pericarditis

70. George, 40-year-old male, died of sepsis with DIC. Upon autopsy, small masses of
fibrin thrombi were seen in the heart valves. How do you interpret this?
A. Patient had infective endocarditis C. Patient had NBTE
B. Patient had rheumatic heart disease D. It is normal finding

71. The bile salts aid in the emulsification of dietary fats in the intestines. They are
composed of bile acids that have been conjugated with:
A. Albumin C. Glucoronic acid
B. Amino acid D. Glucose

72. The milky appearance of serum after fat ingestion is called post prandial lipemia. The
lipemic appearance is caused by the presence of:
A. Cholesterol C. Fatty acids
B. Chylomicrons D. Phospholipids

73. Which of the following apolipoproteins, when present in an increased concentration,


would be associated with a decreased risk of coronary artery disease?
A. Apo A-I C. Apo B-100
B. Apo B-48 D. Apo C-II

74. Which type of inflammation is most characteristic of acute rheumatic fever?


A. Myocarditis C. Pancarditis
B. Pericarditis D. Endocarditis

75. This disease of the heart presents on early cyanosis:


PATHO BOARD EXAM QUESTIONS 7
A. Atrial septal defect C. Patent ductus arteriosus
B. Truncus arteriosus D. Atrioventricular septal defect

76. Which of the following statements about heart failure is true?


A. Right sided failure may result in hepatomegaly
B. Cor pulmonale usually is due to severe pulmonic stenosis
C. The clinical manifestation of heart failure most commonly reflect right-sided failure
D. Dyspnea is a result of blood stasis in the extremities

77. Which of the following diseases results from a familial absence of high-density
lipoprotein?
A. Krabbe's C. Gaucher's
B. Tangier D. Tay-Sachs

78. The enzyme that exists chiefly in skeletal muscle, heart and brain is grossly active
muscular dystrophy and rises early in myocardial infarction is:
A. Lipase C. Transminase
B. Lactate dehydrogenase D. Creatine kinase

79. Juanito, 34-year-old was found to have total cholesterol of 225 mg/dl and an HDL
cholesterol of 83 mg/dl. Based on these results, this individual:
A. Is a borderline high risk of coronary heart disease
B. Should be counseled to modify his diet to reduce his total cholesterol
C. Should be consulted to see his physician immediate to follow-up testing
D. Is probably not borderline high risk for coronary heart disease.

80. A blood specimen is drawn for lipoprotein phenotyping. The test results obtained are:
1. Triglycerides - 235 mg/dl (NV 40-164 mg/dl)
2. Total cholesterol - 190 mg/dl (NV less than 200 mg/dl)
3. Prebeta - lipoprotein fraction increased
4. Beta- lipoprotein fraction normal
5. Chylomicrons present
6. Serum appearance milky
The best explanation for these results would be that the individual exhibited characteristic of:
A. A normal individual C. Type II hyperlipoproteinemia
B. A non-fasting serum protein D. Type IV hyperlipoproteinemia

81. Hemoglobin Bart's is composed of:


A. Four alpha chains C. Four gamma chains
B. Four beta chains D. Two alpha, two beta chains

82. Howell-Jolly bodies are composed of:


A. DNA C. Iron
B. RNA D. Mitochondria

83. The most mature cell that can undergo mitosis is the:
A. Myeloblast C. Metamyelocyte
B. Promyelocyte D. Myelocyte

84. Vasodilation and bronchoconstriction would be associated with which blood cell:
A. Eosinophilic C. Basophils
B. Neutrophils D. Monocytes

85. Lymphocytes that produce immunoglobulins in response to antigenic stimulation are


designated:
A. B Lymphocytes C. Plasma cells
B. T lymphocytes D. Thymocytes

86. Toxic granulation is most commonly observed as a Cytoplasmic inclusion of:


A. Lymphocytes C. Monocytes
B. Eosinophils D. Neutrophils

87. Which of the following organs is NOT a site for hematopoiesis in the fetus?
A. Liver C. Spleen
B. Bone marrow D. Kidney

88. In early infancy the most numerous cells of the bone marrow are:
A. Erythroblasts C. Granulocytic precursors
B. Lymphocytes D. Histiocytes-monocytes

89. Which cell classification is described by the following: Second most numerous cell in
PATHO BOARD EXAM QUESTIONS 8
the blood; usually small and round; intensely blue cytoplasm; and nucleus with
clumped dark purple chromatin?
A. Monocyte C. Lymphocyte
B. Null cell D. Plasmacyte

90. Which of the following is most variable in normal marrow?


A. Differential count of 500 cells C. M:E ratio
B. Cellularity D. Iron stage

91. Which of the following is not a crucial area of RBC survival and function?
A. Integrity of RBC cellular membrane C. Cell metabolism
B. Intravascular hemolysis D. hemoglobin structure

92. Which of the following groups of abnormal hemoglobins are unable to transport or
deliver oxygen?
A. Carboxyhemoglobin and methemoglobin
B. Methemoglobin and fetal hemoglobin
C. Carboxyhemoglobin, sulfhemoglobin, and fetal hemoglobin
D. Carboxyhemoglobin, methemoglobin and sulfhemoglobin

93. Production of primary granules ceases and production of secondary granules


commences with what cell stage?
A. Myelocyte C. Myeloblast
B. Promyelocyte D. Metamyelocyte

94. A decreased in which of the following laboratory results is NOT a usual diagnostic
criterion for anemia?
A. Hemoglobin C. hematocrit
B. Platelet count D. RBC count

95. What is diagnostic value of reticulocyte count in the evaluation of anemia?


A. Determines response and potential of the bone marrow
B. Determines compensation mechanisms for anemia
C. Determines the corrected RBC count after the calculation
D. Determines the potential sampling error for RBC count

96. A bone marrow has large cells that have eccentric pyknotic nuclei. The cytoplasm of
these cell stains very pale and has a striated appearance:
A. Mega karyoblasts C. Gaucher's cells
B. Reed-Sternberg cells D. Large myeloblasts

97. Which of the following is NOT a factor to be evaluated in the interpretation of a bone
marrow aspirate smear?
A. Maturation of red and white blood cells series
B. M:E ratio
C. Type and amount of hemoglobin
D. Estimate of bone marrow activity

98. Lito 27-year-old has a total WBC count of 4x109/L. The differential count is as
follows: Neutrophils - 28 (NV . 56)
Lypmhocytes - 65 (NV .34)
Band - .02 (NV .2-)
Monocytes - .5 (NV - .4)
Which of the following statements is true?
A. The percentage of lymphocytes is normal
B. The absolute number of lymphocytes is low
C. There is an absolute lymphocytosis
D. There is a relative lymphocytosis

99. The following conditions favors diagnosis of CML rather than a leukomoid reaction,
EXCEPT:
A. Absence of eosinophils and basophils in the peripheral blood
B. Low LAP score with myeloblasts through segs in the peripheral blood
C. Ph chromosome
D. Enlarge spleen

100. The principal defect in chronic granulomatous disease is in:


A. Chemotaxis C. Phagocytosis
B. Lysosomal function D. Production of oxygen and radical

PATHO BOARD EXAM QUESTIONS 9


Reference : Pathologic Basis of Disease 7th Edition

B 1 The central factor in the pathogenesis of irrevsible cell injury would be R 9 0.33
A. mitochondrial dysfunction
B. membrane damage
C. cytoskeletal abnormalities
D. free radical formation

C 2 Which would be the consequence(s) of the occurance of mitochondrial I 12 0.5


permeability transition ?
A. promotes ATP production
B. increase in the number of mitochondria
C. cel death
D. A,B,C

B 3 The presence of chalky, white areas in the adipose tissue in the I 18 0.5
abdominal cavity would be interpreted to be associated with:
A. activation of pancreatic amylases into the peritoneal cavity
B. combination of fatty acids released with calcium
C. histologic reaction of numerous lipophages
D. A, B, C

A 4 The ultimate mechanism of sudden cardiac death is: R 564 1


A. lethal arrythmia
B. ischemic cardiomyopathy
C. dilated cardiomyopathy
D. aortic valve stenosis

D 5 Myxomatous degenerationof the mitral valve is associated with: R 564 0.5


A. prolapse of the mitral valve into the left ventricle
B. ischemic areas in the left atrial septum
C. fussion of the mitral commissures
D. complication of mitral insufficiency

B 6 Which is NOT a complication or consequence of myocardial infarction? I 562 0.5


A. cardiogenic shock with involvement of more than 40% of left ventricle
B. rupture of the ventricular septum leading to cardiac tamponade
C. mural thrombus following stasis and endocardial change
D. post-infarct mitral regurgitation following ischemic dysfunction
of a papillary mscle

D 7 A consequence of the following complication(s) would lead to chronic I 565 0.5


cor pulmonale:
A. metabolic acidosis
B. kyphoscoliosis
C. extensive pulmonary arteritis
D. A, B, C

A 8 In peripartum cardiomyopathy, the systolic dysfunction of the patient is P 580 0.25


associated or explained by:
A. large and flabby heart
B. massive ventricular hypertrophy without dilatation
C. impaired ventricular filling during diastole
D. assymmetric septal hypertrophy

C 9 The most characteristic histologic lesion of acute pancreatitis I 905 0.5


A. Neutrophilic infiltration
B. Hemorrhage
C. Fat necrosis
D. Necrosis of parenchyma

A 10 Most important environmental factor for pancreatic cancer R 910 0.5


A. Smoking

PATHO BOARD EXAM QUESTIONS 10


B. Hyperlipoprotenemia
C. Alcoholism
D. Viral infection

C 11 Ritter disease is also known as R 367 0.3


A. gas gangrene
B. Hidradenitis suppurativa
C. Staphylococcal scalded skin syndrome
D. Quinsy sore throat

B 12 Elephantiasis is associated with R 398 0.5


A. Onchocerca volvulus
B. Wuchereria bancrofti
C. Ancylostoma duodenale
D. Any of the above

D 13 Fungi within small cysts in the Virchow-Robin perivascular space is I 380 0.5
characteristic of
A. Mucor
B. Aspergillus
C. Pneumocystis carinii
D. Cryptococcus neoformans

D 14 Differential diagnosis of a child who presents with fever P 0.33


A. malaria 389
B. dengue 383
C. poliovirus infection 373
D. Any of the above

A 15 Consequence of lead exposure R 422 0.33


A. memory loss
B. lung fibrosis
C. asthma
D. cancer of nasal cavity

A 16 Indoor air pollutant R 418 0.5


A. carbon monoxide
B. sulfur dioxide
C. ozone
D. acid aerosol

D 17 A patient with hypochromic, microcytic anemia is deficient in I 452 1


A. zinc
B. copper
C. selenium
D. iron

C 18 A child came in at the out-patient department because of generalized P 438 0.5


edema accompanied by easily pluckable hair, listlessness, and loss
of appetite. Most likely she is suffering from
A. anorexia
B. marasmus
C. kwashiorkor
D. bulimia

C 19 Traveler's diarrhea is associated with what organism R 808 0.5


A. Salmonella typhi
B. Campylobacter jejuni
C. Escherichia coli
D. Shigella

A 20 Astler-Coller classification when the malignant glands in the colon are I 835 0.5
limited to the mucosa
A. A1

PATHO BOARD EXAM QUESTIONS 11


B. A2
C. B
D. None of the above

B 21 Grossly appears as a blind pouch on the antimesenteric side of the I 805 1


small bowel
A. Hirschprung disease
B. Meckel diverticulum
C. Celiac sprue
D. None of the above

C 22 Information to elicit in the clinical history of a patient with P 799 0.5


gastric carcinoma
A. fondness for barbecued foods
B. alcholism
C. presence of relatives with gastric carcinoma
D. Any of the above

D 23 Alcoholic patients may manifest P 0.5


A. longitudinal tears in the esophagogastric junction 779
B. as squamous cell carcinoma of the esophagus 783
C. as acute gastritis 789
D. Any of the above

C 24 Schiller-Duval body is associated R 1076 0.33


A. thecoma-fibroma
B. dysgerminoma
C. yolk sac tumor
D. granulosa-theca tumor

D 25 Which is an ovarian cyst? I 1


A. serous cyst 1069
B. follicular cyst 1066
C. luteal cyst 1066
D. Any of the above

B 26 Differential diagnosis of a patient with sessile masses projecting into P 1058 0.5
the endometrial cavity
A. papillary hidradenoma
B. polyp
C. lichen sclerosus
D. Any of the above

B 27 A 30 year old female complained of loss of consciousness. History revealed P 926 0.5
that the episodes were precipitated by fasting. Laboratory exam at the time
of attack showed glucose level at less than 50mg/dl. This is usually asso-
ciated with
A. Gastrinoma
B. Insulinoma
C. Glucagonoma
D. Somatostatinoma

C 28 Which of the following is NOT a consequence of oligohydramnios I 465 0.5


A. Pulmonary hypoplasia
B. Breech Presentation
C. Amniotic bands
D. Clubfeet

D 29 A newborn infant was born with a hemolytic disease due to RH incompa- P 474 1
tibility. What is the most serious threat in this disorder.
A. Anasarca
B. Liver Cirrhosis
C. Pancreatic insufficiency
D. Kernicterus

PATHO BOARD EXAM QUESTIONS 12


B 30 The most characteristic renal finding in diabetes mellitus I 923 0.5
A. Hyaline arteriolosclerosis
B. Nodular glomerulosclerosis
C. Acute pyelonephritis
D. Diffuse glomerulosclerosis

A 31 Most frequent type of hyperfunctioning pituitary adenoma R 1125 0.25


A. Lactotroph adenoma
B. Corticotroph adenoma
C. Somatotroph adenoma
D. Gonadotroph adenoma

C 32 A 23 year old female complained of sore throat, fever and malaise. PE P 1135 0.33
revealed tender, enlarged thyroid gland. Lab exam showed increase T4 and
T3, decrease TSH. The most likely diagnosis is
A. Graves disease
B. Hashimoto thyroiditis
C. De Quervain thyroiditis
D. Reidel thyroiditis

B 33 The most characteristic microscopic finding of Retinoblastoma is: R 1373 0.33


A. Homer Wright rosettes
B. Flexner Wintersteiner rosette
C. Blastema cells
D. Rosenthal fibers

A 34 Microscopic features of this lesion in the urinary bladder revealed P 1002 0.5
infiltration with large foamy macrophages with multinucleated giant cells
interspersed with lymphocytes. Michaelis Gutmann bodies are present
A. Malakoplakia
B. Tuberculosis
C. Cystitis glandularis
D. Acute suppurative cystitis

B 35 Which of the following is TRUE of Pleomorphic adenoma I 770 0.5


A. These are also called Adenolymphomas
B. There are the most common tumors of the salivary gland
C. These are highly malignant tumors
D. Occurs much more often in males

C 36 This is a highly vascular tumor of the nasopharyngeal area that occur P 877 0.5
almost exclusively in adolescent males. It has a benign nature but has
tendency to bleed profusely during surgery.
A. Neuroblastoma
B. Squamous cell carcinoma
C. Angiofibroma
D. Inverted papilloma

A 37 Step/s in phagocytosis enhanced by opsonins: I 62 0.33


A. Recognition & attachment
B. Engulfment
C. Killing or degradation
D. A & B only

D 38 Abnormal acquired leukocyte chemotaxis is/are seen in: I 65 0.33


A. Anemia
B. Sepsis
C. Diabetes mellitus
D. B & C only

D 39 Distinctive pattern of inflammation seen in tuberculosis R 83 1


A. Serous
B. Suppurative

PATHO BOARD EXAM QUESTIONS 13


C. Fibrinous
D. Chronic granulomatous

B 40 The biologic behaviour of malignant melanoma is determined by: P 1179 0.5


A. Radial growth phase
B. Nature & extent of vertical growth phase
C. Degree of pigmentation
D. All

A 41 The most commonly accepted cause of squamous cell carcinoma of the I 1185 0.5
skin is:
A. Exposure to ultraviolet light
B. Exposure to arsenic
C. Cutaneous scars
D. PUVA

B 42 The detection of this antibody is specific for systemic sclerosis : R 227 0.5
A. Anti-centromere antibody
B. Anti-Scl 70
C. Anti-DsDNA
D. Anti-Sm

C 43 The receptor used by the HIV strain that is effective in transmitting the I 240 0.25
infection :
A. CCR4
B. CXCR4
C. CCR5
D. CXCR5

B 44 Blood transfusion reaction is an example of which mechanism of R 199-200 0.5


hypersensitivity reaction ?
A. Type IV - delayed hypersensitivity
B. Type II - complement dependent
C. Type II - Ab-mediated cellular dysfunction
D. Type III - immune complex mediated

C 45 True of clonal anergy : I 213-214 0.25


A. Fas-FasL engagement leading to apoptosis of activated T-cells
B. clonal deletion of self-reactive T-cells during maturation
C. Ag is presented by cells not bearing the proper ligand resulting to a
negative signal
D. suppressor T-cells down regulating the other T-cells

C 46 A 53 y/o G10P10(10-0-0-10) female underwent transplantation of the right P 207-208 0.33


kidney. The donor kidney was taken from her husband. Fifteen minutes
after transplantation, the transplanted kidney become cyanotic, mottled
and flaccid. What is the mechanism of rejection?
A. direct cell mediated reaction
B. indirect cell mediated reaction
C. Ab-mediated reaction
D. there is no graft rejection

D 47 The true neoplastic element in Hodgkin disease : R 670 1


A. eosinophil
B. smudge cells
C. plasma cells
D. Reed-sternberg cells

C 48 Left enlarged axillary lymph node from a 45 y/o female with left breast I 649-650 0.33
carcinoma would likely show :
A. paracortical hyperplasia
B. follicular hyperplasia
C. sinus histiocytosis
D. histologically normal

PATHO BOARD EXAM QUESTIONS 14


A 49 Incision biopsy of a submandibular mass of a 6 y/o African boy revealed P 662 0.5
diffuse infiltrates of intermediate-sized lymphoid cells with "starry sky"
pattern. Tumor cells express CD19, CD10 and surface IgM. Give the
probable diagnosis :
A. Burkitt's lymphoma
B. small lymphocytic lymphoma
C. acute lymphoblastic lymphoma
D. follicular lymphoma

B 50 Hypochromic Microcytic RBC is seen in : R H-619 0.5


A. Vitamin B12 deficiency
B. Iron deficiency anemia
C. Sickle cell disease
D. Hereditary spherocytosis

D 51 Sucrose hemolysis test result in a patient with paroxysmal nocturnal I H-635 0.5
hemoglobinuria :
A. no hemolysis
B. <5% hemolysis
C. 5-10% hemolysis
D. >10% hemolysis

A 52 A stillborn infant with severe edema, marked anemia and hepatospleno- P H-647 0.25
megaly shows large quantities of Hb Bart's. True of this condition :
A. complete absence of alpha chains
B. complete absence of beta chains
C. lagre quantities of Hb A
D. small quantities of Hb F

B 53 Angioblasts are involved in: R 103 0.5


A. angiogenesis
B. vasculogenesis
C. granulation tissue formation
D. tissue remodelling

C 54 Vitamin C deficiency retards wound healing because: I 110 0.5


A. infection persists
B. wound contraction is inhibited
C. collagen synthesis is inhibited
D. exuberant granulation occurs

A 55 Increased apoptotic activity will: I 90 0.33


A. decrease the cell population
B. increase cell differentiation
C. decrease cell proliferation
D. decrease cell death

D 56 Grossly, a breast tumor is soft with an appearance of pale gray-blue gelatin. P 1112 0.5
Microscopically, large lobes of lightly staining mucin with floating neoplastic
cells are seen. What is your diagnosis?
A. Schirous carcinoma
B. Tubular carcinoma
C. Medullary carcinoma
D. Colloid carcinoma

B 57 Which is a common benign lesion in male breast? R 1117 1


A. fibroadenoma
B. gynecomastia
C. fibrocystic change
D. tubular adenoma

B 58 Which is TRUE in a patient diagnosed with invasive lobular carcinoma? I 1111 0.33
A. the tumor is always unilateral

PATHO BOARD EXAM QUESTIONS 15


B. tumor frequently metastasize to the CSF
C. tumor cells are confined to the lobule
D. prominent lymphoplasmacytic infiltration

C 59 What is the most common complaints of lung cancer patients? R 745 0.5
A. weight loss
B. chest pain
C. cough
D. dyspnea

B 60 A female patient with no previous smoking history is diagnosed to have P 745 0.5
bronchogenic carcinoma, what is the most probable histologic type?
A. oat cell carcinoma
B. adenocarcinoma
C. squamous cell carcinoma
D. large cell caricnoma

C 61 A patient presented with enophthalmos, ptosis, & miosis. He was previously P 747 0.33
diagnosed with lung cancer. Where is the most probable site of the tumor?
A. lower lobe
B. middle lobe
C. apex
D. whole lung

C 62 A malignant lung tumor is 3 cm in size without pleural involvement & has ipsi- I 745 0.33
lateral hilar node involvement, what is the stage of the tumor?
A. Ia
B. Ib
C. IIa
D. IIb

D 63 Patchy consolidation of the lungs & the presence of suppurative exudate I 721 1
filling up the bronchi, bronchioles, & adjacent alveolar spaces are characte-
ristics seen in:
A. tuberculosis
B. lobar pneumonia
C. primary atypical pneumonia
D. bronchopneumonia

The associated molecular change in the evolution of colorectal tumor from


B 64 late R 296 0.5
adenoma to carcinoma is loss of
A. APC gene chromosome 5q
B. p53 chromosome 17p
C. tumor suppressor gene chromosome 18q
D. ras gene chromosome 12p

B 65 Migration of tumor cells in invasion of extracellular matrix is facilitated by P 302 0.33


A. metalloproteinases
B. thymosin B15
c. cadherin
D. beta catenin

A 66 High risk HPV involve in cervical carcinoma R 311 0.33


A. HPV 16
B. HPV 6
C. HPV 11
D. HPV 4

D 67 Tumor suppressor gene involved in breast carcinoma I 286 0.5


A. p53
B. WT1
C APC
D. BRCA1

PATHO BOARD EXAM QUESTIONS 16


C 68 Tumor composed of more than one neoplastic cells type from more than one P 261 0.3
germ layer
A. pleomorphic adenoma
B. fibroadenoma
C. Mature cystic teratoma
D. leiomyoma

A 69 Enzyme deficiency in Tay Sacch's disease R 155 0.33


A. Hexosaminidase A
B. Sphingomyelinase
C. Glucocerebrosidase
D. alpha 1-iduronidase

D 70 Characteristic clinical manifestation of Von Gierke's disease P 160 0.33


A. painful cramps
B. massive cardiomegaly
C. café au lait spots
D. hypoglycemia

C 71 Most threatening feature of Marfan's syndrome R 149 0.5


A. skeletal abnormality
B. ocular findings
C. cardiovascular lesion
D. hyperextensible skin

C 72 Class III mutation in Familial hypercholesterolemia is a defect in I 152 0.25


A. transport
B. recycling
C. binding
D. synthesis

1232-
A 73 Which of the following is NOT associated with Pyogenic Osteomyelitis? R 1233 1
A. Saber shin
B. Sequestrum
C. Involucrum
D. Subperiosteal abscess

1253-
D 74 Which of the following is NOT TRUE of Gout and Pseudogout? I 1257 0.5
A.Both may arise from enzymatic defects
B.Both affect the elderly age group usually
C.Both may arise secondarily from other disease conditions
D.Both arise from the same genetic aberration or defect

B 75 What is the most characteristic cell found in Rhabdomyoma? R 591 0.5


A.Epithelioid Histiocyte
B.Spider Cell
C.Mallory Cell
D.Langhan's Giant Cell

1281-
D 76 Associated with little or no production of the 427kD protein called Dystrophin: I 1283 0.33
A. Duchenne Muscular Dystrophy
B. Myotonic Dystrophy
C. Becker Muscular Dystrophy
D. A and C only

A 77 A white infarct, in contrast to red infarct, is usually seen in organs which are: R 132 0.5
A. solid
B. with double blood supply
C. infected
D. previously congested.

B 78 Patients with left-sided heart failure may develop pulmonary edema because I 114-116 0.5

PATHO BOARD EXAM QUESTIONS 17


of which of the following mechanisms?
A. increase oncotic pressure
B. increase osmotic pressure
C. vascular permeability
D. sodium retention.

C 79 A 24 years old male drug abuser developed Subacute bacterial endocarditis I 130 0.33
involving the mitral valve.The patient later developed a septic pulmonary
infarc-.
through a process known as:
A. systemic thromboembolisation
B. pulmonary thromboembolisation
C. paradoxical thromboembolisation
D. bacterial dissemination.

A 26 years old primigravida developed severe sudden dyspnea, cyanosis,


D 80 and P 131-132 0.5
hypotension one hour after a normal sponataneous vaginal delivery to a live,
full term baby girl. She manifest seizures and went into coma on the 2nd day
post-partum folowed by petechial hemorrhages and hematoma over the trunk
and extremities. What would be the most probable diagnosis?
A. Disseminated intravascular coagulation
B. Thrombotic thrombocytopenic purpura
C. Global hypoxemic encephalopathy
D. Amniotic fluid embolism.

B 81 A 60 year old male, chronic hypertensive, 40 pack year cigarette smoker I 526-528 0.5
sought admission because of severe anterior chest pain which later localized
to the left lower chest not relieved by nitrites or rest.Pertinent physical
examination finding showed: BP = 130/80 mm Hg; HR = 86/min; RR = 24/min
Heart: regular rate and rhythym; no murmur; bouncing pulse. The most
probalbe diagnosis is:
A. Myocardial infarction
B. Aortic Dissection
C. Unstable angina
D. Peptic ulcer disease.

B 82 A 35 year old female was admitted because of blurring of vision, numbness of P 519-520 0.33
the fingers, and dizziness. Pertinent P.E. findings: BP, upper extermity =
30/0 mm Hg; BP, lower extremity = 160/100 mm Hg; thready to absent
pulse, upper extremity; bouncing pulse, lower extremtiy. The most probable
histologic finding of vascular lesions in this patient is:
A. nonspecific transmural inflammation of arterial wall
B. collagenous fibrosis involving all layers of vessel wall
C. granulomatous vasculitis and necrotizing granulomas
D. fibrous encasement of artery, vein, and nerve.

D 83 A 57 year old seaman was admitted because of dysphagia, sternal pain, P 526 0.5
dyspnea, and chronic cough of 6 months duration.He was treated for a penile
chancre followed by fever, rashes, and lymphadenpathy 30 years ago.
Most patients with this lesion die of:
A. cardiogenic shock
B. hypovolemic shock
C. sepsis
D. congestive heart failure

C 84 The most characteristic histologic lesion in the glomeruli of patients with R 951-952 0.5
Rapidly progressive glomerulonephritis is:
A. focal hyalinosis
B. basement membrane thickening
C. crescent formation
D. hypercellularity.

B 85 An 18 year old female sought admission because of anasarca and darkly P 952;965 0.33
colored urine. Pertinent Urine findings: protein - +++; oval fat bodies. Blood
PATHO BOARD EXAM QUESTIONS 18
chemistry: albumin - 20 g/L; Cholestrol - 340 mm%; Kidney biopsy: uniform
diffuse capillary wall thickening on light microscope; diffuse granular
precipitate
of IgG and C3 on IF microscopy. The most probable diagnosis is:
A. Poststreptococcal glomerulonephritis
B. Membranous glomerulonephritis
C. Lipoid nephrosis
D. IgG nephropathy.

C 86 A 28 year old primigravida had hypotensive shock because of postpartum P 969-971 0.5
hemorrhage. Her urine output ranged from 80 - 200 ml/24 hours. The
basic kidney lesion to explain this decreased urine output among these
patients is:
A. Hemolytic-uremic syndrome
B. Obstructive uropathy
C. Acute tubular necrosis
D. Acute tubulointerstitial nephritis.

D 87 A 40 year old male was admitted because of loss of weight, hematuria, right I 991-993 0.5
costovertebral pain of 5 months duration. Hb = 220/L. Kidney ultrasound
revealed a 5 X 5 cm mass over the upper pole of right kidney which on biopsy
showed kidney parenchyma replaced by clear cells in acinar pattern. The
most probable diagnosis is:
A. Retention cyst
B. Renal papillary adenoma
C Wilm's tumor
D. Renal cell carcinoma

A 88 Histologic findings of markedly enlarged hyperchromatic nuclei, arranged in I 1013 0.33


disorderly fashion with increasd mitosis, and confined within the basement
membrane of the penile skin is consistent with:
A. Bowen's disease
B. Erythroplasia of Queyrat
C. Bowenoid papulosis
D. Squamous cell carcinoma.

An 8 year old male child was brought to the hospital for testicular
B 89 enlargement. P 1024 0.33
Pertinent physical examination findings revealed gynecomastia, with pubic
and axillary hair and husky voice. The most probable testicular lesion is:
A. Seminoma
B. Leydig cell tumor
C. Sertoli cell tumor
D. Embryonal cell carcinoma

Found in the hippocampus and purkinje cells of the cerebellum of patients


D 90 with R 1331 0.33
Alziemer's Disease
A. Cowdry bodies
B. Negri bodies
C. Lewy bodies
D. Hirano bodies

C 91 Characteristic of CNS neoplasia I 1343 0.5


A. metastasize early
B. majority arise form neurons
C. site is more important than grade
D. metastatic cancers are uncommon

D 92 Hydrocephalus may result from P 1298-9 1


A. tuberculous meningitis
B. decreased brain parenchyma
C. space occupying lesion in the brain
D. all of these

PATHO BOARD EXAM QUESTIONS 19


C 93 Characteristic of cirrhosis EXCEPT I 853 0.5
A. bridging fibrous septa
B. parenchymal nodules
C. hypertrophy of bile duct epithelium
D. disruption of architecture

B 94 Acute Viral Hepatitis EXCEPT I 866/869 0.5


A. ballooning degeneration
B. mallory bodies present
C. drop-out necrosis
D. cholestasis

D 95 Clinical manifestation of cholestasis EXCEPT P 851 0.5


A. pruritis
B. jaundice
C. xanthomas
D. angiomas

C 96 Liver cancer seen in young adults without predisposing factors P 890-1 0.33
A. cholangiocarcinoma
B. angiosarcoma
C. fibrolamellar HCC
D. hepatic adenoma

D 97 Adaptation NOT commonly seen in the breast I 36 0.33


A. hypertrophy.
B. hyperplasia
C. atrophy
D. metaplasia

C 98 Mechanism of triglyceride accumulation in malnutrition I 39 0.5


A. excessive production
B. exogenous accumulation
C. defective transport
D. decreased enzyme

C 99 Protein accumulation in cell R 41 1


A. brown atrophy
B. Psammoma bodies
C. Russell bodies
D. steatosis

C 100 Most common initial outcome of acute inflammation in Peptic ulcer: P 79 0.5
A. Abscess formation
B. Healing through fibrosis
C. Progression to chronic inflammation
D. Resolution

TOTAL MPL 49.51

REFERENCE
Robbins Pathologic Basis of Disease by Cotran, Kumar, Collins
sixth edition
H = Clinical Diagnosis and Management by Laboratory Methods by Henry
nineteenth edition

BOARD REVIEW

CHOOSE THE BEST ANSWER.

1. Viruses induce cell injury through:


A. direct cytopathic effect C. both A & B are correct
PATHO BOARD EXAM QUESTIONS 20
B. immune – mediated reactions D. neither A nor B is correct

2. Process (es) involved in cell necrosis is (are):


A. denaturation of proteins C. both A & B are correct
B. enzymic digestion of cells D. neither A nor B is correct

3. The following statements are TRUE of caseous necrosis:


A. it is a combination of coagulation and liquefaction necrosis
B. it is a characteristic feature of tuberculosis
C. both A & B are correct
D. neither A nor B is correct

4. Jaundice is due to the accumulation of this pigment:


A. bilirubin C. lipofuscin
B. hemosiderin D. melanin

5. The sequence of cellular events in inflammation is:


A. margination  diapedesis  chemotaxis  phagocytosis
B. diapedesis  margination  chemotaxis  phagocytosis
C. chemotaxis  margination  diapedesis  phagocytosis
D. chemotaxis  diapedesis  margination  phagocytosis

6. The most effective means of microbial killing is via the:


A. oxygen – independent mechanism
B. myeloperozidase independed mechanism
C. hydrogen peroxide, myeloperoxidase and halide system
D. only A & C are correct

7. The most important cause of delay in wound healing is:


A. infection C. inadequate blood supply
B. presence of foreign bodies D. mechanical factors

8. Keloid is due to excessive:


A. granulation tissue C. amyloid deposition
B. granuloma formation D. collagen accumulation

9. Edema can result from any of the following mechanisms:


A. increased hydrostatic pressure of the blood
B. increased osmotic pressure of interstitial fluid (sodium retention)
C. decreased oncotic pressure of plasma protein
D. all of the above are correct

10. A patient with CHF has generalized edema. The operative mechanism involved in edema are
the following EXCEPT:
A. increased aldosterone secretion
B. increased central venous pressure
C. diminished CO
D. diminished ADH secretion

11. Periorbital edema is usually encountered in:


A. filariasis C. liver cirrhosis
B. nephritic syndrome D. inflammatory states

12. Fluid that collects during acute inflammation and that has a protein content in excess of 3.0 g/L
and SG over 1.015 is termed:
A. exudate C. hydropericardium
B. transudate D. wheal

13. A segment of one chromosome transferred to another segment is known as:


A. mutation C. inversion
PATHO BOARD EXAM QUESTIONS 21
B. translocation D. deletion

14. The most common cause of the occurrence of Klinefelter’s syndrome:


A. non-disjunction in meiotic division in oogenesis
B. non-disjunction in meiotic division in spermatogenesis
C. non-disjunction in meiotic division in embryogenesis
D. translocation

15. The number of Barr bodies in a patient with 48 XXXY karyotype:


A. none C. two
B. one D. three

16. The following clinical manifestations are seen in a patient with Down’s syndrome EXCEPT:
A. dysplastic pelvis and middle phalanx of hand
B. mental retardation
C. microcephaly
D. horizontal palmar crease

17. Pathologic changes that have been described in AIDS include cell of the following EXCEPT:
A. hypogammaglobulinemia
B. viral inclusions in histiocytes of lymph nodes
C. T4/T8 ratio is very low
D. Lymphoid depletion of cortical and paracortical areas of lymph nodes

18. Thymectomy in new born infant will cause:


A. impaired cell-mediated immune response
B. impaired antibody response
C. heavy chain disease
D. no discenible immune dysfunction

19. Tuberculous lesions are brought about by:


A. release of exotoxin C. hypersensitivity reaction
B. release of endotoxin D. activation of lysosomes

20. Immunoglobulins are produced by:


A. neutrophils C. eosinophils
B. basophils D. lymphocytes

21. The following are malignant conditions EXCEPT:


A. hepatoma C. melanoma
B. lymphoma D. adenoma

22. This tumor belong to the category of blastoma:


A. dermoid cyst C. squamous cell carcinoma
B. adenocarcinoma D. Wilm’s tumor

23. The following are non-neoplasia EXCEPT:


A. hamartoma C. dysplasia
B. choristoma D. fibroma

24. The following are suppressor genes EXCEPT:


A. p53 C. APC
B. bcl-2 D. NF – I

25. A 6-year-old boy develops abdominal pain and vomiting preceded by a 3-day period without
bowel movements. A diagnosis of intestinal obstruction is made. Of the following, the most
likely reason for the obstruction is a bolus of:
A. Taenia saginata C. Strongyloides tercoralis
B. Ascaris lumbricoides D. Onchocerca volvulus
PATHO BOARD EXAM QUESTIONS 22
26. The following is/are attributed directly or indirectly to schistosomiasis EXCEPT:
A. solitary liver abscess
B. granulomatous around shcistosoma ova
C. dark (gray) color of the liver and spleen because of a hemoglobin-derived pigment that
accumulate in reticulo-endothelial cells
D. urinary bladder malignancy, predominantly of the squamous cell type

27. The morphologic changes in typhoid fever include the following EXCEPT:
A. accumulation of mononuclear phagocytes throughout the lymphreticular system, with some
lymphocytes and plasma cells
B. enlargement of the liver with areas of focal necrosis surrounded by mononuclear cell
proliferation (typhoid nodules)
C. oval ulcers in the ileum with their long axis along the long axis of the bowel

28. A chronic carrier state of typhoid fever is most likely due to the persistence of the organism in:
A. the appendix C. the gall bladder
B. the cortex of the kidney D. the ileum

29. Function of thiamine:


A. co-enzyme in oxidative decarboxylation of alpha-ketoacids
B. synthesis of DNA and RNA
C. respiratory enzyme in the cytochrome enzyme
D. all of the above are correct

30. The metabolism of calcium is closely regulated by:


A. vitamin D C. thyroid gland
B. parathyroid gland D. all of the above are correct

31. The mechanism of clotting is affected by:


A. Vitamin A C. Thiamine
B. Vitamin K D. Vitamin E

32. This/these pollutants is/are dangerous because once released into the environment they are
“forever” i.e. resistant to natural process of decay.
A. polychlorinated biphenyls C. carbon monoxide
B. chloroform D. all of the above are correct

33. Ethyl alcohol toxicity will cause:


A. venous thrombosis C. fatty liver
B. cheery-red discoloration of skin D. gingivitis

34. Which of the following is the most common adverse effect of smoking?
A. myocardial infarction C. cancer of the pancreas
B. cancer of the bladder D. peptic ulcer disease

35. Exposure to carbon particle in the ambient air will result to:
A. Anthracosis C. progressive massive fibrosis
B. coal worker’s pneumoconiosis D. any of the above

36. The leading cause of death in children between 5-14 years of age is:
A. malignant neoplasm C. injuries resulting from accidents
B. congenital anomalies D. cardiac diseases

37. The highest at risk period for intrauterine cytomegalovirus infection:


A. 1st trimester of pregnancy C. 3rd trimester of pregnancy
nd
B. 2 trimester of pregnancy D. all of the above

38. Organogenesis is completed by:


A. 2nd trimester of pregnancy C. the end of 1st trimester
PATHO BOARD EXAM QUESTIONS 23
B. the end of 2nd trimester D. the middle of 1st trimester

39. Polyarteritis nodosa frequently affected organs include all of the following EXCEPT:
A. Kidney C. Lungs
B. Heart D. GIT

40. All of the following are characteristic features of thromboangitis obliterans (Buerger’s disease)
EXCEPT:
A. lesions are sharply segmental
B. only the lower extremities are affected
C. arteries of small and medium size are affected
D. both upper and lower extremities are affected

41. All of the listed anatomic changes are considered as important features in syphilitic aortitis
EXCEPT:
A. longitudinal wrinkling (tree barking)
B. dilatation of aortic valve ring
C. obliterative endarteritis of vasa vasorum
D. widening of coronary ostia

42. The histological hallmarks of temporal or giant cell arteritis are the following EXCEPT:
A. lymphocytic infiltrate in the intima and inner media
B. disruption of incarnal elastic lamira associated with giant cells
C. patchy necrosis of the media
D. eosinophilic infiltration of all muscular coats

43. Complicated or advanced atherosclerosis would most likely affect this portion of the blood
vessel:
A. intima only C. intima and adventitia
B. intima and media D. adventitia

44. A biopsy of the femoral artery revealed calcification in the media would be compatible with:
A. accelerated hypertension
B. young individuals, usually male
C. usual complication of ischemia and gangrene
D. none of the above

45. A vascular tumor associated with AIDS is:


A. Lymphaniosarcoma C. Angiosarcoma
B. Kaposi’s sarcoma D. Hemangiopericytoma

46. The following are associated with cor pulmonale EXCEPT:


A. concentric left ventricular hypertrophy
B. congestive heart failure
C. pulmonary valve dilatation
D. pulmonary hypertension

47. The following are possible morphologic changes in sickle cell disease EXCEPT:
A. Gamma Gandy bodies in the spleen
B. Papillary necrosis in the kidney due to capillary stasis
C. Micro infarction in the brain
D. Evidences of chronic glumerulonephritis

48. A decrease in the number of platelets is seen in which condition EXCEPT:


A. polycythemia vera C. idiopathic thrombocytopenic purpura
B. aplastic anemia D. acute leukemia

49. B-thalassemia major shows all of the following features EXCEPT:


PATHO BOARD EXAM QUESTIONS 24
A. hypochromic microcytic anemia
B. target cells and anisocytosis in peripheral blood smear
C. diminished synthesis of the alpha chain hemoglobin
D. clinical necessity for multiple blood transfusions

50. The patterns of chronic lymphadenitis include the following EXCEPT:


A. sinus histiocytosis C. follicular hyperplasia
B. paracortical hyperplasia D. suppurative lymphadenitis

51. “Starry sky” pattern of lymph node is characteristic of:


A. Burkitt’s lymphoma C. Histiocytic lymphoma
B. Hodgkins lymphoma D. Sezary’s syndrome

52. Lacunar cells are characteristics of this type of Hodgkin’s disease:


A. lymphocytic predominance C. mixed cellularity
B. lymphocyte depletion D. nodular sclerosis

53. The most common type of Histiocytosis X in infants is:


A. eosinophilic granuloma C. letterer-siwe syndrome
B. hand-schuller-christian disease D. unifocal langerhans cell histiocytosis

54. Sinus histiocytosis are normally seen in nodes draining:


A. infection C. drug abuse
B. cancers D. immunologic disorders

55. A patient presents with hemoptysis and acute renal failure. A diagnosis worth considering in
this clinical picture is:
A. Asbestosis C. Good pasture’s syndrome
B. primary atypical pneumonia D. tuberculosis

56. Alpha-1 antitrypsin deficiency is associated with:


A. panlobular emphysema C. Wegener’s granulomatosis
B. anthracosis D. bronchogenic carcinoma

57. Enlargement of pulmonary alveolar spaces with destruction of septal walls is seen in:
A. chronic bronchitis C. pulmonary infarction
B. emphysema D. alveolar proteinosis

58. A chest x-ray that shows a shaggy cavity with a thick irregular border and satellite densities in
the right lower lobe is most compatible with:
A. bronchogenic carcinoma C. abscess
B. tuberculosis D. histoplasmosis

59. Which statement about esophageal cancer is false?


A. it is usually of squamous cell type
B. most common in the distal portion
C. primarily a disease of elderly males
D. characterized by dysphagia and painless weight loss

60. Macrophages with positive periodic acid-schiff-staining material in intestinal lamina propia and
lymph nodes are characteristically found in:
A. Wilson’s disease C. Whipple’s disease
B. Elevated serum gastrin levels D. Gardener’s syndrome

61. The Mallory-Weiss syndrome is due to laceration in the mucosa of the:


A. Esophagus C. Pylorus
B. Stomach D. Small intestine
PATHO BOARD EXAM QUESTIONS 25
62. Carcinoma of the GIT would be expected to occur with greatest frequency in patients with:
A. familial polyposis of the colon C. Crohn’s disease
B. villous adenoma of the colon D. gastric ulcer

63. Chronic obstruction of the cystic duct results to development of:


A. Choledochal cyst C. Hydrops of the gall bladder
B. Porcelain gall bladder D. Carcinoma of the gall bladder

64. Which of the following condition(s) increase(s) the risk of bile duct carcinoma?
A. Choledochal cyst C. Hydrops of the gall bladder
B. Porcelain gall bladder D. Choledocholithiasis

65. Carcinoma of the pancreas:


A. occurs most often in the body of the pancreas
B. is associated with hypocalcemia
C. arises from the pancreatic ductal cells
D. is usually cured by total pancreatectomy
66. All of the following pediatric conditions are characterized by unconjugated hyperbilirubinemia
EXCEPT:
A. Hemolytic disease of the newborn
B. Physiologic jaundice of the newborn
C. Intrahepatic biliary atresia
D. Gilbert’s syndrome

67. The least common type of gall bladder calculi are:


A. pure calcium bilirubinate C. pure cholesterol
B. pure calcium carbonate D. mixed stone

68. Cholelithiasis is mainly due to:


A. Infection C. bile stasis
B. supersaturation of bile with cholesterol D. inflammation

69. In addition to nephritic injury, the other main cause of acute tubular necrosis is:
A. Sepsis C. severe hypocomplementemia
B. Ischemia D. immune complex deposition

70. Of the complications of acute pyelonephritis enumerated below, one does not belong:
A. retroperitoneal abscess formation C. acute episodes of paroxysmal HPN
B. renal papillary necrosis D. pyonephrosis

71. Of the risk factors for pyelonephritis enumerated below, one does not belong:
A. diabetes mellitus
B. male sex
C. congenital abnormalities of the urinary tract
D. pregnancy

72. Chronic renal disease, pheochromocytoma, Conn’s syndrome, coarctation of the aorta and
acromegaly are all conditions that may lead to:
A. venous thrombosis C. Wegener’s granulomatosis
B. hypersensitivity vasculitis D. hypertension

73. Nests of urothelium may be found in the lamina propia of the urinary bladder representing
normal variation in the morphology of the bladder. These nests of urothelium are called:
A. Brunner’s nests C. Brunn’s nests
B. Burney’s nests D. Burner’s nests

74. The following are implicated in the causation of bladder carcinoma, EXCEPT:
PATHO BOARD EXAM QUESTIONS 26
A. Beta naphthylamine C. Cyclophosphamide
B. S. Haemotobium D. There is no exception

75. A newborn baby was noted to pass out urine through a small opening in the area of the
umbilicus. This is most probably due to:
A. Urachal cyst C. Exstrophy of the bladder
B. Patent urachus D. Any of the above

76. Transitional cell tumors that are more likely to behave as malignant are characterized by the
following EXCEPT:
A. Induce angiogenesis
B. Manifest highly abnormal karyotypes
C. Lack blood group antigens on their cell surface
D. There is no exception

77. The following is caused by human papilloma virus type 6 (HPV-6):


A. Giant condyloma C. condyloma lata
B. Condyloma acuminatum D. all of the above

78. The following are TRUE about hypospadias and epispadias, EXCEPT:
A. Commonly associated with failure of normal descent of the testes and malformation of the
bladder
B. May lead to partial urinary obstruction
C. Possible cause of sterility
D. There is no exception

79. The majority of ovarian neoplasms arise from:


A. germ cells C. sex-cord stroma
B. surface epithelial cell D. metastatic from GIT

80. The most common site of endometriosis:


A. fallopian tube C. recto-vaginal septum
B. ovary D. vagina

81. Choriocarcinoma is most often preceded by:


A. abortion C. normal pregnancy
B. ectopic pregnancy D. H. mole

82. The most common primary malignant tumor of the ovary:


A. endometrioid carcinoma C. serous cystadenocarcinoma
B. mucinous cystadenocarcinoma D. yolk sac tumor

83. Cord like ducts filled with necrotic and cheesy tumorous tissue that can be readily extruded
upon pressure is seen in:
A. mucinous carcinoma C. medullary carcinoma
B. comedocarcinoma D. paget’s disease of the breast

84. Lesions mistaken clinically as malignant are the following EXCEPT:


A. fibroadenoma C. mammary duct ectasia
B. fat necrosis D. mastitis

85. Sites of metastases of cystosarcoma phyllodes are usually:


A. axillary lymph nodes C. liver
B. lungs and bone D. brain

86. The most frequent site of breast cancer is:


A. lower inner quadrant C. upper outer quadrant
B. upper inner quadrant D. subareolar
PATHO BOARD EXAM QUESTIONS 27
For numbers 87 to 89
87. A female patient presented with “moon facies,” truncal obesity and hisrutism. Plasma cortisol
was elevated. These features characterize:
A. Conn’s syndrome C. Waterhouse-Friderichsen syndrome
B. Cushing’s syndrome D. Only B & C are correct

88. If you suspect the presence of Cushing’s syndrome, you should perform a:
A. low dose dexamethasone test
B. high dose dexamethasone suppression test
C. both A & B are correct
D. neither A nor B is correct

89. If this were Cushing’s syndrome, one would expect:


A. suppression of cortisol secretion C. no suppression of cortisol secretion
B. suppression of ACTH secretion D. only A & B are correct

90. Suppression of cortisol secretion with high dose dexamethasone suppression test indicates the
presence of:
A. an adrenal neoplasm C. pituitary ACTH excess
B. ectopic ACTH syndrome D. all of the above are correct

91. Which of the following is a secondary lesion?


A. wheal C. erosion
B. pustule D. bulla

92. Psoriatic patients typically develop silvery:


A. excoriations C. papules
B. scales D. pustules

93. A patient presents with vesicles on the left side of his trunk. A simple lab procedure to do to
support a diagnosis of herpes zoster is:
A. gram stain C. Tzanck smear
B. KOH stain D. Patch test

94. One of the etiologic factors in acne is the:


A. Pityrosporum C. Propionibacterium
B. Staphylococcus D. Enterococci

95. The paucibacillary therapy for Hansen’s disease is:


A. Rifampicin 600 mg; Ofloxacin 400 mg; Minocycline 100mg daily for 6 months
B. Rifampicin 600mg once a month for 6 months; Dapsone 100mg daily for 6 months
C. Rifampicin 600mg once a month for 2 years; Dapsone 100mg daily for 2 years; Clofazimine
50mg daily for 2 years
D. Rifampicin 600mg once a month for 6 months; Dapsone 100mg daily for 6 months;
Clofazimine 50mg daily for 6 months

96. A child presents with an array of macules, papules, vesicles, and bullae, reddish, with pale
eroded center involving the extremities, lips and oral mucosa in a symmetric pattern. There was
intake of penicillin and sulfonamides for urinary tract infection. The patient most likely has:
A. erythema multiforme C. lichen planus
B. psoriasis D. porphyria

97. Most common primary malignant brain tumor:


A. Medulloblastoma C. Ganglioneuroma
B. glioblastoma multiforme D. ependymoma

98. Histologic criteria for glioblastoma multiforme:


A. vascular endothelial proliferation
PATHO BOARD EXAM QUESTIONS 28
B. anaplasia
C. both A & B are correct
D. neither A nor B is correct

99. Medulloblastoma most frequently occurs in:


A. age 60 and above C. children & young adults
B. middle adulthood D. no age predilection is observed

100. Tuberculoma in children frequently occurs in:


A. Supratentorial C. Posterior fossa
B. Infratentorial D. Transtentorial

1. The earliest cellular dysfunction following hypoxia is: R


A. apo ptosis
B. oxidate decom position
C. defective protein synthesis
D. release of hydrolytic enzymes

2. Movement of leukocytes towards the site of injury is: R


A. opsonization
B. chemotaxis
C. diapedesis
D. transmigration

3. The central factor in the pathogenesis of irrevsible cell injury is: R 9 0.33
A. mitochondrial dysfunction
B. membrane damage
C. cytoskeletal abnormalities
D. free radical formation

4. The type of necrosis where there is preservation of the basic R Robbins


7th
outline of the dead cells for a few days is noted in: edition
A. Caseation
B. Coagulation
C. Liquefaction
D. Gangrenous

5.The sequence of cellular events in inflammation is: R


A. margination--diapedesis--chemotaxis--phagocytosis
B. diapedesis--margination--chemotaxis--phagocytosis
C.chemotaxis--margination--deapedesis--phagocytosis
D. chemotaxis--deapedesis--margination--phagocytosis

6. The oxygen-derived free radicals are implicated in: R 76 0.5


A. endothelial cell damage
B. release of histamine
C. tissue regeneration
D. prostaglandin release

7. The immediate transient phase of vascular permeability in most U 51 1


types of tissue injury is mediated by:
A. histamine
B. prostaglandins
C. complement
D. bradykinin

8. The most important cause of delay in wound healing is: R


A. infection
B. presence of foreign bodies
C. inadequate blood supply
D. mechanical factors

9. Keloid is due to excessive R


PATHO BOARD EXAM QUESTIONS 29
A. granulation tissue
B. granuloma formation
C. amyloid deposition
D. collagen accumulation

10. A laceration in the right hand of a 22-year old male was sutured. A 0.9
A week later, the sutures were removed and healing continued.
However, the site of the wound developed a raised, nodular scar
that developed over the following 2 months. What process
occurred?
A. wound dehiscence
B. exuberant granulation
C. organization
D. keloid formation

11. A white infarct, in contrast to red infarct, is usually seen in organs R 132 0.5
which are:
A. solid
B. with double blood supply
C. infected
D. previously congested

12. The substance that is responsibel for massive peripheral R


vasodilatation and schock in sepsis is:
A. cytokines
B. oxygen-derived free radicals
C. fibrin degradation products
D. nitric oxide

13. Most important predisposing factor in the development of U


thrombosis is:
A. stasis
B. turbulence
C. endothelial injury
D. hypercoagulability of blood

14. A 40-year old male with blood type of B+ receives a blood U 0.7
transfusion. Unknowingly, blood from another patient with blood
type A+ was used for compatibility testing prior to the transfusion.
Within 30 minutes after starting the transfusion, the patient
becomes tachycardic and hypotensive and passes tea-colored-
urine. How is this reaction mediated?
A. mast cell degranulation
B. antigen-antibody complex deposition in glomeruli
C. complement-mediated lysis of red blood cells
D. antibody-dependent cellular cytotoxicity

15. Thymectomy in a new born infant will cause: U


A. Impaired cell-mediated immune response
B. Impaired antibody response
C. Heavy chain disease
D. No discernible immune dysfunction

16. A 48-year-old man has chronic cough with fever that have U 204-226 0.25
persited for several months. The chest radiograph reveals a diffuse
reticulonodular pattern. Microscopically on transbronchial biopsy
there are focal areas of inflammation containing epitheloid
macrophages, Langhan's giant cells, and lymphocytes. These
findings are most typical for which of the following immunologic
responses:
A. Type I
B. Type II
C. Type III
D. Type IV

PATHO BOARD EXAM QUESTIONS 30


17. A 53 y/o G10P10(10-0-0-10) female underwent transplantation A 207-208 0.33
of the right kidney. The donor kidney was taken from her
husband. Fifteen minutes after transplantation, the transplanted
kidney become cyanotic, mottled and flaccid. What is the
mechanism of rejection?
A. direct cell mediated reaction
B. indirect cell mediated reaction
C. Ab-mediated reaction
D. there is no graft rejection

18. A 39-year-old man had an acute febrile illness with fever, A 246-248 0.25
pharyngitis, lymphadenopathy, and malaise following a trip to a
city known for its commercial sex workers. Assuming a typical
course of the disease, in how many years would you predict that
his peripheral blood CD4 lymphocyte count will drop below
200/microliter:
A. one
B. five
C. ten
D. fifteen

19. A male infant was born at term. No congenital anomalies were A 232-233 0.25
noted at birth. A year later he now has failure to thrive and has
been getting one bacterial pneumonia after another with both
Homophilus influenzae and Sreptococcus pneumoniae cultured
from his sputum. You should most strongly supect that he has:
A. Di George Syndrome
B. Selective IgA deficiency
C. EBV infection
D. X-inked agammaglobulinemia

20. This is NOT a clinical indication of karyotyping R


A. recurring pregnancy loss
B. single malformation in the newborn
C. mental retardation of unkown cause
D. offspring with chromosomal rearrangements

21. How many Barr bodies does a patient with 48 XXXY karyotype U
have?
A. none
B. one
C. two
D. three

22. A 24-year old male has a workup for infertility and is found to A 0.5
have oligospermia. Physical examination finding include bilateral
gynecomastia, reduced testicular size, and reduced body hair.
Karyotypic analysis will most likely reveal which of the following
abnormalities?
A. 47,XXY
B. 47,XYY
C. 46,XY, del(22q11)
D. 46XX/47XX,+21

23. Which would be a preneoplastic lesion? R


A. Cirrhosis
B. Hepatoma
C. pancreatic chrorista
D. Lymphoma

24. A biopsy of the stomach reveals an area of malignant change U


characterized by vansized, compactly arranged neoplastic glands
lined by moderately differentiated cells. Which of the following
terms refer to malignant tumor or glandular apethelium?
A. Sarcoma
PATHO BOARD EXAM QUESTIONS 31
B. Choristoma
C. Teratoma
D. Adenocarcinoma

25. A 70-year old asymptomatic man is found to have bilateral A


inguinal hernias. At operation for herpia repair, an enlarged lymph
node is removed. Pathologic evaluation disclosed a metastatic
adenocarcinoma. The primary site of such a tumor can be best
determined by:
A. Barium enema
B. Exploratory laparotomy
C. Immunoperoxidase localization of specific tumor markers
D. Computed tomograpy of the pelvis

26. A 12 year old boy from Kalinga was admitted at a local hospital R Robbins 0.9
due to fever and chills. What is the infective stage of malaria to 7th ed.
man? p.401
A. gametocyte
B. trophozoite
C. sporozoite
D. schizont

27. An elderly woman was diagnosed with military tuberculosis. R Robbins 0.7
Which organ will Mycobacterium tuberculosis LEAST likely infect? 7th ed.
A. adrenal p.386
B. bone marrow
C. kidney
D. thyroid

28. Secondary pulmonary TB does NOT relate to U


A. Ghon complex
B. nutiary spread
C. caseation necrosis
D. apical lesions

29. A pigeon raise suddenly developed stiff neck and deteriorating A


sensorium. CSF was obtained and should be stained with:
A. Acid fast stain
B. methylene blue
C. India ink stain
D. gram stain

30. Exposure to radiation is most damaging to human tissue? R


A. Alpha
B. Beta
C. Gamma
D. X-ray

31. What is the term given to a group of disease that results from the R
chronic inhalation of particulate or gaseous agents as a result of
occupational exposure?
A. Granulomatous dissease
B. Pneumoconiosis
C. Mycobacteriosis
D. Bronchiectasis

32. Fatty change is found in chronic alcoholism due to: R


A. Increased fat delivery to the liver
B. Decreased oxidation of fatty acids
C. Decreased transport of lipoproteins from the liver
D. Decreased serum lipid

33. A raving parent drags her child to the doctor, complaining about A 419-422 0.7
the recent deterioration in the child's performance at school. The
doctor observes that the child appears somewhat pale. Peering
PATHO BOARD EXAM QUESTIONS 32
at the child's medical record, the doctor notes a change in the
patient's address. A brief interview reveals that the family had only
recently moved to a new neighborhood, an urban community that
first grew around an old battery factory famed for having been
enclosed down after a series of well-documented industrial
accidents.
Given this information, and assuming that the parent is not merely
hysterical, the doctor suspects that the deterioration in the child's
performance may be the result of exposure to the following
substance:
A. asbestos
B. cigarette smoke
C. lead
D. organophosphates

34. Hypervitamininosis D in adults carries a risk of: U


A. Papilledema
B. renal calculi
C. growth retardation
D. Xeropthalmia

35. A 2-year boy was examined and noted to have generalized A


edema, variedly pigmental hair and chubby appearance. The
diagnosis of kwashiorkor will be supported by:
A. serum albumin and liver biopsy
B. CBC and hair follicle examination
C. renal biopsy
D. urinalysi and culture of the urine

36. A blood specimen is drawn for lipoprotein phenotyping. The test A


results obtained are:
1. Triglycorides-235 mg/dl (NV 40-164 mg/d)
2. Total cholesterol- 190 mg/dl (NV less than 200 mg/dl)
3. Prebeta- lipoprotein fraction normal
4. Beta- lipoprotein fraction normal
5. Chylomicrons present
6. Serum appearnce milky
The best explanation for these results would be that the individual
exhibited characteristics of:
A. A normal individual
B. A non-fasting serum protein
C. Type II hyperlipoproteinemia
D. Type IV hyperlipoproteinemia

37. A child came in at the out-patient department because of A


generalized edema accompanied by easily pluckable hair,
listlessness and loss of appetite. The child is most likely
suffering from:
A. Anorexia
B. Marasmus
C. Kwashiorkor
D. Bulimia

38. The leading cause of death in children between 5-14 years of age R
is:
A. malignant neoplasm
B. congenital anomalies
C. injuries
D. cardiac disease

39. Hirshsprung's disease is usually caused by the congenital R


absence of ganglion cells in the myenteric plexus of which
segment of the large intestine?
A. Cecum
B. Ascending colon
PATHO BOARD EXAM QUESTIONS 33
C. Descending colon
D. Rectum

40. A 16-year old boy presents with a right-sided abdominal mass, A


which examination discloses to be associated with the liver.
X-rays shows partially calcified tumor accupying most ot the right
abdomen. The correct diagnosis is:
A. Wilms' tumor
B. Hepatoblastoma
C. Pancreatoblastoma
D. Neuroblastoma

41. The most important feared complication of atheroma is: R


A. calcification
B. rupture
C. superimposed thrombosis
D. ulceration

42. Emboli of major veins of the legs eventually plug small vessels U
of the:
A. pulmonary circulation
B. coronary circulation
C. cerebral circulation
D. systematic circulation

43. A 35 year old female was admitted because of blurring of vision, A 519-520 0.33
numbness of the fingers, and dizziness. Pertinent P.E. findings:
BP, upper extremity = 30/0 mm Hg; BP, lower extremity =
160/100 mm Hg; thready to absent pulse, upper extremity;
bouncing pulse, lower extremity. The most probable histologic
finding of vascular lesions in this patient is:
A. nonspecific transmural inflammation of arterial wall
B. collagenous fibrosis involving all layers of vessel wall
C. granulomatous vasculitis and necrotizing granulomas
D. fibrous encasement of artery, vein, and nerve

44. When a person dies suddenly from a "heart attack" the most R 562 1
likely event causing it will be a/an:
A. aortic aneurysm
B. coronary artery embolism
C. congestive heart failure
D. arrhythmia

45. Histopathology examination of the heart of a patient who died of U


AMI showed coagulative necrosis, edema, hemorrhage and
neutrophilic intilirates. How old is the infarct?
A. 30 minutes
B. 12 hours
C. 1 day
D. 1 week

46. A 25 year old female consulted a physician for gradual A


enlargement of the abdomen and both legs.
The condition started about 3 weeks ago as dyspnea with ankle
swelling, followed by abdominal heaviness. She later noticed
abdominal enlargement, which shifted when she lies in bed on her
side. Past history showed that she had RHD and was hospitalized
twice in the last 4 years but was sent home improved.

If you could visualize the heart, what change(s) would be


expected to give rise to the complication described?
A. fusion of the tricuspid commissures
B. thick and short mitral chord tendinae
C. verrocae on the valve leaflets
D. occluded cronary arteries
PATHO BOARD EXAM QUESTIONS 34
47. An 8-yr old boy has venticular septal defect. 40 yrs later, he A
developed increasing dyspnea, hypoxia, and cyanosis. The reason
for this is the development of
A. acute myo.infarction
B. left atrial thrombosis
C. pulmonary hypertension
D. endocardial fibroelastosis

48.The mechanism of clotting is affected by: R


A. Vitamin A
B. Vitamin K
C. Thiamine
D. Vitamin E

49. A 21-year old man has lifelong hemorrhagic diathesis. The PT A 639 0.75
and bleeding time are normal, but the aPTT is prolonged. The
most likely cause of the bleeding disorder is:
A. factor VIII deficiency
B. factor IX deficiency
C. factor VII deficiency
D. von Willebrand's disease

50. A 29 year old female underwent cardiac surgery for valve A 638 0.6
replacement. After a week, blood picture showed anemia with
numerous fragmented red cells, helmet cells and burr cells. The
most likely diagnosis is:
A. idiophatic thrombocytopenia
B. thalassemia
C. immunohemolytic anemia
D. microangiopathic hemolytic anemia

51. Which of the following is a T-cell lymhoid malignancy? R


A. Burkitt's lymphoma
B. Mycosis fungoides
C. Hairy cell leukemia
D. Lymphoplasmacytic lymphoma

52. The most important difference between a benign reactive lymp U 649 0.33
node and lymphoma is:
A. interstitial fibrosis
B. follicular hyperplasia
C. effacement of normal architecture
D. precence of plasma cells

53. Lito 27-year-old has a total WBC count of 4x10/L. The A


differential count is as follows:
Neutrophils- .28 (NV .56)
Lymphocytes -.65 (NV .34)
Band - .02 (NV .2)
Monocytes - .5 (NV .4)
Which of the following statement is true?
A. The percentage of lympocytes is normal
B. The absolute number of lymphocytes is low
C. there is an absolute lymphocytosis
D. There is a relative lymphocytosis

54. The histologic hallmark of chronic bronchitis is R


A. inflamation and fibrosis
B. hyperplasio of goblet cells
C.hypertrophy of submucosal glands
D. marked narrowing of bronchial lumen

55. Patchy consolidation of the lungs & the presence of suppurative U 721 1
exudate filling up the bronchi, bronchioles, & adjacent alveolar
PATHO BOARD EXAM QUESTIONS 35
spaces are characteristics seen in:
A. tuberculosis
B. lobar pneumonia
C. primary atypical pneumonia
D. bronchopneumonia

56. The pathogenesis of acute respiratory distress syndrome is U


associated with
A. COPD
B. Multiple pulmonary emboli
C. Endothelial and epithelial injury
D. Antedent pulmonary hypertension

57. A 76-year old female developed Staphyloccoccal pneunomia for A 0.75


which she was given a course of antibiotic therapy. Ten days later,
she no longer had productive cough but fever persisted. Chest
x-ray showed a 3-cm, rounded density in the right lower lobe
whose liquefied contents form a central air-fluid level. Which of the
following outcomes of her pneumonia occurred?
A. abscess formation
B. regeneration
C. progression to chronic inflammation
D. fibrosis

58. Mang Juanito, 60-year-old man has been smoking for many years A
he had chronic cough and weight loss for the past 6 months, no fever
no nausea and vomiting. He had one bout of hemoptysis and went
to the ER for consultation. X-ray shows a 6 cm mass on the
medial upper lobe. Bronchoscopy shows a mass on the segmental
bronchus. Which of the following cytologic findings is likely to be
found in this patient?
A. Presence of acid fast organism on sputum exam
B. Presence of malignant squamous cells in sputum
C. Presence of numerous necrotic debris and inflammatory cell
in sputum
D. Presence of reactive mesothelial cells in pleural fluid exam

59. Mang Juan, 65-year-old male with a history if smoking with A


productive cough for many years came in to the emergency room
with difficulty of breathing. Chest X-ray shows prominent blood
vessels and large heart. Patient went into cardiorespiratory arrest.
Resuscitation done but to no avail. Histologic section taken from
the lung at bronchi. What is the most likely diagnosis?
A. Emphysema
B. Bronchial asthma
C. Chronic bronchitis
D. Broncchlectasis

60. The most characteristic microscopic finding of Retinoblastoma is: R 1373 0.33
A. Homer Wright rosette
B. Flexner Wintersteiner rosette
C. Blastema cells
D. Rosenthal fibers

61. True statements concerning sialolithiasis include: R


A. It ccurs most often in the parotid gland
B. It presents clinically with painless swelling
C. It is treated successfully with antibiotics
D. It occurs most often in the submandibular gland

62. A 42 year old male patient comes in with a chief complaint of A Robbins 0.8
"mouth sore." Patient has been a smoker since 17 years old 7th ed.
This lesion is most likely: 776-780
A. Hairy leukoplakia
B. Erythroplakia
PATHO BOARD EXAM QUESTIONS 36
C. Aphtous ulcer
D. Leukoplakia

63. A 46 year old female has keratoconjunctivitis. She also has oral A 0.45
mucosal atrophy with buccal mucosal ulceration. A biopsy of her
lip reveals marked lymphocytic and plasma cell infiltrates in minor
salivary glands. Which of the following antibodies is most likely to
be found for this patient?
A. anti-centromre antibody
B. SS-B
C. Scl-70
D. anti-ds DNA

64. Which organism is associated with traveller's dearrhea? R 808 0.5


A. Salmonella typhi
B. Campylobacter jejuni
C. escherichia coli
D. Shigella

65. Which is the definitive finding in acute appendicitis: R 839 1


A. obstruction of the lumen by a fecalith
B. mucus inspissation in the lumen
C. neutrophils in the muscularis
D. serosal congestion and margination of polys

66. A 6-year-old boy develps abdominal pain and vomiting preceded A


by a 3-day period without bowel movements. A diagnosis of
intestinal obstruction is made. Of the following, the most likely
reason for the obstruction is a bolus of:
A. Taenia saginata
B. Ascaris lumbricoides
C. Strongngyloides tercoralis
D. Onchocerca volvulus

67. Which of the following accumulates in the cytoplasm of liver cells R


in steatosis:
A. sphingolipids
B. phosholipids
C. cholesterol/cholesterol esters
D. triglycerides

68. Nelia, 38-year-old, female, complains of fatigue and pruritus. She A


is found to have hig serum alkaline phosphatase and slightly
elavated serum bilirubin levels, and serum antimitochondrial
autobodies are present. A liver biopsy reveals a marked
Lymphocytic infiltrate in the portal tracts. Occaional granioumas
are also seen. The most likely diagnosis is:
A Primary Sclerosing cholangitis
B. Primary biliary cirrhosis
C. Viral hepatitis B infection
D. Impacted galistoric

69. Imelda, 6-years-old, girl, had a blocky, reddish brown rash and A
was treated with aspirin, she developed fatty change of the liver.
The most likely diagnosis is:
A. Subacute Sclerosing panencephalitis
B. Varicella-Zoster infection
C. Reye's syndrome
D. Poliomyelitis

70. Carcinoma of the pancreas: R


A. arises from pancreatic acinar cells
B. occurs most often in the tail of the pancreas
C. is associated with spontaneous venous thrombosis
D. is usually cured by total pancreatectomy
PATHO BOARD EXAM QUESTIONS 37
71. Which is seen in the pancreas of infancts of diabetic mothers? U
A. multiple adenomas
B. solitary adenoma
C. diffuse hyperplasia
D. carcinoma

72. The most characteristic histologic lesion of acute pancreatitis is: U 905 0.5
A. Neutrophilic infiltration
B. Hemorrhage
C. Fat necrosis
D. Necrosis of parenchyma

73. Diffuse effacement of foot processes is characteristic of: R


A. idiopathic rapidly progressive glomerulonephritis
B. lipoid nephrosis
C. IgA nephropathy
D. lupus nephritis

74. Membranous glomerulopathy and poststreptococcal U 953 0.5


glomerulonephritis are similar in that they both:
A. are self-limiting and transient renal disease
B. are commonly associated with nephrotic syndrome
C. are most commonly seen in children
D. have glomerular immune complex deposits

75. A 28 year old primigravida had hypotensive shock because of A 969-971 0.5
postpartum hemorrhage. Her urine output ranged from 80-200 ml/
24 hours. The basic kidney lesion to explain this decreased urine
output is:
A. Hemolytic-uremic syndrome
B. Obstructive uropathy
C. Acute tubular necrosis
D. Acute tubulointerstitial nephritis

76.Which of the following agents is thought to be the cause of cancer R


of the urinary bladder?
A. Cyclasmates
B. Dietary tat
C. Vinyl chloride
D. Asbestos

77. A 62-year old male who presented with hematuria was recently A 0.7
diagnosed with a grade IV transitional cell carcinoma that has
infiltrated through the bladder wall. Which of the following
statements regarding the findings is TRUE?
A. He has pooly differentiated tumor
B. The neoplasm is metastatic
C. The stage of this tumor is low
D. A paraneoplastic syndrome is apparent

78. A 30 year old man had an orchietomy for testicular cancer. Which R Robbins 60
biologic marker is NOT valuable in the diagnosis management? 7th ed.
A. LDH p.1045
B. AFP
C. HCG
D. Estrogen

1018-
79. A 30 year old male present with a testicular mass that is A 1023 0.7
suspected to be a malignant tumor. Abdominal computed
tomography reveals enlargement of para aortic lymph nodes and
multiple lung nodules are seen on chest x ray. Serum levels of
HCG and AFP are markedly increased. Which of the following
features is not consistent with the diagnosis of a classic
PATHO BOARD EXAM QUESTIONS 38
seminoma?
A. age
B. presence of lung metastasis
C. enlargement of para aortic lymph nodes
D. elevations in hcg and afp

80. Characteristic histopathologic change associated with HPV R


uterine cervix infection is:
A. thickening of stratum corneum with production of qualitative
abnormal keratin
B. retention of the nuclei in the cell of the stratum corneum
C. kollocytic atypia
D. epidermal thickening

81. The nonpregnant uterus of a 23-year old female measured 8 x 5x4 U 0.85
cm. Which of the following changes explains the increase in the
size of the uterus?
A myometrial smooth muscle hypertrophy
B. endometrial stromal hypertrophy
C. endometrial glandular hyperplasia
D. vascular endothelial hyperplasia

82. A 56 year old postmenopausal female, G3P3, consulted because A Robbins 0.7
of abdominal pain. She also complained of vaginal spotting. 7th ed.
Speculum examination showed whitish, mucoid discharged. On IE p 1100
uterus was not enlarged but a right adnexal mass was felt. Which
of the following ovarian tumors is LEAST likely in this
postmenopausal patient?
A. Dermoid cyst
B. Immature teratoma
C. Granulosa cell tumor
D. Mucinous cystadenoma

83. The most important prognostic feature in invasive breast cancer is R


A. the grade of the tumor
B. the histologic type of the tumor
C. the size of the tumor
D. the status of the draining lymph nodes

84. The usual presentation of bloody nipple discharge is found in: U


A. Intraductal papilloma
B. Fibroadenoma
C. Fibrocystic change
D. Phylloides tumor

85 A 44 year old female was diagnosed with breast carcinoma. The U


affected left breast is covered by thickened, pitted overlying skin
and is slightly larger than the uninvolved right breast. How is the
gross appearance of the left breast best explained?
A. chronic passive congestion
B. chronic inflammation
C. lymphatic obstruction
D. venous thrombosis

1102-
86. A 5 cm well-encapsulated mass was excised from the breast of a A 1103 0.25
21-year-old woman. Histologically, the mass was composed of
elongated, ductlike structures surrounded by loose, fibrous
connective tissue. What is the diagnosis?
A. fibroadenoma
B. intraductal papilloma
C. Pagets disease
D. Phyllodes tumor

87. A 30 year old woman presented with a chief complaint of an U Robbins 0.8
PATHO BOARD EXAM QUESTIONS 39
anterior neck mass noted to be enlarging for the last 4 years. Past 7th ed.
1177-
medical history indicates previous treatment for acne. On PE, the 1180
mass measured 2.2 x 2.0 x 1.5, firm, non-tender. What would be
primary consideration amont the following disease entities?
A. Anaplastic thyroid carcinoma
B. Medullary thyroid carcinoma
C. Papillary thyroid carcinoma
D. Follicular thryroid carcinoma

88. A 30 year old female complained of loss of conciousness. U 926 0.5


History revealed that the episode were precipitated by fasting.
Laboratory exam at the time of attack showed glucose level at
at less than 50mg/dl. This is usually associated with
A. Gastrinoma
B. Insulinoma
C. Glucagonoma
D. Somatostatinoma

89. A 20-year-old female died suddenly and unexpectedly after A Robbins 0.9
complaining only of a mild sore throat the day before. At autopsy, 7th ed.
her adrenal glands are enlarged and have massive bilateral cortical p.1214
hemorrhage. Infection with which of the following organisms best
accounts for these findings?
A. Cytomegalovirus
B. Histoplasma capsulatum
C. Neisseria meningitides
D. Streptococcus pneumonia

90. The most commonly accepted cause of squamous cell R 1185 0.5
carcinoma of the skin is:
A. Exposure to ultraviolet light
B. Exposure to arsenic
C. Cutaneous scars
D. PUVA

91. Healing by 2nd intention resulting in exuberant granulation tissue U


formation that blocks reepithelialization is characteristic of
A. scab
B. keloid
C. repair
D. proud flesh

92. Tuberculosis osteomyelitis usually arises: R


A. spontaneoulsy
B. from a focus on the pelvis
C. from a focus in the spinal cord
D. From hematogenous dissemination from a focus of active
visceral disease

93. The most likely malignant bone tumor in a 65 y/o man is: R 1245 1
A. Giant cell tumor
B. Osteosarcoma
C. Metastatic prostatic cancer
D. Multiple myeloma

94. An 8 year old female can bend her thumb back to touch her A
forearm. She can pull her skin out from her abdomen about 8 cm
and a cut to her skin gapes open and is difficult to repair. Her
underlying disease process results from an inherited defect in
A. LDL receptor
B. factor VIII
C. alpha 1-antitrypsin
D. collagen

PATHO BOARD EXAM QUESTIONS 40


95. Involvement of the dorsal root ganglia is characteristic of R
A. rabies
B. H. simplex
C. Poliomyelitis
D. Subacute panencephalitis

96. Ocular muscle weakness and general fatigability were found in a A 1289 1
38 year old woman who also had antibodies to acetylcholine
receptors. What is the diagnosis?
A. Grave disease
B. Polymyositis
C. Myasthenia gravis
D. Wiscott-Adrich syndrome

97. A 17 year old female college student has 14 scattered 2- to 5-cm A Robbins 0.9
flat, hyperpigmented skin lesion with irregular borders on her 7th ed.
extremities and torso. She had decreased vision year ago, and an 1411
optic nerve glioma was excised. She now presents with a mass
involving the left wrist. The histopathologic exam of the mass is
most likely to show
A. Fibrosarcoma
B. Angiomyolipoma
C. Meningioma
D. Schwannoma

98. Which is found in the hippocampus and purkinje cells of the


cerebellum R 1331 0.33
of patients with Alziemer's Disease
A. Cowdry bodies
B. Negri bodies
C. Lewy bodies
D. Hirano bodies

99. Brain infact associated with TB meningitis is a consequence of U


A. arteris
B. brain edema
C. sinus thrombosis
D. caseous necrosis

100. A 6-cm mass beneath the dura that compresses the underlying
left lateral parietal lobe is found incidentally on head CT scan done
on a 40-year old female who fell and hit her head. The mass is
surgically removed and sent for histopath. Morphologically, the
mass consist of elongated cell with pale oblong nuclie and pink
cytoplasm with occasional psammoma bodies. The most likely
diagnosis is:
A. Ependymoma
B. Medulloblastoma
C. Meningioma
D. Schwannoma

BLUEPRINT FOR QUESTIONS

PATHOLOGY

SUBJECT MATTER QUESTION NO. RECALL COMPRE


HENSION
1. Cellular Pathology, Inflammation and
Tissue Repair (12) 1-12 12
2. Genetic disorders (2) 13-14 2

PATHO BOARD EXAM QUESTIONS 41


3. Diseases of Immunity (4) 15-18 4
4. Neoplasia (5) 19-23 5
5. Infectious Diseases (2) 24-25 2
6. Environmental and Nutritional
Pathology (4) 26-29 4
7. Blood Vessels (2) 30-31 2
8. The Heart (4) 32-35 4
9. Red Cells and Bleeding Disorders (8) 36-43 8
10. White Cells, Lymph Nodes, Spleen &
Thymus (5) 44-48 5
11. The Lungs (6) 49-54 6
12. Gastrointestinal Tract (6) 55-60 6
13. The Liver and Biliary Tract (5) 61-65 5
14. Pancreas (2) 66-67 2
15. Kidney and Lower Urinary Tract (5) 68-72 5
16. Genital Tract (10) 73-82 10
17. Breast (2) 83-84 2
18. Endocrine System (5) 85-89 5
19. Bones, Joints and Soft Tissue
Tumors (7) 90-96 7
20. The Central nervous System (4) 97-100 4
TOTAL 100

MEAN MPL FOR THIS EXAM: 76.89

PATHO BOARD EXAM QUESTIONS 42


CEBU INSTITUTE OF MEDICINE

Department of Pathology

NAME _________________________________ GROUP NO. ____ I.D. NO. ________

DIRECTION: Each question below is followed by four possible answers. Select the one best response and
mark the appropriate spaces on the answer sheet provided for.

1. The morphologic pattern of necrosis in myocardial infarction is :


A. Coagulative necrosis C. Fat necrosis
B. Liquefactive necrosis D. Caseation necrosis
(Answer: A/ Reference: p. 17 / MPL 90)

2. The most common cause of fatty change in the liver is:


A. CC14 poisoning C. Diabetes mellitus
B. Alcohol abuse D. Protein malnutrition
(Answer: B/Reference: p. 39 / MPL 90)

3. The cellular adaptation that occurs in the respiratory tract of a chronic cigarette smoker is:
A. Hyperplasia C. Metaplasia
B. Hypertrophy D. Atrophy
(Answer: C/Reference: p. 36 / MPL 90)

4. The first manifestation of almost all forms of cell injury is:


A. Fatty change C. Dystrophic caicification
B. Hyaline change D. Cellular swelling
(Answer: D/Reference: p. 7 / MPL 90)

5. Refers to the focal accumulation of cholesterol-laden macrophages in the lamina propia of


the gallbladder:
A. Steatosis C. Xanthoma
B. Cholesterolosis D. None of these
(Answer: D/Reference: p. 895 / MPL 90)

6. Which is granulomatous disease?


A. Leprosy C. Tuberculosis
B. Schistosomiasis D All of these
(Answer: D/Reference: p. 65 / MPL 90)

7. Antipyretics inhibit inflammatory process by inhibiting:


A. Cyclooxygenase pathway C. Lipooxygenase pathway
B. Dismutase D. A & C
(Answer: A/Reference: p. 50 / MPL 33)

8. Chronic granulomatous disease is characterized by impaired microbicidal ability


due to:
A. NADH oxidase deficiency C. Lack of adhesion molecules
B. T cell immunodeficiency D. Complement deficiency
(Answer: A/Reference: p. 65 / MPL 33)

9. Increased susceptibility to infection in diabetic patients is due to:


A. Deficiency of emigration and chemotaxis of neutrophils
B. B cell deficiency
C. Impaired microbicidal activity
D. Nitric oxide deficiency
PATHO BOARD EXAM QUESTIONS 43
(Answer: A/Reference: p. 55 / MPL 60)

10. Which is not associated with chronic inflammation?


A. Fibrosis C. Granuloma
B. Neutrophilic infiltration D. Macrophages and epithelioid cells
(Answer: B/Reference: p. 79 / MPL 60)

11. Redness is:


A. Calor C. Dolor
B. Tumor D. None of these
(Answer: D/Reference: p. 50 / MPL 90)

12. Brown atrophy of the heart in aging is due to:


A. Amyloid deposits C. Hemosiderin pigments
B. Lipofuscin D. Basophilic degeneration
(Answer: B p 26 /MPL 90 )

13. 47, XXY is :


A. Cri-du-chat syndrome C. Turner’s syndrome
B. Klinefelter’s syndrome D. Normal female
(Answer: B p 170/MPL 60)

14. Tall stature and arachnodactyly are characteristic of:


A. Ehlers-Danlos syndrome C. Prader-Willi syndrome
B. Trisomy 21 D. Marfan syndrome
(Answer: D p 147 /MPL 90 )

15. Transplant rejection is which type of hypersensitivity reaction?


A. type I C. type III
B. type II D. type IV
(Answer: D p 206 /MPL 60 )

16. Which is NOT a major (Jones) criterion of rheumatic fever?


A. Carditis C. arthralgia
B. erythema marginatum D. sydenham’s chorea
(Answer: C p 570 /MPL 90)

17. Thymic hypoplasia is a feature of:


A. Sjogren’s syndrome C. Di George syndrome
B. SLE D. Wiskott-Aldrich syndrome
(Answer: C p 173 /MPL 60 )

18. CREST syndrome is related to:


A. Hyper-IgM syndrome C. scleroderma, localized
B. Bruton’s hypogammaglobulinemia D. preeclampsia
(Answer: C p 226 /MPL 33)

19. Which is not oncogenic?


A. Human papilloma virus C. Herpes virus
B. Hepatitis B virus D. Epstein-Barr virus
(Answer: C/Reference: p. 311 / MPL 60)

20. Which is not a malignant tumor?


A. Lymphoma C. Leiomyosarcoma
B. Melanoma D. Hamartoma
(Answer: D/Reference: p. 261 / MPL 90)

21. Features of malignant cells, EXCEPT:


A. Hyperchromatic and pleomorphic nuclei C. Anaplasia
B. Tumor giant cell D. No exception
PATHO BOARD EXAM QUESTIONS 44
(Answer: D p 264/MPL 90)

22. Leading type of cancer in men:


A. skin cancer C. lung cancer
B. prostate cancer D. testicular cancer
(Answer: B p 261 /MPL 60)23.

23. Which of the following refers to the degree of cellular differentiation of tumor
cells?
A. Stage C. grade
B. Metastasis D. Doubling time
(Answer: C p 261 /MPL 90)

24. Warthin-finkeldey cells are seen in:


A. Chicken pox C. herpes simplex infection
B. Mumps D. Measles
(Answer: D p 370 /MPL 60)

25. Ghon complex is associated with:


A. Leprosy C. HIV infection
B. Tuberculosis D. all of these
(Answer: B p 723/MPL 90)

26. Ultraviolet rays are implicated in:


A. Gastric cancer C. Squamous cell carcinoma, skin
B. Urinary bladder cancer D. Bronchogenic carcinoma
(Answer: C p 310/MPL 90)

27. Trenchfoot is due to:


A. Blast injury C. greenhouse effect
B. Mercury poisoning D. Hypothermia
(Answer: D p 435/MPL 90)

28. The disease of the displaced child:


A. Marasmus C. Keshan diseae
B. Kwashiorkor D. Bulimia
(Answer: B p 437 /MPL 90 )

29. Bends and chokes are features of:


A. Fat embolism C. air embolism
B. Paradoxic embolism D. amniotic fluid embolism
(Answer: C p 131 /MPL 90)

30. Risk factors for atherosclerosis:


A. Cigarette smoking C. Diabetes
B. Hypertension D. All of these
(Answer: D p 498 /MPL 90 )

31. Which typically presents with headache and facial pain?


A. Temporal arteritis C. Kawasaki disease
B. Polyarteritis nodosa D. Takayasu arteritis
(Answer: A p 516 /MPL 60)

32. Rib-notching on chest x-ray is a finding in:


A. Tetralogy of Fallot C. aortic coarctation
B. Patent ductus arteriosus D. all of these
(Answer: C p 596 /MPL 90)

33. Mid-systolic click can be heard in:


PATHO BOARD EXAM QUESTIONS 45
A. Mitral valve prolapse C. Ventricular septal defect
B. Atrial septal defect D. Carcinoid heart disease
(Answer: A p 568 /MPL 90 )

34 Coronary artery spasm is noted in:


A. Stable angina C. unstable angina
B. Prinzmetal angina D. All of these
(Answer: B p 554 /MPL 60)

35. Troponin rises after:


A. Acute myocardial infarction C. Pancreatitis
B. Angina pectoris D. Liver necrosis
(Answer: A p 561 /MPL 90)

36. Hemoglobin H disease is a variant of:


A. Alpha-thalassemia C. Beta-thalassemia
B. Sickle cell disease D. Immunohemolytic anemia
(Answer: A p 619 /MPL 60)

37. Hemolysis with antimalarial drug administration can occur in:


A. Iron deficiency anemia C. mycosis fungoides
B. G6PD deficiency D. Pernicious anemia
(Answer: B p 610 /MPL 90)

38. Pancytopenia is a feature of:


A. Paroxysmal nocturnal hemoglobinuria C. Folate deficiency
B. Hereditary spherocytosis D. Aplastic anemia
(Answer: D p 630 /MPL 90)

39. Which can lead to DIC?


A. Septic abortion C. Gram negative sepsis
B. Extensive surgery D. All of these
(Answer: D p 640 /MPL 90)

40. Which causes relative polycythemia?


A. High-altitude living C. Gaisbock syndrome
B. Renal cell carcinoma D. Cyanotic heart disease
(Answer: C p 633 /MPL 90)

41. The most common cause of microcytic, hypochromic anemia is:


A. Vitamin B12 deficiency C. Pure red cell aplasia
B. Iron deficiency D. Leukemia
(Answer:B p 627 /MPL 90 )

42. Evaluates the intrinsic coagulation pathway and is most commonly used to
monitor heparin therapy:
A. Prothrombin time C. APTT
B. Bleeding time D. Clotting time
(Answer: C p 633 /MPL 60)

43. Defective platelet aggregation occur in:


A. Glanzmann’s thrombasthenia C. Bernard-Soulier syndrome
B. Henoch-Schonlein purpura D. vonWillebrand disease
(Answer: A p 122 /MPL 60)

44. Which is not a cause of eosinophilic leukocytosis?


A. Allergic disorders C. Viral infection
B. Parasitism D. drug reaction
PATHO BOARD EXAM QUESTIONS 46
(Answer: C p 647 /MPL 90)

45. M spike on protein electrophoresis is noted in:


A. Multiple myeloma C. Hodgkin’s disease
B. Burkitt’s lymphoma D. All of these
(Answer: A p 664 /MPL60)

46. Auer rods are diagnostic of:


A. Acute myeloblastic leukemia C. Acute lymphoblastic leukemia
B. Chronic lymphocytic leukemia D. Chronic myelocytic leukemia
(Answer: A p 678 /MPL 90)

47. Reed-Sternberg cells are diagnostic of:


A. non-Hodgkin’s lymphoma C. Myelodysplastic syndrome
B. Polycythemia vera D. None of these
(Answer: D p 670 /MPL 90)

48. Variants of Langerhans cell histiocytosis include the ff., EXCEPT:


A. Eosinophilic granuloma C. Letterer-Siwe disease
B. Myelofibrosis with myeloid metaplasia D. Hand-Schuller disease
(Answer: B p 685 /MPL 60)

49. Cor pulmonale is associated with:


A. L-sided heart failure C. Both A & B
B. R-sided heart failure D. Neither A nor B
(Answer: B p 565 /MPL 60)

50. Presence of rheumatoid nodules and pneumoconiosis constitutes:


A. Goodpasture’s syndrome C. Loeffler’s syndrome
B. Mickulicz syndrome D. Caplan syndrome
(Answer: D p 729 /MPL 60)

51. May produce pulmonary hypertension, EXCEPT:


A. Mitral stenosis C. Emphysema
B. Recurrent pulmonary emboli D. No exception
(Answer: D p 704 /MPL 60)

52. Low levels of serum alpha-1 antitrypsin can cause:


A. Bronchiectasis C. Chronic bronchitis
B. Emphysema D. All of these
(Answer: B p 707 /MPL 60)

53. Intrinsic asthma is initiated by:


A. Pulmonary viral infection C. Inhaled irritants
B. Aspirin ingestion D. All of these
(Answer: D p 707 /MPL 60)

54. True of adult respiratory distress syndrome, EXCEPT:


A. Diffuse alveolar capillary damage
B. Deficiency of pulmonary surfactant
C. Noncardiogenic pulmonary edema
D. No exception
(Answer: B p 760 /MPL 60)

55. Failure of involution of the vitelline duct leads to:


A. Herniation C. Meckel’s diverticulum
B. Volvulus D. All of these
(Answer: C p 804 /MPL 90)

PATHO BOARD EXAM QUESTIONS 47


56. One segment of the small intestine constricted by a wave of peristalsis suddenly
became telescoped into the distal segment happen in:
A. Intussusception C. Volvulus
B. Diverticula D. Strangulation
(Answer: A p 804 /MPL 90)

57. Absence of ganglion cells in the large bowel leading to functional obstruction
with colonic dilatation:
A. Achalasia C. Hirschsprung disease
B. Ulcerative colitis D. Angiodysplasia
(Answer C: p 805 /MPL 90)

58. A portion of the abdominal wall fails to form altogether with extrusion of the
intestine:
A. Omphalocele C. Meckel’s diverticulum
B. Herniation D. Gastroschisis
(Answer: D p 804 /MPL 60)

59. Complications of colonic diverticula, EXCEPT:


A. Obstruction C. Bleeding
B. Peritonitis D. Malignancy
(Answer: D p 823 /MPL 60)

60. Causes of hematemesis, EXCEPT;


A. Mallory-Weiss syndrome C. Peptic ulcer
B. Esophageal Varices D. Barrett’s esophagus
(Answer: D p 783 /MPL 60)

61. Nutmeg liver is due to:


A. Inflammation C. congestion
B. Neoplasm D. cirrhosis
(Answer: C p 117/MPL 90)

62. Bronze diabetes refers to:


A. Wilson’s disease C. Primary hemochromatosis
B. Budd-Chiari syndrome D. Reye’s syndrome
(Answer: C p 873 / MPL 60)

63. There is conjugated hyperbilirubinemia in:


A. Dubin-Johnson’s syndrome C. Rotor’s syndrome
B. Gilbert’s syndrome D. A & C
( Answer : D p 850 /MPL 90)

64. Most common predisposing cause of ascending cholangitis:


A. Acute pancreatitis C. Choledocholithiasis
B. Benign stricture D. Tumors
(Answer: C p 879 /MPL 90)

65 . Rokitansky-Aschoff sinuses are seen in:


A. Rheumatic heart disease C. Biliary atresia
B. Chronic Cholecytitis D. Criggler-Najjar syndrome
(Answer: B p 892 /MPL 90)

66. Etiologic factors in acute pancreatitis include the ff., EXCEPT:


A. Alcoholism C. Gall stones
B. Genetic D. No exception
(Answer: D p 904 /MPL 90)

67. Late complications of chronic pancreatitis include the ff., EXCEPT:\


A. Diarrhea (Malabsorption) C. Diabetes
PATHO BOARD EXAM QUESTIONS 48
B. Pseudocyst D. Hemorrhage
(Answer: D p 907 /MPL 90)

68. Azotemia, hypocalcemia and hyperparathyroidism are associated with:


A. Acute renal failure C. Nephretic syndrome
B. Chronic renal failure D. Nephrotic syndrome
(Answer: B p 935 /MPL 60)

69. Associated with Proteus infection, also called Staghorn calculi:


A. Calcium stones C. Struvite stones
B. Uric acid stones D. None of these
(Answer: C p 989 /MPL 90)

70. Which is not associated with diabetic nephropathy?


A. Kimmelsteil-Wilson disease C. Papillary necrosis
B. Acute pyelonephritis D. Renal artery stenosis
(Answer: D p 966 /MPL 60)

71. IgA deposit in mesangium can be appreciated by:


A. Immunofluorescence microscopy C. Light microscopy
B. Electron microscopy D. All of these
(Answer: A p 961 /MPL 60)

72 .Nephritic factor, dense deposits are related to:


A. Membranoproliferative glomerulonephritis
B. Lipoid nephrosis
C. Focal segmental glomerulonephritis
D. Acute post-streptococcal glomerulonephritis
(Answer: A p 958 /MPL 90)

73. Maturation index determination is done with:


A. Paps smear C. Tzanck smear
B. Gram stain D. All of these
(Answer: A p 322 / MPL 90)

74. Orifice of the prepuce is too small to permit its normal retraction:
A. Epispadias C. Paraphemosis
B. Chondyloma D. None of these
(Answer: D p 1012 / MPL 60)

75. Positively identified as cause of the cancer of the cervix:


A. Cytomegalovirus C. Herpes virus
B. Candidiasis D. Human papilloma virus
(Answer: D p 1012 /MPL 60)

76. Features of toxemia of pregnancy, EXCEPT:


A. Anemia C. Hypertension
B. Edema D. Proteinuria
(Answer: A p 1082 / MPL 90)

77. Hormone used to determine pregnancy:


A. FSH C. Progesterone
B. Beta-HCG D. All of these
(Answer: B p 1079 / MPL 90)

78. Not seen in secretory endometrium:


A. Mitosis and gland proliferation C. Secretion
B. Subnuclear vacuole D. Prominent spiral arteriole
(Answer: A p 1037 / MPL 90)

PATHO BOARD EXAM QUESTIONS 49


79. The most common complication of gonorrhea in women:
A. Endometriosis C. Vulvovaginitis
B. Endometritis D. Pelvic inflammatory disease
(Answer: D p 1017 / MPL 90)

80. All are important in semen analysis, EXCEPT:


A. Sperm count C. Volume
B. Morphology D. Motility
(Answer: C p 510 / MPL 90)

81. Most common site of endometriosis:


A. Ovaries C. Peritonium
B. Uterine ligament D. Rectovaginal septum
(Answer:A p 1057 / MPL 90)

82. Clinical indicator of hyperestrinism:


A. Mastitis C. Nipple inversion
B. Fat necrosis D. Gynecomastia
(Answer: D p 1117 / MPL 90)

83. Morphologic patterns of fibrocystic change of the breast, EXCEPT:


A. Adenosis C. Fibrosis
B. Cyst formation D. Papilloma
(Answer: D p 1098 / MPL 90)

84. Acquired retraction of the nipple occur in:


A. Lactation C. Breast cancer
B. Silicon breast implant D. All of these
(Answer: C p 1104 / MPL 90)

85. There is insulin resistance in:


A. Type 1 diabetes mellitus C. A & B
B. Type 2 diabetes mellitus D Neither A nor B
(Answer: B p 913 /MPL 9 )

86. Ischemic necrosis of the anterior pituitary leads to:


A. Cushing’s disease C. Sheehan syndrome
B. SIADH D. Cretinism
(Answer: C p 642 /MPL 90)

87. Believe to be cause by viral infection or post viral inflammatory process:


A. Subacute granulomatous thyroiditis C. Reidel thyroiditis
B. Hashimoto’s thyroiditis D. All of these
(Answer: A p 1135 / MPL 60)

88. Most frequent type of hyperfunctioning pituitary adenoma:


A. Somatostatinoma C. Prolactinoma
B. Corticotroph adenoma D. Thyrotroph adenoma
(Answer: C p 1123 / MPL 60)

89. Chronic adrenal insufficiency leads to:


A. Waterhouse-Friederichsen syndrome C. Cushing’s syndrome
B. Addison’s disease D. Hyperaldosteronism
(Answer: B p 1160 / MPL 90)

90. A cause of primary osteoporosis:


A. Post menopausal C. Malnutrition
B. Hyperparathyroidism D. Rheumatologic disease
(Answer: A p 1222 / MPL 90)

PATHO BOARD EXAM QUESTIONS 50


91. A genetic derangement in epiphysial cartillagenous growth resulting in dwarfism:
A. Osteogenesis imperfecta C. Achondroplasia
B. Osteopetrosis D. Paget’s disease
(Answer: C p 1220 /MPL 60)

92. Failure in bone mineralization resulting in exist unmineralize matrix and


abnormally wide osteioid seems occur in:
A. Rickets C. Osteomalasia
B. Ostitis deformans D. A & C
(Answer: D p 1227 / MPL 60)

93. Saber shin is due to:


A. Rickets C. Syphilis
B. Pyogenic osteomyelitis D. Pott’s disease
(Answer: C p 1233 / MPL 90)

94. Chronic tophaceous gout and arthritis occur in:


A. Hyperuresimia C. Hyperglycemia
B. Hypercalcemia D. Hyperproteinemia
(Answer: A p 1253 / MPL 60)

95. Calcium pyrophosphate deposition occur in:


A. Gout C. Pseudogout
B. Rheumatoid arthritis D. All of these
(Answer: C p 1257 /MPL 90)

96. Degenerative joint disease is:


A. Rheumatoid arthritis C. Gouty arthritis
B. Marie-Strumpel disease D. Osteoarthritis
(Answer: D p 1246 / MPL 90)

97. Infection of CNS in neonate during delivery occur in which condition:


A. Syphilis C. Gonorrhea
B. Herpes simplex condition D. Chlamydial infection
(Answer: B p 1318 / MPL 60)

98. CSF albuminucytologic dissociation occur in:


A. Multiple sclerosis C. Guillain- Barre syndrome
B. Pyogenic meningitis D. Subarachnoid hemorrhage
(Answer: C p 1275 / MPL 60)

99. The counterpart of mad cow disease in human is:


A. Creutzfetldt-Jacob disease C. Rabies
B. Progressive multifocal leucoencephalopathy D. None of these
(Answer: C p 1323 / MPL 90 )

100. Gemistocytic astrocytes are noted in:


A. Hypoglycemia C. Damaged brain
B. Ethanol abuse D. All of these
(Answer: C p 1297 / MPL 60)

References:

1. Robbin’s Pathologic Basis of Disease 6th Edition

2. Henry’s Clinical Diagnosis and Management 19th Edition

PATHO BOARD EXAM QUESTIONS 51


Blueprint for Questions on General Pathology (2A)

The Following topics are derived for the General Objectives for General Pathology (2A):

TOPICS NO. OF QUESTIONS

1. CELL INJURY, ADAPTATION, AND DEATH:


4
1.1 Mechanisms of cell injury
1.2 Cell adaptation to injury
1.3 Reversible injuries
1.4 Irreversible injuries (necrosis)
1.5 Apoptosis
1.6 Cellular aging
2. ACUTE AND CHRONIC INFLAMMATION: 4
2.1 Acute inflammation ( vascular and cell events, chemical mediators)
2.2 Chronic inflammation (cells, mediators, chronic granulomateus)
3. TISSUE REPAIR : CELL REGENERATION AND FIBROSIS:
3
3.1 Cell regeneration (mediations, matrix and cell matrix interaction)
3.2 Repair by connective tissue (fibrosis)
3.3 Growth factors in cell regeneration and fibrosis
3.4 Wound healing
4. HEMODYNAMIC DISORDERS, THROMBOSIS, AND SHOCK: 3
4.1 Edema 4.4 Embolism
4.2 Hyperemia and congestion 4.5 Infarction
4.3 Hemostasis and thrombosis 4.6 Shock
5. DISEASES OF IMMUNITY: 3
5.1 Cells of the immune system 5.4 Hypersensitivity
5.2 Histocompatibility molecules ( I, II, III, IV,)
5.3 Cytokines 5.5 Autoimmune desease
5.6 Immunodeficiency diseases
6. NEOPLASIA: 3
6.1 Nomenclature
6.2 Characteristics of benign and malignant tumors
6.3 Epidemiology
6.4 Molecular basis of cancer
6.5 Etiology of cancer
6.6 Host defense vs. tumor
6.7 Clinical features of neoplasia
7. GENETIC AND PEDIATRIC DISEASES: 3
7.1Genetics 7.2 Pediatrics
7.11 Mutation 7.21 Congenital anomalies
7.12 Mendelian disorders 7.22 Prematurity and intra-uterine
7.13 Multifactorial disorders growth retardation
7.14 Cytogenetic 7.23 Respiratory distress in newborn
7.15 Single gene disorders with atypical 7.24 Tumor & tumor like disorder
inheritance 7.25 Diagnosis of genetic disorders
8. ENVIRONMENTAL DISEASES: 4
8.1 Environmental Pollution 8.5 Electrical injury
8.2 Tobacco smoke 8.6 Ionizing radiation
8.3 Chemical injury 8.7 Nutritional diseases
8.4 Physical injury
9. GENERAL PATHOLOGY OF INFECTIOUS DISEASES: 3
9.1 New & emerging infectious disease 9.5 Immune evasion by microbes
9.2 Categories of infectious disease 9.6 Special diagnostic tests
9.3 Host barrier to infection 9.7 Inflammatoryresponse to infect. agents
9.4 How infect. agents cause disease
PATHO BOARD EXAM QUESTIONS 52
______________ Total
Questions: 30

Questions on General Pathology (2A)


(40 out of total 100 Pathology Questions)

Objectives where questions are derived Questions


(note: Key of questions are in separate sheet)

1. Cell injury, adaptation and death 1. The type of necrosis where there is preservation of the
basic outline of the dead cells for a few days is noted
in :
A. Caseation C. Liquefaction
B. Coagulation D. Gangrenous
2. Phospatidyl serine has “flipped out” from the inner layers
of the apoptotic cell in which mechanism of apoptosis?
A. Signaling pathway C. Execetion phase
B. Control & regulation D. Removal phase
3. Which organ is seriously unjured in cases of
hemochromatosis?
A. Heart C. Kidney
B. Liver D. Lungs
4. The susceptibility of cardiac muscles to ischemia is how
long?
A. 3-5 minutes C. 20-40 minutes
B. 10-20 minutes D. 1-2 hours
2. Acute and chronic inflammation 5. Which is true of chronic inflammation?
A. Infiltration by neutrophils
B. Hallmark is tissue destruction
C. Most important cell is the plasma cell
D. The initial phase is vasodilation
6. The histologic hallmark of a granuloma would be:
A. Epitheloid cells C. Lymphocytic infiltrates
B. Fibrosis D. Langhan’s giant cells
7. The hallmark of acute inflammation would be:
A. Tissue damage C. Angiogenesis
B. Vascular leakage D. Hyperemia
8. Which of the following chemical mediators of
inflammation
aid in recognition of the injurious agent?
A. C3a C. Collectins
B. C5a D. Fc fragment of IgM
9. Wound contraction results from?
A. Collagen resorption
B. Fibronectin crosslinking
C. Vascular smooth muscle contraction
D. Myofibroblast action
3. Tissue repair, cell regeneration and 10. The following cells continue to proliferate throughout
life and
fibrosis follow the cell cycles form mitosis to the next
EXCEPT:
A. Hematopoletic tissue
B. Intestinal mucosa
C. Vaginal epithelium
D. Mesencymal cell
11. Which of the following are cardinal features of granulation
tissue?
A. Abundant collagen
PATHO BOARD EXAM QUESTIONS 53
B. Proliferating fibroblasts
C. Proliferating capillaries and fibroblasts
D. Abundant inflammatory cells and collagen
4. Hemodynamic disorders, 12. Turbulence contributes to artenal and cardiac
thrombosis by:
thrombosis, and shock A. dilution by fresh flowing blood by activated clotting
factors
B. enhancing inflow of clotting factor inhibitors
C. causing endothelial injury and dysfunction
D. a & b only

13. On contact with extracellular matrix, platelets undergo the


following changes EXCEPT:
A. Secretion (release reaction)
B. Adhesion and shape change
C. Aggregation
D. Proliferation
14. Which of the following best describes shock?
A. Results from loss blood and plasma volume
B. Caused by intrinsic myocardial damage
C. Systemic hypoperfusion due to reduced cardiac
output or ineffective circulation
D. Results from severe peripheral vasidilation
5. Diseases of immunity 15. Complement-antibody-mediated lysis involves
A. Mast cells C. Leukotrienes
B. Transfusion reactions D. Prostaglandins
16. Antireceptor inhibitory ligand mechanism is associated with:
A. Transplant rejection C. Complement
activation
B. Myasthenia Gravis D. Goodpasture disease
17. Acute graft rejection involves:
A. CD8 interaction
B. Delayed hypersensitivity
C. HLA II incompatibility
D. Arthus reactions
6. Neoplasms 18. Which would be a preneoplastic lesion?
A. Cirrhosis C. pancreatic chorista
B. Hepatoma D. Lymphoma
19. Which would be a highly invasive non-metatasizing
malignant
tumor?
A. Malignant melanoma
B. Squamous cell carcinoma
C. Adenocarcinoma
D. Basal cell carcinoma
20. Which statement is true involving cachexia in many
cancer
patients?
A. In cancer patients, the basal metabolic rate is
decreased
B. Breakdown of skeletal muscle protein by protein
mobilizing factor in some common patients
C. Caloric expenditure in cancer patients is low
D. A, B, & C
7. Genetic and pediatric diseases 21. Which is true of Klinefelter syndrome?
A. There is at least 2 X chromosomes and one or
more Y chromosomes
B. It is a translocation abnormality
C. 45, X Karyotype
D. Phenotypically female patient
PATHO BOARD EXAM QUESTIONS 54
22.Which is true of classic phenylketonuria?
A. Autosomal disorder of galactose metabolism
B. Serum lack of phenylalanine hydroxylase
C. Characterized by very viscerous mucus secretions
D. A & B only
23. Which is true of regulatory distress syndrome in newborn?
A. Results from chromosomal abnormality
B. Presence of generalized edema secondary to
Mother-child blood incompatibility
C. Primary defect is diagnosed by dipalmitozl
phophatidyl choline
D. Associated with decreased dneumucyte I cells in
premature lungs

8. Environmental Diseases 24. Fatty change is found in chronic alcoholism due to:
A. Increased fat delivery to the liver
B. Decreased oxidation of fatty acids
C. Decreased transport of lipoproteins from the liver
D. Decreased serum lipid
25. Amphetamines will:
A. Induced fetal malformation and withdrawal
symoptoms in the neonate
B. Stimulate the CNS
C. Increase the risk of liver cancer
D. A & B only
26. Carbon monoxide in tobacco smoke has which effect?
A. Greater affinity for hemoglobin and cytochrome
oxidase
B. Acute pharmacologic effects associated with tobacco
smoke
C. Responsible for tobacco addiction
D. Induce endocarditis, more on the left side
27. Hypervitaminosis D in adults carry a risk of:
A. Papilledema C. growth retardation
B. renal calculi D. Xeropthalmia
9. General pathology of 28. Viruses will kill host cells and caused tissue damage by:
infectious diseases A. Inhibiting host cell DNA, RNA or protein synthesis
B. Replicating inside host cell followed by cell lypsis
C. Viral proteins on host cell surface lead to immune cells
attack
Of the virus infected cell
D. A, B & C
29. Damage to host cells by bacteria depends on adherence &
entrance
into the cell by presence of:
A. Protein C C. Flagella
B. Pili D. A, B & C
30. Which infective organism produces necrotizing inflammation?
A. H. influenza C. perfringens
B. N. gonnorhae D. V. cholerae

Blueprint for Diseases for Organ System (Patho 2B)


The following topics are derived from the general objectives for Systemic pathology (2B) – 70 questions

TOPICS NO. OF QUESTIONS

1. BLOOD VESSEL DISEASES 4


PATHO BOARD EXAM QUESTIONS 55
1.1 Vascular disorder 1.5 Vasculitides
1.2 Atherosclerosis 1.6 Veins and lymphatics
1.3 Hypertensive vascular disease 1.7 Tumors
1.4 Aneurysm
2. HEART DISEASES 9
2.1 Congestive heart failure 2.5 Primary myocardial diseases
2.2 Ischemic heart disease 2.6 Congenital heart diseases
2.3 Hypertensive HD 2.7 Pericardial heart diseases
2.4 Valvular HD 2.8 Cardiac tumors
3. HEMOPOETIC AND LYMPHOID SYSTEMS 5
3.1 Hemorrhage; blood loss, anemia 3.6 Bleeding disorders
3.2 Hemolytic anemia 3.7 Thrombocytopenia
3.3 Anemia of decreased hemopoiesis 3.8 Coagulation disorders
3.4 Non-neoplastic WBC disorders 3.9 Splenomegaly
3.5 Neoplastic WBC disorders 3.10 Thymic disorders
4. LUNG AND UPPER RESPIRATORY TRACT DISEASES 8
4.1 Atelectasis 4.7 Pleural lesions
4.2 COPD 4.8 Upper respiratory tract lesions
4.3 Restrictive lung diseases 4.81 Infections
4.4 Vascular lung diseases 4.82 Nasopharyngeal cancer
4.5 Pulmonary infections 4.83 Laryngeal cancer
4.6 Lung tumors
5. KIDNEY AND COLLECTING SYSTEM DISORDERS 8
5.1 Clinical manifestations of renal diseases5.6 Urine outflow disorder
5.2 Glomerular diseases & syndromes 5.7 Tumors
5.3 Tubulointerstitial disorders 5.8 Tumors of urinary bladder & collecting
5.4 Blood vessel diseases systems
5.5 Cystic renal diseases
6. LIVER AND BILIARY TRACT DISEASES 8
6.1 Hepatic failure 6.5 Obstructive biliary diseases
6.2 Cirrhosis 6.6 Tumors
6.3 Inflammations – viral hepatitis,
drug-induced
6.4 Inborn errors of metabolism
7. PANCREATIC DISEASES 4
7.1 Exocrine pancreas: pancreatitis, cancer
7.2 Endocrine pancreas: diabetes mellitus and islet cell tumors
8. FEMALE GENITAL SYSTEM 5
8.1 Vulva - vulvitis and tumors
8.2 Vagina – inflammation
8.3 Cervix – tumor
8.4 Uterus – endometriosis, adenomyosis, endocervicitis, DUB, tumors
8.5 Ovaries – cysts, tumors ( surface, epithelial, stomal)
8.6 Diseases of pregnancy - ectopic pregnancy, gestational trophoblastic diseases
9. BREAST DISEASES ( Given in Surgical Patho) 3
9.1 Fibrocystic diseases
9.2 Inflammation
9.3 Tumors
9.4 Male breast diseases

10. GASTRO-INTESTINAL DISEASES 5


10.1 Esophagus – esophagitis, Barett’s esophagus, cancer
10.2 Stomach – gastric ulcer, tumors
10.3 Small and large intestines – developmental disorders, vascular, diarrheal, idiopathic inflammatory
bowel disease, bowel obstruction, tumors
10.4 Appendix – acute appendicitis, tumors
11. ENDOCRINE DISEASES 3
11.1 Pituitary – hypo and hyperpituitary syndromes
11.2 Thyroid – hypo and hyperthyroidism, neoplasms
11.3 Parathyroid – hypo and hyperparathyroidism
PATHO BOARD EXAM QUESTIONS 56
11.4 Adrenal cortex – hypo and hyperadrenalism, tumors
12. MUSCULOSKELETAL DISEASES 3
12.1 Bone diseases – congenital and hereditary, osteoporosis, osteomyelitis,
Pagets disease, tumors
12.2 Joint diseases – osteoarthritis, gout, infectious arthritis
12.3 Skeletal muscles – atrophy, myasthenia gravis, inflammatory myopathies,
muscular dystrophies
13. CENTRAL NERVOUS SYSTEM DISEASES 5
13.1 Cells of the CNS
13.2 Edema, hematoma, hydrocephalus
13.3 Vascular diseases
13.4 CNS trauma
13.5 Congenital malformations
13.6 Infections
13.7 Neoplasms
13.8 Primary myelin diseases
13.9 Degenerative diseases
13.10 Disease of peripheral nervous system

_____________________
Total : 70 questions

Total: 30 questions – general pathology


70 questions – organ system pathology
-------------
100 questions

Note: The questions were contributed by lecturers of the various topics.

Collated by:

Remedios A. Magkasi, MD,


MHED
Chairperson
Department of Pathology and
Laboratory Diagnosis

Questions on General pathology (2B)


MULTIPLE CHOICE: CHOOSE THE BEST ANSWER

TOPICS QUESTIONS

1. GASTRO-INTESTINAL DISEASES 1. Most common neoplastic lesion of the colon would be?
A. Hyperplastic polyp C. Lipoma
B. Adenomatous polyp D. Carcinoid
2. Which of the following can cause toxic megacolon?
A. Hirschsprungs dis. C. Ulcerative
colitis
B. Chagas dis. D. cancer of the
rectum
3. Most common location of carcinoid tumor is?
A. Appendix C. Ileum
PATHO BOARD EXAM QUESTIONS 57
B. Cecum D. Sigmoid
4. Persistence of vitelline duct causes which anomaly?
A. Congenital aganglionic megacolon
B. Imperforate anus
C. Meckel’s diverticulum
D. Intussusception
5. The most common type of epithelial polyp is?
A. Villous C. Tubular
B. Tubulovillous D. Pseudopolyp
2. MUSCULO-SKELETAL DISEASES 6. Primary Osteosarcoma is characretized by which
feature(s)?
A. Most common type
B. RB gene mutation
C. Usually arise in the metaphysis of the long bones
D. A,B,C
7. The pathogenesis of Osteogenesis Imperfecta is:
A. Mutation resulting to inadequate production of
collagen
B. Clustering of chondrocytes in zone 2
C. Both A & B
D. Neither
8. Morphologic findings of mosaic pattern of lamellar bone is
associated
with which of the following?
A. Osteitis deformans C. Fibrous dysplasia
B. Osteopetrosis D. Achondroplasia
3. LIVER DISEASES 9. Acute hepatitis B infection is associated with
A. Sanded nuclei
B. PAS (+), Diastase (-) intracytoplasmic globules
C. (+) HA IgM
D. Councilman bodies
10. Extrahepatic cholestasis
A. is sometimes termed as “medical jaundice”.
B. is caused by Criggler Najar type II.
C. is directly associated with portal HPN.
D. Shows bile lakes in section
11. Hepatocellular intrahepatic cholestasis is associated with
A. a predominance of serum ALP
B. a decreased serum ALP
C. obstruction of the common bile duct
D. Bile plugs, bile stained hepatocytes, & increased
of bile ducts

3. (CONT. LIVER DISEASES) 12. Criggler Najar type I:


A. causes jaundice due to decrease bilirubin
conjugation.
B. is a hemolytic cause of kernicterus.
C. is caused by extrahepatic biliaryobstruction.
D. leads to increased serum bile acids
13. Autoimmune hemolytic anemia:
A. leads to bilirubinuria.
B. causes jaundice with a predominant indirect
hyperbilirubinemia.
C. is associated with decreased urobilin in the urine.
D. is associated with Gilbert’s syndrome.
14. Window period of Acute Hepatitis B infection would present:
PATHO BOARD EXAM QUESTIONS 58
A. ballooning degeneration, lobular disarray, councilman
bodies. B. elevated transaminase levels.
C. (+) HbsAg.
D. A and B only.
15. Portal vein occlusion is associated with:
A. Pyelephlebitis
B. Caused by abnormal sepsis
C. Budd Chlari syndrome
D. A and B
16. Hemolytic jaundice
A. predominantly elevated polar bilirubin.
B. primarily manifested as bilirubinuria.
C. is due to biliary obstruction.
D. is due to increased bilirubin production.
4. ENDOCRINE DISEASES 17. The following are true of hypothyroidism EXCEPT:
A. most common causes is Graves disease.
B. increased T3 and T4 levels in the serum.
C. increased TSH level in the serum.
D. None of the above
18. The most common cause of hypoparathyroidism is:
A. latrogenic (postathyroidectomy)
B. congenital.
C. radiation, ionizing.
D. Hyperthyroidism.
19. The following is true of hyperaldosteremia EXCEPT
A. increased plasma renin.
B. overproduction of aldosterone with suppression of
renin-
angiotensin system.
C. associated with decreased renal perfussion.
D. pregnancy.
5. RENAL DISEASES 20. Nephritic syndrome is associated with:
A. lipoid nephrosis. C. both
B. Goodpasture disease. D. neither
21. Renal cell carcinoma is associated with which characteristic(s)?
A. invades renal vein C. both
B. clear cell type D. neither
22. Acute post-strepto. GN. Is characterized by:
A. tubulorrhexis C. both
B. thick basement membrane D. neither
23. Nephrotic syndrome is found in which disease(s)?
A. diabetes mellitus C. both
B. membranous GN D. neither
24. Chronic PN has gross change(s) as
A. coarse irregular scars C. fine surface nodularity
B. necrotic renal papilla D. A & B only

5. (CONT. RENAL DISEASES) 25. The most common cause(s) of acute pyelonephritis is (are)
A. Endamoeba coli C. both
B. Staphylococcus aureus D. neither
26. Linear fluorescence on immunofluorescent stain is
characterizing in:
A. Goodpasture’s disease C. Membranous GN
B. Acute proliferative GN D. Lipoid nephrosis
27. Wilm’s tumor is associated with
A. originated from prox. Tubules
B. abortive glomerulus and tubules
PATHO BOARD EXAM QUESTIONS 59
C. clear and solid cells
D. paraneoplastic syndrome
CASE:
A 25 year old female consulted a physician for gradual
enlargement of the abdomen and both legs.
The condition started about 3 weeks ago as dyspnea with
ankle swelling, followed by abdominal heaviness. She later noticed
abdominal enlargement, which shifted when she lies in bed on her
side. Past history showed that she had RHD and was hospitalized
twice in the last 4 years but was sent home improved.
6. HEART DISEASES 28. The significance of the abdominal enlargement shifting
when she lies
on her side would be:
A. ascites
B. hepatomegaly
C. splenomegaly
D. A,B,C
29. Correlating the history with the complaints, what would be the
significance of leg edema?
A. pure right sided heart failure
B. pure left sided heart failure
C. left sided failure progressive to right sided failure
D. pneumonia
30. If you were to examine the patient’s lungs, what finding(s) do
you
expect to discover to explain dyspnea?
A. lifting up of the diaphragm
B. pulmonary edema
C. pleural effusion
D. lung abscess
31. If you could visualize the heart, what change(s) would be
expected
to give rise to the complication shown?
A. fusion of the tricuspid commissures
B. thick and short mitral chord tendinae
C. verrccae on the valve leaflets
D. A,B, C
------------ end of case--------------
32. Complication(s) of left ventricular MI would be:
A. Left to right shunt with ventricular septal rupture
B. Embolization of thrombus of pulmonary artery
C. Arrythimia
D. A and C only
33. Small, warty friable vegetations underneath the mitral valve is
seen
in:
A. RHD
B. acute infective endocarditis
C. endocarditis
D. libman sacks
E. carcinoid HD

6. (CONT. HEART DISEASES) 34. Diastolic murmur is heard in:


A. mitral insufficiency
B. aortic insufficiency
C. mitral valve prolapse
D. Roger’s disease
PATHO BOARD EXAM QUESTIONS 60
35. Mitral midsystolic click is associated with:
A. myxomatous degeneration of aortic valve
B. fusion of chorda tendinae
C. syphylitic heart disease
D. None of the above
36. Stable angina chest pain is explained by:
A. the atherosclerotic coronaries cannot cope with
increased demand
B. chest pain relieved by rest
C. transmural ischemia
D. pericardial friction rub
7. FEMALE GENITAL SYSTEM 37. Characteristic histopathologic change associated with
HPV uterine
cervix infection is:
A. thickening of stratum corneum with production of
qualitatively abnormal keratin.
B. retention of nuclei in the cells of the stratum
corneum.
C. kollocytic atypia.
D. epidermal thickening.
38. The most common gene mutated in patients with ovarian
carcinoma
is: A. Tp53 C. ERBB
B. BRCA D. K-RAS
39. An obese female sought consultation because of
oligomenorrhea.
Physical examination showed increased body hair and
masculine features. Ultrasound showed the presence of
numerous cysts in the ovaries. True of this conditionwould be: it
is
A. is associated with anovulation
B. an estrogen-responsive
C. both
D. neither
40. The genetic mutation implicated in cases of lelomyoma involve
is:
A. over-expression of KRAS.
B. mutation of PTEN gene.
C. t (12:14) rearrangement.
D. microsatellite instability.
41. A 40 y/o G3P3 came to your office complaining of increased
amount
of “bleeding” during her menstrual period. Her previous
mestruation
occurred on a 30-day cycle for which she consumed 3
napkins/day.
However, for the past few months, she noted her menses to
occur
every 3 weeks for which she consumes 6 napkins/day. Probable
cause of the abnormal bleeding could include:
A. an organic lesion in the uterus
B. hormonal imbalance
C. failure of ovulation
D. all of the above
8. BLOOD VESSELS 42. The most important feared complication of atheroma is:
A. calcification C. superimposed
thrombosis
B. rupture D. ulceration

PATHO BOARD EXAM QUESTIONS 61


4

8. (CONT. BLOOD VESSEL) 43. The lesion in Monckeberg sclerosis:


A. causes narrowing of lumen
B. destroys tunica intima
C. has unclear etiology
D. is confined within the tunica adventitia
44. The clincial significance of arteriovenous fistula is:
A. aneurysmal dilatation.
B. chronic hemodialysis.
C. malignant transformation.
D. mycotic aneurysm.
45. This chemical substance is implicated in the development of
hepatic
hemangiosarcoma:
A. aflatoxin C. CCI4
B. barium D. polyvinyl chloride
9. CNS DISEASES 46. A 60 years old nursing home resident complained of
severe headache.
She has been irritable for the past 2 days and later during the
day
developed clouding of consciousness. P. E showed rigidity
and a
temperature of 39 degrees Celsius. Spinal tap yielded a cloudy
CSF
with neutrophile, elevated protein, and markedly reduced
glucose
content. The patient is suffering from:
A. acute viral meningitis
B. epidural hemorhage
C. acute bacterial meningitis
D. chronic bacterial meningoencephalitis
47. The most likely etiologic agent is:
A. Mycobacterium TB C. E. coli
B. Strep. pneumonia D. enterovirus
---------------- End of Case----------------
48. This glial cell responsible for repair and scar formation of CNS
tissue is:
A. Astrocyte C. Oligodendrocyte
B. Microglia D. Ependymal cell
49. This congenital malformation of the neural tube is not compatible
with life
A. Meningocoele C. Encephalocoele
B. Anencephaly D. Meningmyelocoele
50. This CNS tumor is characterized by laminated psammoma bodies:
A. Meningcoma C. Ependymoma
B. Astrocymoma D. Schwanomma
10. LUNG AND URT DISEASES 51. Associated with alpha/antitrypsin deficiency is:
A. Centro-acinar emphysema
B. Paraseptal emphysema
C. Panlobular emphysema
D. Senile emphysema
52. Factors for the development of chronic bronchitis include the
following,
EXCEPT:
A. Microbiologic infection
PATHO BOARD EXAM QUESTIONS 62
B. Chronic irritation by inhaled substance
C. Cigarette smoke
D. Surfactant deficiency
53. The lung tumor most sensitive to chemotherapy and radiation
therapy is:
A. Small cell cancer C. Large cell CA
B. Squamous cell Ca D. Adenocarcinoma
54. Primary mediators of asthma are the following, EXCEPT:
A. Histamine C. Prostaglandines
B. Leucotrienes D. Acetylcholine

10. (CONT. LUNG AND URT DISEASES)55.Viral pneumonias differ histologically from bacterial
pneumonia
because of:
A. presence of alveolar exudates
B. lack of alveolar exudate
C. low levels of cold agglutinin
D. high HCT. level
56. Macrophages are alternative sources of injury in patients with
ARDS by: A. generating toxic oxygen products and
proteases
B. extra pulmonary activation of complement via
alternative pathway
C. mediator release leading to vasoconstriction
D. A and B
57. Which of the following is/are seen in patients with severe
emphysema?
A. blood gas value is relatively normal
B. alveolar overdistention is severe
C. diffusing capacity is low
D. A,B&C
58. Histologic findings of asthma include the following, EXCEPT:
A. thickening of basement membrane of the bronchial
epithelium
B. edema and inflammatory infiltrate in the bronchial
wall
C. hyperplasia of submucosal glands
D. hypertrophy of bronchial wall muscle
11. HEMOPHATIC AND LYMPH 59. Clinical feature(s) of Acute Luekemia includes;
NODE DISEASES A. Abrupt onset
B. Depression of normal marrow function
C. Bone pain & tenderness
D. A,B, and C
60. Acute Lymphoblastic Leukemia (ALL) characteristics are the
following
EXCEPT:
A. common in children and young adults
B. peak incidence at approx. 4 years of age
C. Leukemic blasts lack peroxidase
D. Leukemic blasts lack PAS (+) material
61. Characteristics of Chronic Myeloid Leukemia EXCEPT;
A. Presence of PH chromosome as diagnostic feature
B. Lack of alkaline phosphatase in granulocytes
C. Rapid progression of disease

PATHO BOARD EXAM QUESTIONS 63


D. Bone marrow transplantation as curative treatment
62. Lymph node follicular hyperplasia differs from follicular (nodular)
lymphomas by the following feature EXCEPT:
A. Preservation of the nodal architecture
B. Marked alteration in shape and size of lymphoid
cells
C. Mixed population of lymphocytes in different
stages of differentiation
D. Prominent phagocytotic activity in germinal
centers
63. Characteristic morphologic feature distinguishing Non-Hodgkin’s
from
Hodgkins Lymphoma:
A. Cleaved nuclear contour of lymphocytes
B. Plasmacytoid features of tumor cells
C. Prominence of Reed-Sternberg cells
D. “Starry sky” pattern

12. PANCREATIC DISEASES 64. The following statements are all true of carcinoma of the
pancreas
EXCEPT:
A. 60 to 70% of pancreatic cancer arise in the tail of
the pancreas.
B. Almost all of pancreatic cancers are
adenocarcinomas.
C. Carcinomas of the body and tail of the pancreas
remain silent for some time.
D. Histologically, there is no difference between
pancreatitic carcinomaof the head and those of the
body and tail.
65. Which is not a basic alteration in acute pancreatitis?
A. Hemmorhages and blood vessel necrosis
B. Necrosis of muscle by lipolytic enzymes.
C. Acute inflammatory reaction
D. Proteolytic destruction of pancreatic substance
66. Which would cause death of a patient with severe acute
pancreatitis?
A. Shock
B. Secondary abdominal sepsis
C. Acute respiratory distress syndrome
D. A,B,C
67. Important mechanism(s) linking hyperglycemia in long-standing
DM
to complications would be:
A. trapping of circulation low-density lypoprotein,
providing
cholesterol deposition and accelerate atherogenesis.
B. in DM, AGEs bind to receptors of various cells and
lead
to increased endothelial permeability.

PATHO BOARD EXAM QUESTIONS 64


C. Sorbitol accumulation promotes injury of pericytes
of
retinal cappilaries, resulting to microaneurysms
D. A,B.C
13. (BREAST DISEASES) 68. Which breast cancer shows infrequent lymph node
metastasis?
A. Infiltrating duct ca C. Infiltrating lobular ca
B. Intraductal carcinoma D. Colloid carcinoma
69. Large fleshy breast tumors with characteristic lymphoid infiltrates
histologically is:
A. Infiltrating duct carcinoma C. Colloid
carcinoma
B. Medullary carcinoma D. Infiltrating lobular
carcinoma
70. The usual presentation of bloody nipple discharge is found in:
A. Intraductal papillome C. Fibrocystic change
B. Fibroadenoma D. Phylloides tumor

End of Organ System Pathology

Total : 30 questions – general pathology


70 questions – organ system pathology
---------------
100 questions
Reference textbook:
Robbins Basic Pathology
by Kumar, Cotran, and Robbins :
7th Edition (2003) Collated by: Remedios A. Magkasi, MD
FPSP, MHPed.
Chairperson
Department of pathology and Laboratory
Diagnosis
February 4, 2005

DAVAO MEDICAL SCHOOL FOUNDATION


College of Medicine

GENERAL PATHOLOGY

Chapter 1 CELLULAR ADAPTATIONS, CELL INJURY, AND CELL DEATH

OBJECTIVES:

1. Define and explain atrophy, hypertrophy and hyperplasia and give appropriate examples of each.
2. Define and explain metaplasia and dysplasia and give appropriate examples of each.
3. Explain how the functions of cells are coordinated and integrated and define homeostasis and steady state.
4. Define reversible and irreversible cell injury and describe the flux of water and minerals across the plasma membrane of
injured cells.
5. Explain the cytoplasmic changes in reversible cell injury.
6. Define and describe the nuclear changes in irreversible cell injury.
7. Define apoptosis and give examples of its significance.
8. List the most important causes of cell injury, and explain the role of oxygen radicals in cell injury.
9. Describe outcomes of cell death with special emphasis on calcification.
10. Describe and give clinical examples of extracellular and intracellular accumulations in cell injury.
11. Discuss the current theories of aging with special emphasis on genetic aspects of aging.

QUESTIONS:

________ 1. Acetaminophen may be toxic under certain conditions, for example if taken in large amounts together with alcohol.
It has been shown that the toxicity of acetaminophen is mediated by oxygen radicals, which are produced in large amounts and
due to the depletion of an important scavenger molecule that normally inactivates hydrogen peroxide. Name that cytosolic
PATHO BOARD EXAM QUESTIONS 65
scavenger of oxygen radicals.

A. Catalase
B. Iron
C. Oxidase
* D. Glutathione

________ 2. A patient with autoimmune hemolytic anemia was transfused with 24 blood units over a period of 3 years. He died
of pneumonia. At the autopsy the liver was dark brown and contained increased amounts of Prussian blue positive pigment.
Name this pigment.

* A. Hemosiderin
B. Carbon
C. Lipofuscin
D. Melanin

________ 3. A coal-miner suffered from chronic bronchitis and emphysema. At autopsy the lungs appeared black. The
mediastinal lymph nodes were also black. Although you have no histologic slide, you can still make unequivocally the diagnosis
of

A. Alkaptunuria
B. Ochronosis
C. Hemosiderosis
* D. Anthracosis

________ 4. At autopsy of a 60 year old man who died of myocardial infarction, histologically the heart demonstrated: loss of
nuclei, loss of cell outlines, deeply eosinophilic staining of the cytoplasm and a loss of cross striation of cardiac myocytes. This
heart shows signs of

* A. Coagulative necrosis
B. Fat necrosis
C. Caseous necrosis
D. Liquefactive necrosis

________ 5. A mediastinal lymph node of a patient who died of miliary tuberculosis was examined histologically. There were
numerous granulomas, the central areas of which were composed of amorphous granular material. On gross examination these
lymph nodes appear yellow-white, and are a typical example of

A. Coagulative necrosis
B. Fat necrosis
* C. Caseous necrosis
D. Liquefactive necrosis

________ 6. Following a fracture the leg was placed into a cast. Six weeks later the skeletal muscle cells appear angulated in
histologic sections. These cells have reduced energy requirements. The muscles of the extremity are weak. Name the form of
adaptation that occurred in this muscle.

A. Hypertrophy
B. Hyperplasia
* C. Atrophy
D. Metaplasia

________ 7. A chronic smoker presented to the hospital with relentless coughing. A bronchial biopsy was performed. It
contained fragments of mature squamous epithelium with no evidence of nuclear atypia. This pathologic change is called

A. Hypertrophy
B. Hyperplasia
C. Atrophy
* D. Metaplasia

________ 8. Deposition of calcium salts in lymph nodes affected by tuberculosis is an example of a pathologic process called

A. Peroxidation
B. Lipolysis
C. Metastatic calcification
* D. Dystrophic calcification

________ 9. Multiple opacities in the cornea of a child given large amounts of vitamin D were found to contain deposits of
calcium phosphate. This pathologic process is an example of

A. Peroxidation
B. Lipolysis
PATHO BOARD EXAM QUESTIONS 66
* C. Metastatic calcification
D. Dystrophic calcification

________10. A thrombus of the right coronary artery was lysed by infusion of plasminogen activator into the occluded coronary
artery. A reperfusion injury was induced. Activated oxygen species that induced the injury was in part derived from the
myocardial cells and in part from another source. Name that other source of oxygen radicals in this situation.

* A. Leukocytes
B. Thrombus
C. Plasminogen
D. Mitochondria

________11. Hydropic swelling of the cell is characterized by each of the following EXCEPT

A. increased number of cytoplasmic organelles


B. dilatation of cisternae of endoplasmic reticulum
C. impairment of cellular volume regulation
* D. influx of sodium into the cell

________12. Hypertrophic heart muscle cells contain increased amounts of

A. water in the sarcoplasmic reticulum


B. smooth endoplasmic reticulum
C. rough endoplasmic reticulum
* D. messenger RNA

________13. Erythroid hyperplasia of the bone marrow is often found in people who live

A. at sea level
* B. at extremely high altitude (10,000 feet)
C. in the equatorial jungles of Brasil
D. above the polar circle in Canada

Chapter 2 ACUTE AND CHRONIC INFLAMMATION

OBJECTIVES:

At the completion of this unit, the student should be able to ...

1. Know the clinical signs of inflammation, the cause(s) of each, and appreciate their importance
to you in a diagnostic sense.
2. Differentiate exudate from transudate.
3. Know the vascular and cellular changes happening in acute inflammation.
4. Know the various chemical mediators involve in acute inflammation.
5. Understand that the inflammatory process can lead to several outcomes, and that it will tend to persist
until the inciting agent is eliminated.
6. Know the different morphologic patterns of acute inflammation.
7. Know the causes, morphologic features, and inflammatory cells involved in chronic inflammation.
8. Define granulomatous inflammation and list down its morphologic characteristics.
9. Know the clinical and pathologic changes that make up the acute phase response.

QUESTIONS:

________ 1. Formylated peptides derived from bacteria, leukotrienes derived from plasma membranes and complement
fragments have in common that they all mediate a process important for formation of an exudate. This process is called

* A. Chemotaxis
B. Phagocytosis
C. Extravasation
D. Margination

________ 2. Which biogenic amine released from mast cells accounts for the increased vascular permeability at the site of
inflammation?

A. Bradykinin
B. Interferon
C. Tumor necrosis factor -alpha
* D. Histamine

________ 3. The arachidonic acid derivations produced through the lipoxygenase pathway in neutrophils, mast cells and
macrophages in bronchial asthma, and known as slow reacting substances of anaphylaxis belong to the group of chemical
substances known collectively as
PATHO BOARD EXAM QUESTIONS 67
A. Prostaglandin
* B. Leukotrienes
C. Lipoxin
D. Histamine

________ 4. Hageman factor activates kallikrein, which in turn produces a polypeptide that is an important plasma derived
mediator of increased vascular permeability. Name this substance.

* A. Bradykinin
B. Interferon
C. Tumor necrosis factor -alpha
D. Histamine

________ 5. Name the group of plasma proteins that upon activation form intermediate complexes and fragments which act as
vasoactive mediators of inflammation, prime phagocytic cells for a more active response to bacteria, and act as chemotactic
factors.

A. Hageman factor
B. Kinins
* C. Complement
D. Kallikreins

________ 6. Which plasma protein if activated by a variety of stimuli can initiate the clotting cascade, the fibrinolytic pathway
and kinin production?

* A. Hageman factor
B. Kinins
C. Complement
D. Kallikreins

________ 7. The process in which fragments of complement or IgG bind to bacteria facilitating phagocytosis is called

* A. Opsonization
B. Margination
C. Extravasation
D. Adhesion

________ 8. The adhesion of leukocytes to the endothelial cells in the early stages of inflammation is preceded by changes in
the blood flow, most notably apportioning of leukocytes to the peripheral parts of the blood stream. This process is called
A. Opsonization
* B. Margination
C. Extravasation
D. Adhesion
________ 9. Which process representing an essential aspect of inflammation is mediated by cell surface glycoproteins known as
LFA, ICAM-1, GMP-140? _______________

A. Opsonization
B. Margination
C. Extravasation
* D. Adhesion

________10. Name the mobile cell in tissue that actively presents the antigen to lymphocytes and by secreting interleukins,
regulates lymphocyte function.

A. Neutrophil
B. Natural killer cell
C. Langerhans cell
* D. Macrophage

Chapter 3 TISSUE RENEWAL AND REPAIR: REGENERATION, HEALING, AND


FIBROSIS

OBJECTIVES:

At the completion of this unit, the student should be able to ...

1. Discuss the phases of the cell cycle and their relation to the proliferative potential of the cells of the body.
to you in a diagnostic sense.
2. Have a basic knowledge about stem cells and its potentials in the process of tissue regeneration.
PATHO BOARD EXAM QUESTIONS 68
3. Know the several growth factors important to tissue regeneration and repair.
4. Know the general modes of signaling.
5. Know the major types of receptors and how they deliver signals to the cell interior.
6. List the 5 major components of the extracellular matrix. Describe the function(s) of the matrix .
7. Have a basic knowledge of the healing processes like angiogenesis, synthesis of ECM proteins and tissue
remodeling
8. Describe the process of wound healing in skin
9. Discuss 3 complications of wound healing and their probable etiologies.
10. Know the clinical and pathologic changes that make up the acute phase response.

QUESTIONS:

________ 1. Name the glycoprotein that is deposited early in wound healing and is known for its binding sites for fibrin,
fibrinogen, collagen, and other components of the extracellular matrix.

* A. Fibronectin
B. Laminin
C. Fibrillin
D. Collagen

________ 2. ) Name the tissue filling the bottom of a non-healing wound that consists histologically of capillaries, fibroblasts,
collagen, and a variable number of inflammatory cells.

A. Collagen
B. Granuloma
* C. Granulation Tissue
D. Fibrin

________ 3. Name the cell that accounts for the primary contraction of the wound. This cell has features of both a fibroblast and
a smooth muscle cell.

A. Collagen
* B. Myofibroblast
C. Fibroblast
D. Myoblast

________ 4. An exuberant (hypertrophic) scar developed after plastic surgery in a 20 year old Afro-American woman. This
complication of wound healing is called

A. Hypertrophy
* B. Scar
C. Granulation tissue
D. Keloid

________ 5. Name the family of transmembrane proteins that interact with the cytoskeleton and the extracellular matrix.

A. Fibrillin
B. Collagen
C. Fibrin
* D. Integrin

________ 6. Renal proximal tubule cells are capable of repair after tubular necrosis unless there is destruction of the

A. Extracellular matrix
B. Basement membrane
C. Upper layer of the epithelium
* D. A & B are correct

________ 7. What is the name of the process characterized by a growth of newly formed blood vessels, stimulated by growth
factors? _______________(Angiogenesis (Granulation Tissue).)

A. Fibrosis
B. Extravasation
* C. Angiogenesis
D. Granulation

________ 8. Bursting of a wound is called _______________(Dehiscence.)

A. Contracture
* B. Dehiscence
C. Keloid
D. 2nd intenetion

________ 9. Name a growth factor derived from platelets that are capable of inducing new growth of mesoderm-derived
components (fibroblasts, etc.) in wounds. _______________(Platelet-Derived Growth Factor.)

PATHO BOARD EXAM QUESTIONS 69


* A. Platelet derived growth factor
B. Platelet factor
C. Interferon
D. Platelet growth factor

________10. Each of the following are examples of atrophy EXCEPT

A skeletal muscle following transection of its nerve


B skeletal muscles following long term immobilization
C ovary following hypophysectomy
D endometrium following administration of estrogen
E brain of an 100 year old man

Chapter 5 GENETICS

OBJECTIVES:

At the completion of this unit, the student should be able to ...

1. Able to define and discuss the 3 categories of mutation with special emphasis on the gene mutation.
2. Able to know and briefly discuss the transmission patterns of single gene disorders.
3. Able to briefly explain the biochemical and molecular basis of single-gene (Mendelian Disorders).
4. Able to know morphologically, clinically and pathogenesis of some selected disorders associated with defects in structural
proteins, receptor proteins, enzymes, regulatory proteins.
5. Able to know the terminology system in karyotyping.
6. Able to briefly discuss the most common forms of alterations in chromosome structure and the notations used to
signify them.
7. Able to know morphologically, clinically and pathogenesis of some selected cytogenetic disorders involving autosomes and
sex chromosomes.
8. Able to know morphologically, clinically and pathogenesis of some selected single gene disorders with non-classic
inheritance.

QUESTIONS:

________ 1. Worried parents come to your office concerned about an increased risk of transmitting genetic disease to their
offspring. You reassure them that there is no increased risk of having a second, afflicted child even if their first child is born
with:
A. Cleft lip
B. Hydrops fetalis
C. Albinism
* D. Torch embryopathy

________ 2. Neonatal lymphedema, short stature, infertility, normal intelligence, and coarctation of the aorta are
characteristics typical of which genetic abnormality?
A. Trisomy 21
* B. Turner syndrome
C. Any autosomal aneuploidy
D. Cri du Chat syndrome
E. Fragile X syndrome

________ 3. Epicanthal fold, Brushfield spots, dysplastic ears, megacolon, congenital heart disease, and a simian crease are all
clinical features of a numerical abnormality of chromosome number:
A. 11
B. 17
C. 18
* D. 21

________ 4. Cardiovascular disorders, particularly dissecting aortic aneurysm, are the most common cause of death in which
autosomal dominant disorder?
A. Neurofibromatosis
B. Familial hypercholesterolemia
C. Erhler-Danlos syndrome
* D. Marfan syndrome

________ 5. Inherited as an autosomal dominant trait:


* A. Neurofibromatosis type I
B. Cystic fibrosis
C. Alcaptonuria
D. Neimann-Pick disease
E. Gaucher disease
PATHO BOARD EXAM QUESTIONS 70
________ 6. The inheritance pattern of cleft lip and cleft palate is:
A. Autosomal dominant
B. Autosomal recessive
C. X-linked dominant
* D. Multifactorial

________ 7. An exchange of fragments of chromatids between non-homologous chromosomes may occur during the first
meiotic division. This chromosomal structural abnormality is called:
A. Deletion
B. Inversion
C. Nondisjunction
D. Segregation
* E. Translocation

Chapter 6 DISEASES OF IMMUNITY

OBJECTIVES:

At the completion of this unit, the student should be able to ...

1. Define and distinguish between natural immunity and acquired immunity


2. List the main organs and cells that participate in the immune response.
3. Understand the major histocompatibility complex and its relationship to transplantation and disease.
4. Discuss the mechanisms of the four hypersensitivity reactions.
5. Describe the main forms of transplants: homograft, isograft, autograft, and xenograft.
6. Able to discuss the mechanisms and morphology of transplant rejection.
7. Able to know the methods on increasing graft survival.
8. Able to know the concept of auto tolerance and how they can be bypassed.
9. Able to know the etiology, pathogenesis , morphology and clinical presentation of selected autoimmune disorders
and immunologic deficiency syndromes.
10. Able to know the etiology, pathogenesis, morphology and clinicla presentation of amyloidosis.

QUESTIONS:

________ 1. A 30 year old woman with leukemia was irradiated and transfused with bone marrow cells of a related donor. She
developed skin rash, diarrhea and jaundice. What is the name of this disease?

A. Hypersensitivity reaction
* B. Graft vs. host reaction
C. Immune deficiency reaction
D. AIDS

________ 2. A complex of genes on chromosome 6, the products of which are important in antigen presentation, and for
mounting an immune reaction to transplanted foreign tissues.

* A. Major histocompatibility complex


B. T-cell receptor
C. CD3 proteins
D. Human leukocyte antibody

________ 3. A skin biopsy in a 30 year old woman with a skin rash showed deposits of IgG and complement along the
basement membrane in a granular pattern. The patient had positive ANA antinuclear anti ds-DNA antibody, arthritis and
glomerular inflammation. Name this disease. _______________

A. Amyloidosis
B. AIDS
* C. Sytemic lupus erythematosus
D. Scleroderma

________ 4. A 40 year old female presents with “tight”, atrophic skin, dyspnea (shortness of breath), esophageal dysphagia,
Raynaud’s phenomenon and kidney disease. Skin biopsy shows markedly condensed connective tissue. What autoimmune
disease could account for this constellation of symptoms?

A. Amyloidosis
B. AIDS
C. Sytemic lupus erythematosus
* D. Scleroderma

________ 5. A 40 year old man presents with hemoptysis, renal failure. He has antibodies to collagen type IV that react with
glomerular basement membranes in a linear manner. Name this disease. _______________

A. Systemic lupus erythematosus


PATHO BOARD EXAM QUESTIONS 71
B. Graft vs. host reaction
C. Serum sickness
* D. Goodpasture syndrome

________ 6. Name the mobile cell in tissue that actively presents the antigen to lymphocytes and by secreting interleukins,
regulates lymphocyte function.

A. Lymphocyte
B. Plasma cell
* C. Macrophage
D. Neutrophils

________ 7. Each of the following represents a type II hypersensitivity (cytotoxic) reaction EXCEPT
A. Coomb's positive hemolytic anemia
B. myasthenia gravis
C. pemphigus vulgaris
* D. bronchial asthma

________ 8. A 28 year old female presents herself to your clinic complaining of a rash that worsens upon exposure to sunlight.
She reports fatigue x 1 month and swollen, painful joints. Serology for anti-nuclear antibody and anti-double-stranded DNA are
positive, in high titers. What is the best diagnosis?
A. dermatomyositis
B. graft vs. host reaction
C. rheumatoid arthritis
* D. type III hypersensitivity

________ 9. Severe myalgia (muscle pain), elevated CPK with MM isotype, and a rash involving the eyelids is characteristic of
this disease.
* A. dermatomyositis
B. Goodpasture syndrome
C. graft vs. host reaction
D. mixed connective tissue disease

________10. A 40 year old female patient experiences a difficulty swallowing (due to esophageal fibrosis upon biopsy),
myositis, and later in her course even develops arthritis. Anti-double-stranded DNA serology is negative, and serologic markers
for dermatomyositis are not pathognomonic. A short course of corticosteroids seems to improve her condition somewhat. What
is the best diagnosis?
A. dermatomyositis
B. Goodpasture syndrome
* C. mixed connective tissue disease
D. graft vs. host reaction

Chapter 7 NEOPLASIA

OBJECTIVES:

At the completion of this unit, the student should be able to ...

1. Define neoplasia and its related terms: tumor, cancer, and oncology.
2. Differentiate the two basic components of tumor.
3. Understand the nomenclature of tumors.
4. Differentiate benign from malignant tumors based on differentiation, rate of growth, local invasion and
metastasis.
5. Discuss the basic principles of the molecular basis of cancer.
6. List down and briefly describe the seven fundamental changes in cell physiology that determine malignant
phenotype.
7. Discuss the normal cell cycle and how it progress from one phase to another.
8. Define oncogene, list their main groups and give examples of each, and discuss the mechanism of how
they promote cancer.
9. Discuss the importance of ras oncogenes.
10. Give a brief discussion on cancer suppressor genes.
11. Discuss the concepts of multistep carcinogenesis, tumor progression and heterogeneity.
12. Give a brief description on the steps involved in chemical carcinogenesis.
13. Give a brief description on the steps involved in radiation carcinogenesis.
14. Give a brief description on the steps involved in microbial carcinogenesis as exemplied by several viruses.
16. Able to discuss the possible ways and means a body can defend itself against tumors.
17. Able to know the effects of tumor to the body.
18. Able to know the difference between grading and staging of cancer.
PATHO BOARD EXAM QUESTIONS 72
19. Able to list down the laboratory aids in diagnosing cancer.

QUESTIONS:

________ 1. Each of the following statements concerning multi-stage carcinogenesis is CORRECT, EXCEPT
A. DNA damage is involved during initiation
B. requires a long time for tumors to evolve
C. occurs in animal models, but not in humans
D. involves multiple genetic events
* E. implicated in tumors of epithelial origin

________ 2. The following are common characteristics of metastatic cells EXCEPT


* A. often resemble primary tumor
B. spread by lymphatic vessels
C. vascular spread
D. invade basement membranes
E. incapable of degrading extracellular matrix

________ 3. Select the gene most likely associated with the condition. A 50 year old woman presents to her physician with
fatigue, fever and splenomegaly. Blood work-up indicates granulocytic leukocytosis in peripheral blood (20,000/ul). Her
physician diagnoses the patient as being in the early stages of chronic myelogenous leukemia.

A. Ki-ras oncogene
* B. c-src oncogene
C. BCR/abl oncogene
D. c-myc oncogene

________ 4. Select the gene most likely associated with the condition. A 2 cm mass is removed from a 45 year old woman's
breast. Frozen sections are prepared and the diagnosis from the surgical pathologist is ductal adenocarcinoma, grade three. As the
attending physician, you request that the pathologist perform immunohistochemistry staining for this oncogene product to assist
you in predicting this woman's prognosis.
A. Ki-ras oncogene
B. c-src oncogene
* C. BCR/abl oncogene
D. c-myc oncogene

________ 5. Select the MOST LIKELY tumor suppressor gene to be implicated. A 42 year old woman patient visits her
physician for a yearly examination and a 1 cm lump is detected in her left breast. An excisional biopsy is performed and the
diagnosis is grade 2 adenocarcinoma. Further examination of the above patient's family history reveals that she has a mother and
sister who both have had breast cancer diagnosed before menopause. It is likely that these women have inherited a germline
mutation in this tumor suppressor gene.
* A. p53
B. Rb
C. Wilms tumor gene
D. APC gene

________ 6. The following are common characteristics of metastatic cells EXCEPT


* A. often resemble primary tumor
B. spread by lymphatic vessels
C. vascular spread
D. invade basement membranes
E. incapable of degrading extracellular matrix

________ 7. Which of the following statements regarding oncogenic DNA viruses is CORRECT?

A. transforming genes are homologous to c-oncs


B. play no role in human cancers
C. only one connected with human cancer is HTLV-1
* D. produce proteins capable of inactivating Rb and p53
E. cause cancer by insertional mutagenesis.

________ 8. Each of the following factors can contribute to weight loss in a cancer patient EXCEPT

A. anorexia
B. decreased food intake
* C. lowered metabolic rate
D. reduction of stored fat due to TNF
PATHO BOARD EXAM QUESTIONS 73
E. depression

________ 9. Select the most likely gene product. A nuclear protein which can induce apoptosis or alternatively can cause cell
cycle arrest by turning on expression of cyclin inhibitors.

A. Ras p21
B. HER2/neu
* C. BCR/abl
D. PDGF

________10. Select the most likely gene product. A nuclear protein which controls cell cycle by regulating the entry into S
phase by binding up essential transcription factors.

A. Ras p21
* B. HER2/neu
C. BCR/abl
D. PDGF

Chapter 9 ENVIRONMENTAL AND NUTRITIONAL PATHOLOGY

OBJECTIVES:

At the completion of this unit, the student should be able to ...

1. Know the basic components of a typical dose-response curve for acute toxicity.
2. Know the basic principles of xenobiotic metabolism.
3. Know the pathogenic mechanism of tobacco and alcohol abuse and its effects on various organs.
4. Know the pathogenic mechanism of selected dangerous and therapeutic drugs.
5. Know the pathogenic mechanism and its sources of selected outdoor and indoor pollutants.
6. Know the sources and effects on various organs of selected industrial and agricultural pollutants.
7. Define ionizing radiation, give its units of measurement, and describe its adverse effects--short and long term--on
tissues.
8. Discuss the radiosensitivity of the various types of tissues to radiation.
9. Discuss total body radiation, amounts involved, and acute and chronic effects.
10. List the adverse long-term and short-term effects of sunlight on the skin.
11. Know the various terms that is being use in the various types of injuries caused by mechanical, thermal, electrical and
athmosperic pressure.
12. Know the complications in the early and late stages of burns.
13. Know the differences between kwashiorkor and marasmus.
14. Know the active compound, consequences of the deficiency, and pathogenesis of the fat and water soluble vitamins
deficiency.
15. Know the 5 important trace elements and their function and deficiency syndromes.
16. Know the simplified scheme of the development of obesity.
17. Know the various methods in determining fat accumulation.

QUESTIONS:

________ 1. A patient with anorexia, dyspnea, a smooth sore tongue, numbness and tingling of feet, and mild paralysis of legs,
was found to have megaloblastic anemia which was not reversed by folate therapy. What was the vitamin deficiency?

* A. Vitamin B12
B. Vitamin B1
C. Vitamin B6
D. Vitamin A

________ 2. An infant developed petechial hemorrhages 5 days following birth. Deficiency in what vitamin might have been
contributory?

A. Vitamin A
B. Vitamin D
C. Vitamin E
* D. Vitamin K

________ 3. ) A chronic alcoholic developed signs and symptoms of beriberi (Wernicke encephalopathy, cardiac failure, sight
loss, peripheral neuropathy, muscle wasting, edema). What was the vitamin deficiency?

A. Vitamin B12
* B. Vitamin B1
C. Vitamin B6
D. Vitamin A

PATHO BOARD EXAM QUESTIONS 74


________ 4. A patient presented with night blindness, keratomalacia, corneal ulceration and bronchopneumonia. What vitamin
deficiency might be suspected?

* A. Vitamin A
B. Vitamin D
C. Vitamin E
D. Vitamin K

________ 5. A woman was diagnosed as having diabetes, myocardial infarction and gallstones. She was suffering from the
most common nutritional disorder in industrial counties. Name this disorder

A. Kwashiorkor
B. Marasmus
* C. Obesity
D. Beri-beri

________ 6. A patient presented with dermatitis, diarrhea and dementia. What is the most likely vitamin deficiency?

* A. Niacin
B. Pyridoxin
C. Ascorbic acid
D. Riboflavin

________ 7. A boy presented with irritability and ataxia, and was found to have hemolytic anemia, basophilic stippling of
erythrocytes, and dark gray gingival pigmentation. Chronic poisoning with _______________ was diagnosed.

A. Chromium
B. Cobalt
C. Cadmium
* D. Lead

_______ 8. A man was found semiconscious in his backyard beside a metal ladder, suffering from a burn on his hand and
cardiac arrhythmias. What was probably the cause of his problem?

A. Heat stroke
* B. Electrical injury
C. Too much alcohol
D. High altitude illness

________ 9. A healthy adult running a marathon in the summer developed hot dry skin, cessation of sweating, lactic acidosis,
hypocalcemia, and rhabdomyolysis. What is your diagnosis?_______________(Heat stroke.)

A. Heat cramps
B. Heat exhaustion
* C. Heat stroke
D. Hypothermia

________10. A boxer won his fight but received a black eye in the process. This lesion is best classified as

A. Laceration
* B. Contussion
C. Abrasion
D. Incision

________11. A severely depressed individual committed suicide by running his car engine in his closed garage. The cherry-red
color of his tissues at autopsy is due to the presence of

* A. Carboxyhemoglobin
B. Nitrogen
C. Carbon monoxide
D. Hemoglobin

________12. A patient involved in a house fire was hospitalized with subepidermal bullae which eventually healed without skin
grafts. These skin lesions represent

A. 1st degree burn


* B. 2nd degree burn
C. 3rd degree burn
PATHO BOARD EXAM QUESTIONS 75
D. 4th degree burn

________13. A boy, roller blading without knee and hand protection, fell and scraped his skin. Name this skin lesion.

A. Laceration
B. Contussion
*C. Abrasion
D. Incision

________14. A chronic alcoholic ran out of liquor and imbibed some anti-freeze containing ethylene glycol. He died and many
of his renal tubules contained oxalate crystals. This is an example of

* A. Biotransformation
B. Bioaccumulation
C. Biologic dose
D. Toxic dose

________15. An HIV positive drug abuser was found dead and autopsy demonstrated severe pulmonary edema and foreign
body granulomas. What drug most likely cause his death?

A. Cocaine
B. Marijuana
C. Shabu
* D. Heroin

Chapter 10 DISEASES OF INFANCY AND CHILDHOOD

OBJECTIVES:

At the completion of this unit, the student should be able to ...

1. Know the four time spans of a child.


2. Know at least 3 causes of death related with age.
3. Define the following: malformations, disruptions, deformations and syndrome.
4. Know the 3 major causes of anomalies and discuss some examples of each.
5. Discuss some important principles in developmental pathology.
6. Know how infants are classified based on gestational age and the basis of the classification.
7. Differentiate prematurity from fetal growth restriction and discuss the major risk factors of each.
8. Know the major organs affected in a preterm infant and briefly describe the changes seen in each.
9. Know and understand the Apgar score system.
10. Know the common and most important birth injuries.
11. Know the etiology, pathogenesis, morphology and clinical course of the following disorders
a. idiopathic respiratory distress syndrome of the newborn
b. necrotizing enterocolitis
c. fetal hydrops
12. Know the clinical presentation, pathogenesis, morphology (if any) and treatment of the following inborn errors of
metabolism:
a. phenylketonuria
b. galactosemia
c. cystic fibrosis
13. Know the definition, incidence and epidemiology, morphology and pathogenesis of SIDS.
14. Able to know the clinical presentation of the following benign tumors:
a. hemangiomas
b. lymphangiomas
c. teratomas
15. Able to know the morphology and clinical course of the following malignant tumors:
a. neuroblastoma
b. retinoblastoma
c. Wilm’s tumor

QUESTIONS

________ 1. The patient was a 2 year old female with a large right sided abdominal mass that was discovered by her parents
while changing a diaper. Histologic examination of the kidney after nephrectomy revealed a triphasic combination of blastemal,
stromal, and epithelial cell types. What is the diagnosis?
A. Leukemia
B. Neuroblastoma
* C. Wilms tumor
D. Renal cell carcinoma

________ 2. A baby was born without kidneys and a typical Potter complex. The abnormal amount of amniotic fluid found in
PATHO BOARD EXAM QUESTIONS 76
this complex is called oligohydramnios. Oligohydramnios is a form of

* A. Sequence
B. Malformation
C. Disruption
D. Deformation

________ 3. A child born to an Rh negative mother was found to have severe edema, which was apparently caused by a
hemolytic anemia and congestive heart failure. The subsequent work-up of the mother and the newborn disclosed severe Rh
incompatibility between them and the existence of anti-Rh antibodies in the maternal circulation. What is the clinical name for
this severe form of Rh incompatibility?

* A. Hydrops fetalis
B. Phenylketonuria
C. Oligohydramnios
D. Acute Lymphocytic Leukemia (ALL)

________ 4. A child born with an abnormality involving the cystic fibrosis membrane conductance regulator was found to have
signs of intestinal obstruction. Name this neonatal complication of this autosomal recessive disease.

A. Galactosemia
* B. Meconium Ileus
C. Phenylketonuria
D. Necrotizing enterocolitis

________ 5. A baby boy was born prematurely at 30 weeks of gestation. He weighed 2,000 grams. After birth the baby became
short of breath and had difficulties with breathing. The alveolar ducts of the lungs of such babies are lined with eosinophilic
structures called

A. Bronchiole
B. Alveolus
C. Meconium
* D. Hyaline membrane

________ 6. A newborn child was found to have an inborn error of amino acid metabolism. The progressive mental
deterioration that develops typically in this disease can be prevented by placing the child on a special diet that does not contain
an essential amino acid. Name this autosomal recessive disorder

A. Galactosemia
B. Meconium ileus
* C. Phenylketonuria
D. Cystic fibrosis

Chapters 11 & 12 CARDIOVASCULAR DISORDERS

OBJECTIVES:

At the completion of this unit, the student should be able to ...

1. Describe the anatomic structure and normal physiology of the heart and circulatory system.
2. Appreciate the inter-relationship among the cardiovascular, pulmonary and renal systems.
3. Define and describe the autonomic nervous system's role in cardiovascular regulation.
4. Describe the pathogenesis, pathophysiology, and symptoms of atherosclerotic coronary and peripheral vascular
disease.
5. Define and be able to use essential cardiovascular terminology, such as angina pectoris, ischemia, infarction,
hypertrophy, dilatation, stenosis, regurgitation, pre-load, after-load, stroke volume, and cardiac index.
6. Identify the major risk factors for coronary artery disease.
7. Describe the use of laboratory tests in the diagnosis and treatment of cardiovascular disease (i.e., cholesterol
fractionation, CPK isoenzymes).
8. Define and describe the pathogenesis, pathophysiology and symptoms of heart failure: right-sided vs. left-sided;
systolic vs. diastolic dysfunction.
9. Explain the compensatory mechanisms in heart failure.
10. Recognize and discuss exacerbating factors in heart failure, such as anemia, hypoxemia, fever, uncontrolled
hypertension, hyperthyroidism, tachyarrhythmias, drug toxicity and ischemia.
11. Identify common types of valvular heart disease, such as aortic stenosis, mitral regurgitation and (rheumatic)
mitral stenosis.
12. Discuss the pathogenesis of acute myocardial infarction, as well as its electrical and mechanical complications.
13. Identify the cardiovascular complications of hypertension, such as left ventricular hypertrophy and aortic
dissection.
PATHO BOARD EXAM QUESTIONS 77
14. Recognize the gross and microscopic findings associated with atherosclerotic coronary artery disease,
myocardial infarction, chamber hypertrophy and dilatation.
15. Define the role and pharmacology of commonly prescribed cardiac medications in the treatment of cardiac
diseases (digoxin, diuretics, beta-blockers, ACE inhibitors, anti-arrhythmic, nitrates, anti-coagulants and lipid lowering
agents).
16. State the differential diagnosis of chest pain and briefly describe how commonly used non-invasive studies
(EKG, Echo, stress test) may be helpful diagnostically.

QUESTIONS

________ 1. Microscopic examination of the myocardial infarction in a patient who expired 24-48 hours following
the occlusion of the coronary artery shows:
A. Well-developed granulation tissue
* B. Necrosis of myocardium and infiltrates of polymorphonuclear leukocytes
C. Plasma cell infiltrates
D. Fibroblastic proliferation

________ 2. A 60 year old man is admitted to the hospital with a chief complaint of substernal chest pain and
perspiration. Examination reveals an obese man with a long history of angina and intermittent claudication.
a blood test after 4 hours would usually reveal elevated levels of:
A. Aspartate transaminase
B. Acid phosphatase
* C. Creatine kinase
D. Lactate dehydrogenase

________ 3. Hematuria and flank pain are observed in a patient with a seven-day history of trans-mural myocardial
infarction. What is the most likely underlying cause?
A. Emboli from ventricular mural thrombi
B. Emboli from atrial mural thrombi
* C. Emboli from aortic valve vegetations
D. Acute pyelonephritis

________ 4. A patient with a massive myocardial infarction dies suddenly on the sixth hospital day. Autopsy reveals: E
A. Lymphocytic infiltrate of myocardium
B. Plasma cell infiltrate of coronary arteries
C. Bilateral occlusions of coronary sinuses
* D. Rupture of left ventricle and hemopericardium

________ 5. A newborn infant is noted to have a holosystolic murmur. The infant is not cyanotic. Which of the following
congenital abnormalities is most likely to be present. B
A. Ebstein's anomaly
* B. Complete transposition of the great vessels
C. Ventricular septal defect
D. Atrial septal defect

________ 6. A benign, painful tumor originating from the neuromyoarterial receptors in the tips of the fingers or toes. Select
the most likely diagnosis.
A. aldosterone-secreting adrenal tumor
* B. glomus tumor
C. bacillary angiomatosis
D. berry aneurysm

________7. A 65 year old male patient with a history of poorly controlled hypertension develops sudden 'tearing'
pain in the chest followed by signs of cardiac tamponade. Select the most likely diagnosis. F
* A. dissecting aneurysm
B. angiosarcoma
C. bacillary angiomatosis
D. berry aneurysm

________ 8. A 60 year old male presents with a history of sinusitis and cough. Chest x-ray shows large cavitary pulmonary
infiltrates. Routine urinalysis reveals hematuria. A renal biopsy shows evidence of vasculitis in addition to glomerulonephritis.
Select the most likely diagnosis.
* A. Wegener granulomatosis
B. angiosarcoma
C. bacillary angiomatosis
D. berry aneurysm

________ 9. A patient develops a petechial reaction to a thiazide diuretic. A skin biopsy reveals fibrinoid necrosis
of small blood vessels with acute inflammation and nuclear debris. Select the most likely diagnosis.
A. aldosterone-secreting adrenal tumor
* B. leukocytoclastic vasculitis
C. bacillary angiomatosis
PATHO BOARD EXAM QUESTIONS 78
D. berry aneurysm

________10. A 30 year old woman complains of severe pain and pallor of her hands and toes, precipitated by
exposure to cold. Select the most likely diagnosis.
* A. Raynaud phenomenon
B. angiosarcoma
C. bacillary angiomatosis
D. berry aneurysm

________11. A 50 year-old female who has had a radical mastectomy and axillary node dissection for breast cancer a year ago,
now notices that her arm becomes swollen by the end of the day. Select the most likely diagnosis.
A. aldosterone-secreting adrenal tumor
B. angiosarcoma
C. bacillary angiomatosis
* D. lymphedema

12. A 75 year-old male presents with headache and visual disturbances. A thickened, tortuous temporal artery is
palpable. Select the most likely diagnosis.
A. aldosterone-secreting adrenal tumor
B. angiosarcoma
* C. giant cell arteritis
D. berry aneurysm

13. A renal biopsy of a 40 year-old African-American male revealed fibrinoid necrosis and "onion-skin" lesions of renal
arterioles. Select the most likely diagnosis.
A. aldosterone-secreting adrenal tumor
* B. malignant hypertension
C. bacillary angiomatosis
D. berry aneurysm

________14. Which of the following congenital heart diseases presents with cyanosis at birth or during the first few weeks of
postnatal life?
A. atrial septal defect, ostium primum type
* B. Tetralogy of Fallot
C. ventricular septal defect, membranous
D. ventricular septal defect, muscular

________15. Myocardial infarct involving the posterior portion of the interventricular septum is caused by an occlusion of
which coronary artery?
* A. right
B. left anterior descending
C. left circumflex
D. left diagonal

________16. Which of the following is the most characteristic cardiac lesion of acute rheumatic fever?
A. streptococcal abscess
* B. Aschoff body
C. calcification of aortic valves
D. mitral stenosis

________17. Which of the following diseases is most often associated with non-bacterial thrombotic endocarditis?
A. rubella
B. syphilis
* C. carcinoma of the stomach
D. intestinal carcinoid metastatic to liver

________18. Classically, left ventricular rupture occurs at 1-2 weeks post-MI when the heart is maximally infiltrated by
A. lymphocytes
* B. macrophages
C. fibroblasts
D. mature collagen

________19. Jones major criteria for diagnosing rheumatic fever include each the following EXCEPT
A. carditis
* B. aortitis
C. polyarteritis
D. chorea

________20. The most common cause of death during the first two hours following an acute myocardial infarction is
* A. arrhythmia
B. loss of myosin from injured cells
C. loss of troponin T from necrotic cells
PATHO BOARD EXAM QUESTIONS 79
D. loss of calcium from sarcoplasmic reticulum

Instructions: Choose the BEST answer

1. Acute myelogenous leukemia is:


a. most common in females
b. most common in adults over 60 years old
c. occasionally aleukemic
d. associated with massive splenomegaly
1. Answer: C Robbins 6th ed pp. 677 MPL 0.25

2. Hodgkin’s lymphoma with the worst prognosis:


a. lymphocyte predominance
b. nodular sclerosis
c. mixed cellularity
d. lymphocyte depletion
2. Answer: D Robbins 6th ed pp. 672 MPL 0.25

3. The most important difference between a benign reactive lymph node and lymphoma :
a. interstitial fibrosis
b. follicular hyperplasia
c. effacement of normal architecture
d. presence of plasma cells
3. Answer: C Robbins 6th ed pp. 649 MPL 0.33

4. Plasma cells are characterized by


a. abundant surface immunoglobulins
b. Fc surface receptors
c. Abundant cytoplasm immunoglobulins
d. Complement surface receptors
4. Answer: C Robbins 6th ed pp. 82 MPL 1

5. Paroxysmal hypertension is most typically associated with which of the following?


a. adrenal adenoma
b. thyroid adenoma
c. parathyroid adenoma
d. pheochromocytoma
5. Answer: D Robbins 6th ed pp. 1166 MPL 1

6. The association of medullary carcinoma of the thyroid with pheochromocytoma with or without
parathyroid hyperplasia or adenoma is called:
a. Conn’s syndrome
b. Cushing’s syndrome
c. Sipple’s syndrome
d. Waterhouse-Friedricksen’s syndrome
6. Answer: C Robbins 6th ed pp. 1167 MPL 0.25

7. Which of the following is the most common cause of Cushing’s syndrome


a. exogenous corticosteroids
b. adrenal cortical carcinoma
c. oat cell carcinoma of the lungs
d. basophilic adenoma of the pituitary
7. Answer: A Robbins 6th ed pp. 1153 MPL 0.33

8. A destructive lesion in this area will result in diabetes insipidus:


a. mamillothalamic tract
b. adenohypophysis
c. subthalamic fasciculus
d. supraoptic and paraventricular hypothalamic nuclei
PATHO BOARD EXAM QUESTIONS 80
8. Answer: D Robbins 6th ed pp. 1129 MPL 0.25

9. Conn’s syndrome is primarily associated with an excess of :


a. glucocorticoids
b. mineralocorticoids
c. thyroid hormone
d. ADH
9. Answer: B Robbins 6th ed pp. 1155 MPL 0.25

10. Primary hyperaldosteronism is associated with:


a. hyponatremia
b. hypotension
c. elevation of plasma renin levels
d. hypokalemia
10. Answer: D Robbins 6th ed pp. 1155 MPL 0.5

11. The single most frequent adrenal lesion producing primary hyperaldosteronism:
a. single unilateral adenoma
b. multiple unilateral adenomas
c. bilateral adenomas
d. carcinoma
11. Answer: A Robbins 6th ed pp. 1155 MPL 0.25

12. Truncal obesity, easy bruising and osteoporosis is associated with:


a. craniopharyngioma
b. adrenal adenoma
c. thyroid adenoma
d. parathyroid adenoma
12. Answer: B Robbins 6th ed pp. 1154-1155 MPL 1

13 Urinalysis results show proteinuria, red blood cells and casts, and 1-2 pus cells per HPO. These findings
are consistent with:
a. acute cystitis
b. acute pyelonephritis
c. chronic pyelonephritis
d. acute post-streptococcal glomerulonephritis
13. Answer: D Robbins 6th ed pp. 950 MPL 1

14. Membranous glomerulopathy and poststreptococcal glomerulonephritis are similar in that they both:
a. are self-limiting and transient renal diseases
b. are commonly associated with nephrotic syndrome
c. are most commonly seen in children
d. have glomerular immune complex deposits
14. Answer: D Robbins 6th ed pp. 953 MPL 0.5

15. A patient with hemoptysis and renal failure has a renal biopsy which reveals crescentic
glomerulonephritis, with linear deposits of IgG and C3 on immunofluorescence. Diagnosis?
a. acute post-infectious glomerulonephritis
b. membranous nephropathy
c. Goodpasture’s syndrome
d. Minimal change disease
15. Answer: C Robbins 6th ed pp. 951 MPL 1

16. The most characteristic feature of rapidly progressive glomerulonephritis:


a. crescent formation
b. hyaline nodules
c. subepithelial dense deposits
d. thickened capillary loops
PATHO BOARD EXAM QUESTIONS 81
16. Answer: A Robbins 6th ed pp. 951 MPL 1

17. Benign nephrosclerosis is best defined as:


a. fibrinoid necrosis of afferent arterioles with ischemic atrophy of nephrons
b. hyalinization of afferent arterioles with ischemic atrophy of nephrons
c. diffuse thickening of glomerular capillary basement membrane leading to glomerulosclerosis
d. deposition of basement-like material within the glomerular mesangium leading to
glomerulosclerosis
17. Answer: B Robbins 6th ed pp. 981-982 MPL 0.25

18. Ischemic acute tubular necrosis occurs most commonly in association with:
a. septic shock
b. ruptured aneurysm
c. carcinoma of the prostate
d. malignant hypertension
18. Answer: A Robbins 6th ed pp. 969 MPL 1

19. The most common appearance of nephrotic syndrome in adults is:


a. proliferative glomerulonephritis
b. membranous glomerulonephritis
c. rapidly progressive glomerulonephritis
d. wire-loops

19. Answer: B Robbins 6th ed pp. 953 MPL 0.25

20. “Nutmeg liver” is a gross finding seen in:


a. petechial hemorrhages
b. viral hepatitis
c. alcoholism
d. chronic passive congestion
20. Answer: D Robbins 6th ed pp. 116-117 MPL 1

21. Which of the following is associated with high levels of serum alkaline phosphatase?
a. hemolytic anemia
b. metastatic carcinoma to the liver
c. Dubin-Johnson syndrome
d. Gilbert’s syndrome
21. Answer: B Robbins 6th ed pp. 891 MPL 1

22. The most common outcome in-patients with acute Hepatitis B is:
a. complete recovery without residual dysfunction
b. chronic persistent hepatitis
c. chronic active hepatitis
d. postnecrotic cirrhosis
22. Answer: A Robbins 6th ed pp. 858 MPL 1

23. In a patient suspected of having viral hepatitis, which laboratory finding would indicate the most
severe and life threatening amount of liver damage?
a. prolonged prothrombin time
b. high serum bilirubin
c. high serum aspartate transaminases
d. high serum alkaline phosphatase
23. Answer: A Robbins 6th ed pp. 867 MPL 1

24. The most important substance in the formation of gallstones is:


a. calcium c. bile salts
b. bilirubin d. lecithin
th
24. Answer: C Robbins 6 ed pp. 894 EXAM QUESTIONSMPL 0.25
PATHO BOARD 82
25. Cirrhosis produces most of its clinical manifestations by means of:
a. hepatic necrosis leading to insufficient number of liver cells to sustain life
b. cancerous change in regenerating nodules
c. obstruction of large biliary passages
d. vascular derangements in the liver
25. Answer: D Robbins 6th ed pp. 853 MPL 0.25

26. The major cause of portal hypertension is:


a. Intrahepatic c. prehepatic
b. Posthepatic d. cardiac
th
26. Answer: A Robbins 6 ed pp. 855 MPL 1

27. The most common factor in the formation of esophageal varices is obstruction at the level of:
a. esophageal vein
b. hepatic vein
c. intrahepatic vessels
d. portal vein
27. Answer: C Robbins 6th ed pp. 783 MPL 0.25

28. When a person dies suddenly from a “heart attack” the most likely event causing it will be:
a. aortic aneurysm
b. coronary artery embolism
c. congestive heart failure
d. arrhythmia
28. Answer: D Robbins 6th ed pp. 562 MPL 1

29. Acute rheumatic heart disease is associated with:


a. arthritis
b. emboli to multiple organs
c. cardiac tamponade
d. right to left shunt
29. Answer: A Robbins 6th ed pp. 572 MPL 0.5

30. The most frequent and clinically significant residual lesion of acute rheumatic fever is:
a. myocardial fibrosis
b. aortic stenosis
c. mitral stenosis
d. pericardial adhesion
30. Answer: D Robbins 6th ed pp. 570-571 MPL 0.25

31. The absolute criterion of cardiomegaly:


a. increase in size
b. increase wall thickness
c. increase in weight
d. change in shape
31. Answer: C Robbins 6th ed pp. 544 MPL 1

32. Myocardial disease gradually producing 4-chamber dilatation:


a. viral myocarditis
b. hypertrophic cardiomyopathy
c. congestive cardiomyopathy
d. restrictive cardiomyopathy
32. Answer: C Robbins 6th ed pp. 579-581 MPL 1

33. Necrotizing infection caused by highly virulent organisms seeding a normal valve:
a. rheumatic heart disease
b. SLE endocarditis
c. acute infective endocarditis
PATHO BOARD EXAM QUESTIONS 83
d. marantic Endocarditis
33. Answer: C Robbins 6th ed pp. 572-573 MPL 1

34. The most common cause of intestinal obstruction is:


a. adhesions
b. volvulus
c. neoplasm
d. hernia
34. Answer: A Robbins 6th ed pp. 825 (table) MPL 1

35. A two-week old boy has projectile vomiting. The most likely diagnosis is :
a. pyloric stenosis
b. esophageal atresia
c. annular pancreas
d. incomplete rotation of the gut
35. Answer: A Robbins 6th ed pp. 789 MPL 0.25

36. An endoscopic gastric biopsy finding of intestinal type epithelia is most likely due to:
a. chronic gastritis
b. congenital heterotopia
c. precancerous dysplasia
d. metastatic carcinoma
36. Answer: A Robbins 6th ed pp. 790 MPL 0.25

37. Most characteristic of right sided colon cancer compared to left-sided cancer:
a. anemia
b. associated with polyposis
c. bowel obstruction
d. diarrhea
37. Answer: A Robbins 6th ed pp. 834 MPL 0.33

38. Acute erosive gastritis is characterized by:


a. superficial and multiple gastric mucosal ulceration
b. pus in the gastric mucosa
c. deep ulcer with scarred base
d. frequent association with gastric cancer
38. Answer: A Robbins 6th ed pp. 789-790 MPL 0.25

39. The most common site of chronic gastric peptic ulcer:


a. lesser curvature at the antral-body junction
b. anterior wall of the duodenal verge
c. greater curvature at the mid-antrum
d. esophago-gastric junction
39. Answer: A Robbins 6th ed pp. 795 MPL 0.25

40. The definitive finding in acute appendicitis:


a. obstruction of the lumen by a fecalith
b. mucus inspissation in the lumen
c. neutrophils in the muscularis
d. serosal congestion and margination of polys
40. Answer: C Robbins 6th ed pp. 839 MPL 1

41. Morphologic appearance favoring benign rather than malignant gastric ulcer:
a. large ulcer
b. heaped up margin
c. antral location
d. folds radiating from the ulcer
41. Answer: C Robbins 6th ed pp. 795 EXAM QUESTIONS
MPL 0.25
PATHO BOARD 84
42. Most frequently found neoplasm in the appendix:
a. adenoma
b. polyp
c. carcinoid
d. adenocarcinoma
42. Answer: C Robbins 6th ed pp. 840 MPL 1

43. The most common malignant soft tissue tumor:


a. rhabdomyosarcoma
b. malignant fibrous histiocytoma
c. leiomyosarcoma
d. liposarcoma
43. Answer: D Robbins 6th ed pp. 1261 MPL 0.25

44. Brown tumors of bone is associated with:


a. hyperparathyroidism
b. trauma
c. metastatic carcinoma
d. vitamin deficiency
44. Answer: A Robbins 6th ed pp. 1228 MPL 1

45. The most frequent site of compression fracture is:


a. femur
b. radio-ulnar area
c. clavicle
d. vertebrae
45. Answer: D Robbins 6th ed pp. 1229-1231 MPL 1

46. Rheumatoid arthritis usually has its origin in:


a. ball and socket joints
b. lumbo-sacral joints
c. small peripheral joints
d. major weight-bearing joints
46. Answer: C Robbins 6th ed pp. 1251 MPL 1

47. Most common primary malignant bone tumor in 15-25 years of age:
a. chondrosarcoma
b. osteosarcoma
c. Ewing’s sarcoma
d. Chondroblastoma
47. Answer: B Robbins 6th ed pp. 1236 MPL 1

48. The most likely malignant bone tumor in a 65 yo man is:


a. Giant cell tumor
b. Osteosarcoma
c. Metastatic prostatic cancer
d. Multiple myeloma
48. Answer: C Robbins 6th ed pp. 1245 MPL 1

49. The most common primary type of bronchogenic carcinoma is:


a. metastatic carcinoma
b. adenocarcinoma
c. squamous cell carcinoma
d. small cell carcinoma
49. Answer: C Robbins 6th ed pp. 743-744 MPL 1

50. The source of surfactant is believed to be:


a. Type I pneumocyte
PATHO BOARD EXAM QUESTIONS 85
b. Type II pneumocyte
c. Mast cell
d. Alveolar mcrophage
50. Answer: B Robbins 6th ed pp. 471 MPL 0.33

51. The single most characteristic feature of bronchiectasis is:


a. excessive mucus secretion
b. immotile cilia
c. dilatation of bronchi
d. alpha-1 antitrypsin deficiency
51. Answer: C Robbins 6th ed pp. 716 MPL 1

52. Non-communicating hydrocephalus means obstruction:


a. between ventricles and subarachnoid space
b. at the cisterna ambiens
c. at the arachnoid granulations
d. at the vein of Galen
52. Answer: B Robbins 6th ed pp. 1298-1299 MPL 0.33

53. The most common cause of bacterial meningitis in a patient <2 months old is:
a. Escherichia coli
b. Hemophilus influenza
c. Streptococcus pneumoniae
d. Neisseria meningitides
53. Answer: A Robbins 6th ed pp. 1314 MPL 1

54. The most important and consistent site for histologic lesion in parkinsonism is:
a. caudate nucleus
b. substantia nigra
c. red nucleus
d. putamen
54. Answer: B Robbins 6th ed pp. 1334 MPL 1

PATHO BOARD EXAM QUESTIONS 86


55. Increased hormonal stimulation may cause cells to undergo:
a. atrophy c. hyperplasia
b. metaplasia d. dysplasia
th
55. Answer: C Robbins 6 ed pp. 32-33 MPL 1

56. A reversible change or alteration in adult cells characterized by variation in size, shape and organization:
a. metaplasia c. dysplasia
b. hyperplasia d. atrophy
56. Answer: C Robbins 6th ed pp. 466 MPL 1

57. Type of cellular adaptation where there is marked increase in the number of autophagosomes accompanied
by decrease in the number of mitochondria, myofilaments, ad endoplasmic reticulum:
a. hypertrophy c. hyperplasia
b. atrophy d. metaplasia
th
57. Answer: B Robbins 6 ed pp. 36 MPL 1

58. The most common cause of hypoxic injury is;


a. ischemia
b. depletion of oxygen-carrying capacity of blood
c. poisoning of the oxidative enzymes within the cells
d. thrombosis
58. Answer: A Robbins 6th ed pp. 4 MPL 1

59. The sum of all morphologic changes following cell death:


a. necrosis c. autolysis
b. heterolysis d. autophagy
th
59. Answer: A Robbins 6 ed pp. 2 MPL 1

60. The characteristic cellular infiltrate in the early phase of acute inflammation:
a. lymphocyte c. plasma cells
b. neutrophils d. eosinophil
60. Answer: B Robbins 6th ed pp. 51 MPL 1

61. The immediate transient phase of vascular permeability in most types of tissue injury is mediated by:
a. histamine c. complement
b. prostaglandins d. bradykinin
th
61. Answer: A Robbins 6 ed pp. 66 MPL 0.5

62. The characteristic cell of a granuloma:


a. Langhan’s type giant cell c. lymphocyte
b. epithelioid cell d. plasma cell
62. Answer: B Robbins 6th ed pp. 83-84 Answer: MPL 1

63. The essential features of a granulation tissue:


a. fibroblasts and inflammatory cells c. edema & inflammatory cell
b. small blood vessels and fibroblasts d. inflammatory cells & vessels
th
63. Answer: B Robbins 6 ed pp. 102-103 MPL 0.5

64. Acute pancreatitis:


a. coagulation necrosis c. liquefaction necrosis
b. caseation necrosis d. enzymatic fat necrosis
64. Answer: D Robbins 6th ed pp. 17-18 MPL 1

65. Acute inflammation is characterized by the following:


A. relatively long duration C. emigration of macrophages
B. proliferation blood vessels and connective tissue D. exudation
65. Answer: D Robbins 6th ed pp. 51 MPL 1

66. The most common mechanism of vascular leakage elicited by free oxygen radicals:
87
PATHO BOARD EXAM QUESTIONS
A. endothelial contraction C. leukocyte-dependent leakage
B. direct endothelial cell injury D. regeneration of endothelial cell
66. Answer: B Robbins 6th ed pp. 55 / 76 MPL 0.25

67. Leukocyte adhesion, chemotaxis and activation are important inflammatory mediator processes performed
by complement factor:
A. C7a C. C5a
B. C6a D. C1a
th
67. Answer: C Robbins 6 ed pp. 61 MPL 0.33

68. Oxygen-derived free radicals are implicated in:


A. endothelial cell damage C. tissue regeneration
B. release of histamine D. prostaglandin release
th
68. Answer: A Robbins 6 ed pp. 76 MPL 0.5

69. This inflammatory infiltrate is the prima donna of chronic inflammation because of the great number of
biologically active substances it releases:
A. lymphocytes C. neutrophils
B. macrophage D. plasma cells
69. Answer: B Robbins 6th ed pp. 79 MPL 1

70. Adaptation has happen:


a. if the cell undergoes necrosis c. ..is replaced by another cell type
b. loses its genetic apparatus d. undergoes dystrophic calcification
70. Answer: C Robbins 6th ed pp. 36-37 MPL 1

71. Characteristics of benign tumors:


a. invasive c. immortal
b. cohesive d. escape from regulatory control
th
71. Answer: B Robbins 6 ed pp. 270 MPL 0.33

72. The most reliable feature of malignancy:


a. metastasis c. rate of growth
b. differentiation d. invasion
72. Answer: A Robbins 6th ed pp. 268 MPL 1

73. The most common route of metastasis of a carcinoma:


a. contiguity c. hematogenous
b. lymphatic d. iatrogenic
th
73. Answer: B Robbins 6 ed pp. 269 MPL 1

74. The most important prognostic factor for malignancy:


a. stage c. invasion
b. grade d. localization
74. Answer: A Robbins 6th ed pp. 321-322 MPL 1

75. A 25-year-old man with a history of intravenous drug use has had one bout of Pneumocystis carinii
pneumonia which was successfully treated a year ago. He now has a WBC count of 3600/microliter with
differential count of 78 segs, 3 bands, 5 lymphs, and 14 monos. He has recently developed painful, progressive
visual loss involving both eyes. Which of the following infections is he most likely to have:
A. Klebsiella pneumoniae C. Mycobacterium tuberculosis
B. Staphylococcus aures D. Cytomegalovirus
75. Answer: D Robbins 6th ed pp. 1319 MPL 0.25

76. A 65-year-old previously healthy woman has had a non-productive cough increasing in severity over the
last 10 days. She has a temperature of 38.0 C. A chest radiograph shows faint interstitial infiltrates but no areas
of consolidation. A CBC reveals a WBC count of 8900/microliter with differential count of 60 segs, 3 bands, 25
lymphs, and 12 monos. A sputum gram stain reveals a few gram positive cocci, gram negative cocci, and
Candida with few neutrophils. Her condition gradually improves over the next week. Which of the following
infectious agents is most likely responsible for her illness:
A. Klebsiella pneumoniae C. Mycobacterium tuberculosis
88
PATHO BOARD EXAM QUESTIONS
B. Staphylococcus aures D. Influenza A virus
76. Answer: D Robbins 6th ed pp.348 MPL 0.25

77. A 39-year-old man had an acute febrile illness with fever, pharyngitis, lymphadenopathy, and malaise
following a trip to a city known for its commercial sex workers. In how many years would you predict that his
peripheral blood CD4 lymphocyte count will drop below 200/microliter:
A. one C. ten
B. five D. fifteen
77. Answer: C Robbins 6th ed pp.246-248 MPL 0.25

78. A 20-year-old commercial sex worker has lower abdominal pain of 24 hours duration. There is no previous
history of pain. She is febrile, her total WBC is 29,000/mm3 with 85% neutrophils. She has a markedly tender
lower abdomen on palpation. Laparotomy reveals a distended, fluid-filled reddened left fallopian tube that is
about to rupture. The tube is removed. A culture of the fluid from the tube is most likely to grow:
A. TB bacilli C. AIDS virus
B. Neisseria gonorrhea D. Human Papilloma virus
78. Answer: B Robbins 6th ed pp.362 MPL 0.25

79. A 10-year-old child has a severe watery diarrhea that causes her to become severely dehydrated over the
past two days. The persistent activation of adenylate cyclase by GTP leads to excessive intestinal fluid loss.
Which of the following organisms is most likely to produce a toxin that has this effect:
A. Eschericia coli C. Vibrio cholera
B. Entamoeba histolytica D. Salmonella enteritidis
79. Answer: C Robbins 6th ed pp.808 MPL 0.5

80. An 18-year-old man is plowing a field on a hillside. He discovers a deep, penetrating puncture wound to
his foot. He walks home and washes the soil off the wound site and puts a bandage over it. A week later the
wound is healing, and the leg is not swollen, but he begins to experience generalized muscle spasms, with
stiffness and pain in his shoulders and back. He has difficulty with eating and swallowing, with painful stiff jaw
and facial muscles. Which of the following organisms is most likely responsible for these findings :
A. Clostridium tetani C. Staphylococcus aureus
B. Clostridium perfringens D. Streptococcus pneumoniae
80. Answer: A Robbins 6th ed pp.368-369 MPL 1

81. A 41-year-old man has a chronic cough and recently experienced an episode of hemoptysis. A chest
radiograph shows a cavitary 4 cm nodule in the right upper lung. There are a few adjacent smaller nodules with
calcifications. The best diagnosis is:
A. AIDS with cytomegalovirus C. Bronchopneumonia with pseudomonas
B. Secondary tuberculosis D. peripheral organizing pulmonary thromboembolus
81. Answer: C Robbins 6th ed pp.376-377 MPL 0.25

82. A 40-year-old man has had a productive cough with yellowish sputum for several days. A complete blood
count shows that he has a total WBC count of 15,700/microliter with differential count of 73 segs, 10 bands, 12
lymphs, and 5 monos. He has a fever of 38.9 C. A chest radiograph reveals a nodule in the left upper lobe that
has a central cavity with an air-fluid level. Which of the following findings is most likely to be present in his
sputum|:
A. +2 acid fast bacilli C. 4+ gram positive cocci
B. atypical squamous cells D. hemosiderin-laden macrophages
82. Answer: C Robbins 6th ed pp.367 / 722 MPL 0.33

83. A 78-year-old woman has a history of Alzheimer's disease. She dies after an illness lasting only a two-day
history of fever and cough productive of copious amounts of yellowish sputum. Her vital signs on admission
included blood pressure 110/75 mm Hg, pulse 105, respirations 26, and temperature 38.1 C. She had dullness to
percussion at the right lung base. The microscopic findings from the right lower lobe at autopsy include
numerous neutrophils within the alveolar spaces, with minimal destruction of alveolar walls. What was the most
likely infectious cause for her pulmonary disease:
A. Influenza A virus C. streptococcus pneumoniae
B. Mycobacterium tuberculosis D. Cytomegalovirus
83. Answer: C Robbins 6th ed pp.719 MPL 0.33

89
PATHO BOARD EXAM QUESTIONS
84. A 22-year-old woman has experienced episodes of myalgias, pleural effusions, pericarditis, and arthralgias
without joint deformity over the course of several years. She has continued working at her job. A CBC reveals a
mild normocytic anemia. The best laboratory screening test to begin the workup for her condition would be
a(an):
A. CD4 lymphocyte count C. antinuclear antibody test
B. Blood culture D. creatine phosphokinase
th
84. Answer: C Robbins 6 ed pp.216-217 MPL 0.25

85. A male infant was born at term. No congenital anomalies were noted at birth. A year later he now has
failure to thrive and has been getting one bacterial pneumonia after another with both Hemophilus influenzae
and Streptococcus pneumoniae cultured from his sputum. You should most strongly suspect that he has which
of the following diseases
A. Di George Syndrome C. EBV infection
B. Selective IgA deficiency D. X-inked agammaglobulinemia
th
85. Answer: D Robbins 6 ed pp.232-233 MPL 0.25

86. An appropriate, useful type I hypersensitivity response of the immune system, accompanied by eosinophilia,
would be directed against:
A. amyloid protein C. neoplasms
B. pinworm infection D. inhaled dusts
th
86. Answer: B Robbins 6 ed pp.82 MPL 1

87. A 54-year-old woman is found to have a blood pressure of 170/110 mm Hg. Her fingers become cold and
painful upon exposure to cold. She has mild dyspnea, but no wheezing. The antinuclear antibody test is positive
with a titer of 1:256 and a nucleolar pattern. Her serum urea nitrogen is 15 mg/dL with creatinine of 1.1 mg/dL.
These findings most strongly suggest that she has which of the following autoimmune diseases:
A. discoid lupus erythematosus C. polymyositis-dermatomyositis
B. progressive systemic sclerosis D. rheumatoid arthritis
87. Answer: B Robbins 6th ed pp.226-229 MPL 0.25

88. Several hours after going on a hike through dense foliage, a 40-year-old man notices a slightly raised and
tender irregular reddish rash on one forearm that was not covered by clothing. This rash gradually increases in
intensity for a couple of days and then fades after two weeks. The rash is most indicative of:
A. Type I C. Type III
B. Type II D. Type IV
88. Answer: D Robbins 6th ed pp.204-206 MPL 0.25
89. The second pregnancy of a 23-year-old woman appears uncomplicated until ultrasound performed at 19
weeks shows hydrops fetalis. The fetal organ development is consistent for 19 weeks, and no congenital
anomalies are noted. Her first pregnancy was uncomplicated and resulted in the birth of a normal girl at term.
The current pregnancy yields a baby born prematurely at 32 weeks gestation with marked icterus. The baby is
also markedly anemic, and an exchange transfusion is performed. Which of the following immunologic
mechanisms best explains these findings
A. anti-receptor antibody C. immune complex formation
B. complement-mediated cell destruction D. delayed type hypersentsitivity
89. Answer: B Robbins 6th ed pp.199-201 MPL 0.25

90. A man in his mid-30's presents with malar skin rash, polyarthritis with swelling and warmth of his hands,
and sensitization to cold. On physical examination he has generalized lymphadenopathy and pale conjunctivae.
Laboratory findings include a hemoglobin of 9.5 g/dL, total WBC count of 2100/microliter, total serum protein
8.8 g/dL, albumin 3.6 g/dL, creatinine 1.1 mg/dL, and creatine kinase of 468 U/L. His antinuclear antibody test
is positive at 1:256. The best additional serologic test to help determine his underlying disease process is:
A. Anti-centromere C. Rheumatoid Factor
B. Anti-RNP D. Serum C1q complement level
th
90. Answer: B Robbins 6 ed pp.218 MPL 0.25

91. Several weeks following a pharyngitis in which the throat culture grew group A hemolytic streptococcus, a
child is noted to have hematuria on urinalysis. An immunofluorescence staining pattern in a renal biopsy shows
granular deposition of IgG and complement around glomerular capillary loops. Which of the following immune
hypersensitivity mechanisms is most likely responsible for this pattern of findings
A. Type I C. Type III
B. Type II D. Type IV
90
PATHO BOARD EXAM QUESTIONS
91. Answer: C Robbins 6th ed pp.204-226 MPL 0.25

92. A 48-year-old man has a chronic cough with fever that have persisted for several months. The chest
radiograph reveals a diffuse reticulonodular pattern. Microscopically on transbronchial biopsy there are focal
areas of inflammation containing epitheloid macrophages, Langhan's giant cells, and lymphocytes. These
findings are most typical for which of the following immunologic responses:
A. Type I C. Type III
B. Type II D. Type IV
92. Answer: D Robbins 6th ed pp.204-226 MPL 0.25

93. A 31-year-old woman with chest pain for the past week has a chest radiograph that shows modest bilateral
pleural effusions. On chest CT scan, the pleural effusions, as well as a pericardial effusion, are observed. A
thoracentesis on the left yields clear fluid with a low protein and cell count. She is found to have an anti-double
stranded DNA titer of 1:512. If she is later found to have a serum urea nitrogen that is 55 mg/dL, it will
probably be the result of which of the following pathologic processes
A. atherosclerosis C. amyloid deposition
B. glomerular immune deposits D. Anti-glomerular basement membrane
93. Answer: B Robbins 6th ed pp.216-225 MPL 0.25

94. A 5 cm well-encapsulated mass was excised from the breast of a 21-year-old woman.
Histologically, the mass was composed of elongated, ductlike structures surrounded by
loose, fibrous connective tissue. Diagnosis
A. fibroadenoma
B. intraductal papilloma
C Pagets disease
D. Phyllodes tumor
94. Answer: A Robbins 6th ed pp. 1102-1103 MPL 0.25

95. An 18-year-old medical student complains of poor concentration and easy fatigability. She
had heavy menstrual bleeding but was otherwise healthy. Examination of the blood showed
a microcytic, hypochromic erythrocytes. Diagnosis?
A. aplastic anemia
B. sickle cell anemia
C. thalassemia
D. iron deficiency anemia
95. Answer: B Robbins 6th ed pp. 627-630 MPL 1

96. A 30-year-old man known to be a heavy smoker developed gangrene of the leg, which had
to be amputated. Intraluminal thrombi associated with microabscesses in the wall of
medium sized arteries were found in the resected leg. Diagnosis?
A. Henoch-Sholein purpura
B. Syphilis
C. Polyarteritis nodosa
D. Buerger disease
96. Answer: D Robbins 6th ed pp.523 MPL 1

97. Ocular muscle weakness and general fatigability were found in a 38-year-old woman who
also had antibodies to acetylcholine receptors. Diagnosis?
A. Graves disease
B. Polymyositis
C. Myasthenia gravis
D. Wiscott-Aldrich syndrome
97. Answer: C Robbins 6th ed pp.1289 MPL 1

98. Congenital aplasia of the thymus and parathyroid glands was discovered in a neonate that
developed spastic contraction on the second postpartum day. Diagnosis?
A. Goodpasture syndrome
B. DiGeorge syndrome
C. Isolated IgA deficiency
D. Brutons agammaglobulinemia

91
PATHO BOARD EXAM QUESTIONS
98. Answer: B Robbins 6th ed pp.235 MPL 1

99. Hemorrhagic cutaneous nodules in a male homosexual with AIDS. Diagnosis?


A. hemangioma
B. lentigo maligna
C. neurofibroma
D. Kaposi sarcoma
99. Answer: D Robbins 6th ed pp535 MPL 0.33

100. Short female with web neck and amenorrhea. Her karyotype was 45,X. Diagnosis?
A. Turner syndrome
B. Klinefelter syndrome
C. Down syndrome
D. Fragile-X syndrome
Answer: A Robbins 6th ed pp.174-176 MPL 0.33

92
PATHO BOARD EXAM QUESTIONS

Das könnte Ihnen auch gefallen